Sunteți pe pagina 1din 125

Universidad La Salle.

Facultad Mexicana de Medicina.


Curso de Extensin Universitaria para la Preparacin del Examen Nacional para
Aspirantes a Residencias Mdicas.
Examen de Gineco-obstetrcia.



Nombre: Examen del Mdulo I.
Nmero de intentos: 3.
Vigencia: 26 de Febrero del 2014.
Horario: 7:00 a 9:00 p.m.
Programar aleatorio. (4 bloques con 20 preguntas cada uno).


1.- Se trata de femenino que acude a consulta externa con diagnstico de
amenorrea, acude con resultados de laboratorio los que reportan los siguientes
niveles hormonales: GnRH elevada, FSH y LH elevadas, hormonas ovricas
(estrgenos y progesterona) bajas. El defecto est a nivel de:

a) Hipotlamo.
b) Ovario
c) Hipfisis.
d) Endometrio.

Evaluacin de la paciente con amenorrea secundaria
La mayora de las pacientes con AS que no estn embarazadas o no entraron en el
climaterio tienen una alteracin en algn nivel de la cascada reguladora del ciclo
menstrual femenino. A continuacin, esquematizamos la evaluacin de la AS en
pasos. Cada mdico deber adaptar los tiempos de la evaluacin a la situacin
particular de cada paciente. No obstante, recomendamos no saltear ninguno de
estos pasos ya que un diagnstico preciso permitir realizar un tratamiento
racional y dar un pronstico respecto de la futura funcin menstrual y de la
fertilidad.





Primer paso (descartar el embarazo)

Su prueba se basa en el dosaje cuali o cuantitativo de la subunidad beta de la
gonadotrofina corinica humana. Si se certifica la ausencia de embarazo podr
avanzarse al siguiente paso, pero teniendo en cuenta siempre que si la probabilidad
de embarazo es muy alta se deber repetir la prueba nuevamente. Se recomienda
no obviar esta prueba aunque la paciente asegure que no ha mantenido relaciones
sexuales.
Segundo paso (prueba de progesterona)

Debe realizarse slo si el test de embarazo es negativo y consiste en dar
medroxiprogesterona entre 30 a 50mg por va oral (un comprimido de 10mg
durante 5 das) o progesterona oleosa 100 a 200mg por va intramuscular en una
sola dosis. El resultado de esta prueba provee informacin acerca de si se
produjeron estrgenos. La prueba se considera positiva si se produce un sangrado
luego de 2 a 14 das de la suspensin de la progesterona. La respuesta positiva
indica que existe integridad anatmica del aparato genital femenino y que el ovario
produce estrgenos. En estos casos, la AS se debe a que no ha habido ovulacin. La
causa ms frecuente de AS con prueba de progesterona positiva es la disfuncin
hipotalmica leve. En este caso, no se dispara el pico de LH necesario para producir
la ovulacin, no se ovula (ciclo anovulatorio), no hay cuerpo lteo y no hay
produccin de progesterona. El sistema reproductor queda en un estado folicular,
con grados variables de desarrollo folicular, acompaados de concentraciones
cambiantes de estradiol. Puede producirse hemorragia en ausencia de ovulacin
dado que el endometrio proliferativo sufrir ocasionalmente un desprendimiento
parcial en respuesta a estos niveles cambiantes de estradiol. Estos sangrados luego
de un ciclo anovulatorio se denominan sangrados uterinos disfuncionales (SUD).
Como dijimos, ejemplos frecuentes de situaciones de anovulacin son el estrs, la
prdida brusca de peso y el ejercicio intenso. Otras causas menos frecuentes son
la hiperprolactinemia, el hipotiroidismo y el sindrome del ovario poliqustico. Otras
veces no hay una causa clara (idioptica) de este trastorno.
La prueba se considera negativa si NO se produce un sangrado luego de 2 a 14 das
de la suspensin de la progesterona, lo que ocurre cuando los niveles de estrgenos
son bajos (menores a 40 pg/ml). La causa ms frecuente de prueba negativa es la
insuficiencia gonadal de la menopausia.



Tercer paso

Para avanzar al tercer paso es preciso distinguir si la prueba de progesterona fue
positiva o negativa.

Pacientes con prueba de progesterona positiva

Una prueba de progesterona positiva hace diagnstico de ciclos anovulatorios.
Segn los antecedentes, el examen clnico y la probabilidad previa el mdico podr
orientarse hacia cul es el diagnstico. Hay situaciones de estrs, crisis vitales,
cambios de peso brusco o ejercicio extremo que ocasionan un trastorno del ciclo
aislado. En estos casos, la prueba de progesterona es diagnstica y teraputica. Es
decir, la paciente no requiere estudios posteriores. Es conveniente reasegurarla y
explicarle que su periodo se normalizar una vez solucionado el problema. En una
paciente con trastornos anovulatorios frecuentes, que presenta obesidad, acn e
hirsutismo, existe alta sospecha de sndrome de ovario poliqustico (SOP). En este
caso, debemos solicitar una ecografa pelviana y un dosaje de LH/FSH o de
hormonas masculinas: testosterona libre (To), dehidroepiandrosterona sulfato
(DHEA-S) y 17OH progesterona.
Si la paciente tiene trastornos del ciclo y galactorrea, se debe solicitar un dosaje
de prolactina (PRL). El valor normal de PRL para la mujer no embarazada es de 20 a
25ng/ml. Cuando el valor de PRL es menor de 100ng/ml, generalmente es de causa
idioptica o farmacolgica; si est entre 100 y 200ng/ml, puede ser por causa
farmacolgica o por enfermedad hipotalmica; si es mayor de 200ng/ml, la causa
ms probable es el adenoma hipofisario. Cuando el valor de la PRL es mayor de
100ng/ml, se debe solicitar un estudio por imgenes que puede ser una tomografa
computada de cerebro con contraste o una resonancia magntica nuclear con
gadolinio. Si se observa una imagen menor de 10mm, se trata de un microadenoma
de hipfisis y si es mayor de 10mm, se hace diagnstico de macroadenoma. Si la
paciente consulta por trastornos del ciclo frecuentes y no presenta galactorrea o
signos o sntomas de hipotiroidismo, igualmente deber solicitarse un dosaje de
tirotrofina (TSH) y de prolactina (PRL) sricas.
Pacientes con prueba de progesterona negativa

Deber dosarse el nivel srico de FSH cuyo valor refleja los niveles de estrgenos
circulantes (por el fenmeno de retroalimentacin negativa). Si la FSH es mayor de
40 UI/ml, los niveles de estrgenos son bajos. En este caso, se asume que existe
una insuficiencia ovrica. El valor de la FSH puede hacer el diagnstico diferencial
entre la falla ovrica (castracin temprana, menopausia precoz o menopausia

normal) y la disfuncin hipotlamo-hipofisaria severa. En la primera, la ausencia de
retroalimentacin negativa estimula la liberacin de las gonadotrofinas y, en
consecuencia, stas se encuentran elevadas en la sangre. En la segunda, los valores
de gonadotrofinas son bajos debido a un defecto en su produccin. El dosaje de
FSH es ms sensible que el de LH para este propsito, por lo tanto, si la prueba de
progesterona es negativa y la FSH es baja, lo ms probable es que la causa sea una
disfuncin hipotlamo-hipofisaria severa. En este caso debera solicitarse un
estudio de diagnstico por imagen del cerebro (RMN con gadolinio o TAC con
contraste). En el cuadro 4 se muestran los valores normales de las hormonas
mencionadas.
Cuadro 4: valores hormonales normales
PRL (prolactina): 20 a 25ng/ml en mujeres no embarazadas.
TSH: de 0.5 a 5mUI/ml.
FSH: 5 a 30mUI/ml.
LH: 5 a 20mUI/ml (en el pico ovulatorio este valor se debe multiplicar por dos o
tres).
Falla ovrica: FSH mayor de 40mUI/ml.
Disfuncin hipotlamo hipofisaria: FSH menor a 5mUI/ml y LH menor a 5mUI/ml.
En general, una paciente con disfuncin hipotalmica con gonadotrofinas bajas est
expresando una gravedad mayor que la disfuncin hipotalmica leve con prueba de
progesterona positiva.

En los casos en los que la prueba de progesterona es negativa, la FSH es normal, la
paciente no est en una edad cercana a la menopausia y existe alta sospecha de que
exista una enfermedad uterina, se recomienda realizar una prueba de estrgenos
y progesterona. Esta prueba sirve para conocer si el efector (el tero) responde a
los estmulos hormonales. Consiste en administrar estrgenos (1.25mg por da
durante 20 das) ms progesterona (10mg por da durante los ltimos 5 das en que
se administran los estrgenos). Si no hay sangrado, se debe repetir la prueba y, si
nuevamente no hay sangrado, la prueba se considera negativa. Esto indica que hay
una falla en el efector, o sea, en el tero.

En este caso la paciente debe ser derivada al gineclogo. Las causas ms probables
de falla uterina son la endometritis por abortos o partos spticos y las sinequias
por curetajes vigorosos. Cuando falla el efector, la ecografa transvaginal puede
ser normal. Por eso debe realizarse una prueba de estrgenos y progesterona para
evaluar directamente la funcionalidad del tero. La prueba se considera positiva si
hay sangrado (aunque sean slo gotitas). Esto significa que el tero est indemne
para responder a estmulos hormonales y el problema es de origen hipotlamo-
hipofisario.




1. Jonathan R, Pletcher, Gail B. Slap. Menstrual Disorders Amenorrhea.
Pediatric Clinics of North America. June 1999; 46: Issue 3.
2. Bryan McIver, Susan A Romanski; Todd B Nippoldt. Evaluation and
Management of Amenorrhea. Mayo Clinic Proceedings Dec 1997; 72: 1161-
1169.
3. Tarannun Master Hunter, Diana L.Heiman. Amenorrhea: Evaluation and
Treatment. Am Fam Physician 2006; 73: 1374-1382.
The Practice Committee of the American Society for Reproductive Medicine.
Current Evaluation of Amenorrhea. Fertility and Sterility Nov 2006: 86 Supl 4:
148-155.



2.- Se trata de femenino de 22 aos de edad, refiere que desde hace un par de
meses ha presentado hemorragia irregular o postcoital, actualmente presenta
disuria y dolor abdominal usted debe sospechar en cervicitis por:



a) b) Micoplasma
b) Gardnerella
c) Chlamydia
d) Candida


Segn los datos de la Organizacin Mundial de la Salud, anualmente se detectan 89
000 000 de nuevas infecciones por Chlamydia trachomatis en el mundo. Esta
infeccin provoca uretritis y cervicitis, y las secuelas incluyen enfermedad
inflamatoria plvica, embarazo ectpico, infertilidad por dao tubrico,
epididimitis, proctitis y artritis reactiva. Se considera principalmente un problema
de salud en la mujer, en ella las manifestaciones y consecuencias son ms dainas
para la salud reproductiva. Los individuos infectados con Chlamydia trachomatis
pueden portar el microorganismo por meses o aos y transmitir la enfermedad a
sus parejas sexuales. Su diagnstico sigue siendo un reto, ya que quienes la
padecen presentan sntomas muy leves o son portadores asintomticos.




Manifestaciones clnicas

La cervicitis es la manifestacin clnica ms frecuente de la infeccin por C.
trachomatis en la mujer. Sin embargo, el 70% de las mujeres infectadas no tienen
sntomas, mientras que en el tercio restante las evidencias clnicas son poco
especficas de infeccin, como flujo genital, dolor abdominal o pelviano, sangrado
y/o disuria.

La presencia de disuria puede indicar una uretritis acompaante, lo que sucede en
el 35% de los casos. En otras oportunidades, solo la uretra est comprometida, y la
infeccin uretral se manifiesta como piuria o disuria con cultivo negativo (23% de
los casos).

El diagnstico se realiza al examinar el hisopado endocervical, que muestra flujo
amarillento o verdoso con ms de 10 PMN por campo de inmersin en el examen de
Gram. Este resultado define la cervicitis mucopurulenta (CMP) la cual tambin
puede ser producida en casos de infeccin por gonococo o mixta (C. trachomatis y
gonococo). Por lo tanto, el diagnstico debe confirmarse mediante estudios de
mayor especificidad, como las tcnicas moleculares (test de ligasa, PCR), que
tienen una sensibilidad del 96% aproximadamente, o la deteccin del antgeno por
tcnica de ELISA, con una sensibilidad del 75%. Tambin se ha demostrado que las
tcnicas moleculares en el primer chorro de orina son especficas y altamente
sensibles.
Cuatro de cada diez mujeres con cervicitis no tratada adquieren enfermedad
inflamatoria pelviana (EPI), con mayor riesgo de sufrir embarazo ectpico,
infertilidad y dolor crnico pelviano. El riesgo de infertilidad se eleva segn el
nmero e intensidad de los episodios: alrededor del 10% despus de un episodio,
del 30% despus de dos, y mayor del 50% si ha habido tres o ms episodios. Por
otro lado, el embarazo ectpico es cinco a siete veces ms frecuente cuando se
trata de pacientes con antecedentes de EPI.


Tratamiento

CLAMIDIA TRACHOMATIS
Azitromicina 1 g VO dosis nica
Doxiciclina 100 mg VO cada 12 horas por 7 das
Eritromicina 500 mg VO cada 6 horas por 7 das
Ofloxacina 300 mg VO cada 12 horas por 7das

Levofloxacina 500 mg VO cada 24 horas por 7 das


Basado en Guas Clnicas para el manejo de las ITS, OMS, 2003 / Norma Oficial
Mexicana NOM -039-SSA2-2002

Berek J. (2002) Ginecologa de NOVAK. Mxico. Ed. Mc Graw Hill Interamericana.
Pag 293.


3.- Se trata de femenino de 28 aos, con tumor anexial de 6 cm lquido, dolor
abdominal, fiebre, leucorrea, con historia de cervicovaginitis de repeticin y
dispareunia crnica, ltima menstruacin hace una semana. El diagnstico ms
probable es:

a) Cistadenoma
b) Embarazo ectpico
c) Quiste de ovario
d) Enfermedad plvica inflamatoria

La EIP puede cursar con los siguientes sntomas:
Dolor abdominal bajo (incluyendo dolor anexial, dispareunia). Es el sntoma
ms frecuente (95%)
Aumento del flujo vaginal, flujo de caractersticas anormales (74%)
Sangrado anormal (intermestrual, poscoital) (45%)
Sntomas urinarios (35%)
Vmitos (14%)
Es posible la ausencia de sntomas
Y en ella podemos encontrar estos signos:
Dolor a la movilizacin del cuello, dolor anexial en la exploracin vaginal
bimanual (99%)
En el examen con espculo observamos cervicitis y descarga endocervical
purulenta (74%)
Fiebre (> 38 C) (menos del 47%).
Masa plvica: sugiere abceso tuboovrico (ATO)
Peritonitis

CRITERIOS CLINICOS PARA EL DIAGNOSTICO DE SALPINGITIS

a. Dolor abdominal con o sin rebote.
b. Sensibilidad a la movilizacin del crvix.
c. Sensibilidad anexial.
Los tres criterios anteriores son necesarios para establecer el diagnstico, con
uno o ms de los siguientes:
a. Extendido de Gram de endocrvix positivo, para diplococos gram
negativos intracelulares
b. Temperatura mayor de 38C
c. Leucocitosis (mayor de 10.000 por c.c.)
d. Material purulento (positivo para leucocitos) en la cavidad peritoneal
obtenido por culdocentesis o laparoscopia.
Establecido el diagnstico clnico de EPI, se debe hacer la definicin del estado
clnico y anatmico de la patologa plvica:
a) No complicada (limitada a trompas u ovarios)
1) Sin peritonitis plvica
2) Con peritonitis plvica
b) Complicada (masa inflamatoria o absceso que compromete trompa (s) u ovario (s)
1) Sin peritonitis plvica
2) Con peritonitis plvica
Beigi RH, Wiesenfeld HC. Pelvic inflammatory disease: new diagnostic criteria and
treatment. Obstet Gynecol Clin Norh Am. 2003; 30 (4): 777 93
Center for Disease Control. Guidelines for treatment of sexually transmited
diseases. MMWR Recomm Rep. 2002 May 10;51(RR-6):1-78
Center for Disease Control. Guidelines for prevention and management (MMWR.
40: 1 - 25 1991) Pelvic inflammatory disease: guidelines for prevention and
management.
MMWR Recomm Rep. 1991 Apr 26;40(RR-5):1-25.
Hager WD, Eschenbach DA, Spence MR, Sweet RL. Criteria for diagnosis and
grading of salpingitis. Obstet Gynecol. 1983 Jan;61(1):113-4.
Prodigy Guidance. Pelvic inflammatory disease. [Internet]. UK : NHS, Department
of Health; 2003. [Acceso 18 de Junio de 2005]. Disponible en:
Ross J. Pelvic inflammatory disease. Clin Evid. 2004 Dec;(12):2259-65.
Royal College of Obstetricians and Gynaecologists. Pelvic Inflammatory Disease.
Guideline n 32. [Internet]. RCOG; Mayo 2003. [Acceso 18 de Junio de 2005].



4.- Femenino de 33 aos que cursa con 38.5 semanas de gestacin que ingresa al
servicio con trabajo de parto. En el transcurso de trabajo de parto durante la
dilatacin presenta dolor intenso y brusco. A la exploracin usted observa
metrorragia escasa y aumento del tono uterino a la palpacin abdominal que resulta
muy doloroso. El diagnstico ms probable es:

a) Desprendimiento de placenta.
b) Placenta previa.
c) Rotura de vasos previos.
d) Crioamnionitis hemorrgica.


Fisiopatologa de la hemorragia
La hemorragia es el signo fundamental que domina el cuadro clnico de la placenta
previa. Para explicar su mecanismo existen distintas teoras:

Mecanismo de Jacquemier: Se produce crecimiento armnico de la placenta y del
tero hasta la semana 26, 28. Despus el segmento inferior crece ms deprisa y
favorece el despegamiento lo que origina la hemorragia en el embarazo.
Mecanismo de Schroeder: Las contracciones uterinas en el parto traccionan del
segmento inferior hacia arriba y empujan al feto hacia abajo despegando la
placenta.
Mecanismo de Pinard: Explica las hemorragias gestacionales y del parto. El
estiramiento de las membranas de la zona de menor radio (orificio interno cervical)
como consecuencia de las contracciones tira de la placenta y la desprenden.
Mecanismo de Bartholomew: Explica la hemorragia en los casos de placenta previa
central. La zona placentaria que reviste el orificio interno es un rea isqumica ya
que no recibe vasos deciduales. A este nivel disminuye la presin sangunea, por lo
que la sangre tiende a dirigirse hacia esta zona y escapa por la cara materna.

En el alumbramiento tambin puede haber una hemorragia importante producida
por un doble mecanismo:
-desprendimiento parcial antes de la expulsin en los casos de placenta oclusiva.
-atona uterina en la zona de insercin despus de expulsada la placenta y
vascularizacin anmala.

Manifestaciones clnicas de la placenta previa en el embarazo

- Sntomas: Principalmente la hemorragia. Toda hemorragia vaginal acontecida en el
tercer trimestre debe hacer pensar en una placenta previa. Las hemorragias suelen
ser espontneas, no acompaadas de dolor, de sangre roja y se presentan de forma
intermitente, con intervalos variables entre las mismas. Progresivamente se van

haciendo ms frecuentes y ms graves. La primera hemorragia suele aparecer en
forma inesperada generalmente nocturna, cesando en menos de media hora. Las
hemorragias ulteriores son ms graves y ms precoces.

- Signos: La consecuencia fundamental es la anemia materna que depende de la
cuanta de la hemorragia (la sangre es de origen materna ya que procede de
espacios intervellosos).

- Exploracin:
Exploracin general para valorar la existencia de signos de anemia.
Exploracin obsttrica: valorar el tamao del tero (adecuado para la edad
gestacional), es blando e indoloro. A menudo la esttica fetal est alterada
(transverso, oblicuo, nalgas). No debe efectuarse tacto vaginal cuando haya
existido hemorragia en embarazo avanzado por el riesgo de infeccin y de
despegamiento y aumentar as la hemorragia.
Auscultacin fetal normal.
Exploracin ecogrfica: es una tcnica fundamental en el diagnstico de la placenta
previa. Permite determinar la localizacin placentaria y la variedad de la placenta
previa. En general, la placenta puede identificarse a partir de la 9 semana. No
obstante, a lo largo de la gestacin por crecimiento uterino se produce un cambio
en sus relaciones con el tero ("emigracin placentaria", imagen de
desplazamiento). As el diagnstico de certeza de lmites placentarios solo puede
establecerse hacia la semana 34. Siempre ser necesario hacer una adecuada
identificacin del orificio cervical interno (ms fcil con sonda transvaginal).
Aadiendo al estudio Doppler color se observa la vascularizacin y las zonas que
sangran.

5.- Mujer de 26 aos, es atendida en consulta en la clnica de displasias por
papanicolau con lesin NIC I. Antecedentes: menarca 14 aos, ritmo 30x5
eumenorreica, inicio de vida sexual a los 15 aos, 2 parejas sexuales, mtodo de
planificacin familiar oclusin tubaria bilateral, gestas 3 partos 3, crvix con lesin
acetoblanca con extensin lineal de 2 cm. Para confirmar el diagnstico se debe
realizar:



a) Crioterapia de lesin.
b) Captura de hbridos.
c) Repetir colposcopa
d) Biopsia de la lesin.










9.5.2 Las pacientes a quienes se les realiz citologa cervical, cuyo resultado es
LEIBG (infeccin por VPH, displasia leve o NIC 1); LEIAG (displasia moderada y
grave o NIC 2 y 3) o cncer deben enviarse a una clnica de colposcopa, para
realizar estudio colposcpico.

9.5.3 Si el resultado de la citologa es LEIBG, la colposcopa es satisfactoria y sin
evidencia de LEIBG, se realizar control citolgico en un ao (Apndice Normativo
A)
9.5.4 Si la citologa es de LEIBG, la colposcopa es satisfactoria y existe
evidencia de lesin, se debe tomar una biopsia dirigida.
9.5.4.1 Si la biopsia dirigida es negativa, se realizar nueva colposcopa para
verificar el diagnstico y en caso necesario, tomar nueva biopsia dirigida y
revalorar.
9.5.4.2 Si la biopsia dirigida es reportada como LEIBG se podr dar tratamiento
conservador: criociruga, electrociruga o laserterapia (slo si cumple con las
condiciones referidas en el Apndice 1) o se podr mantener a la paciente en
vigilancia en la clnica de colposcopa, con colposcopa y estudio citolgico cada seis
meses, durante 24 meses.
Jueves 31 de mayo de 2007 DIARIO OFICIAL (Primera Seccin)
9.5.4.3 Si la biopsia dirigida es reportada como LEIAG (Lesin Intraepitelial
Escamosa de Alto Grado) se realizar tratamiento conservador (electrociruga o
laserterapia). En las mujeres posmenopusicas, dependiendo de las condiciones
anatmicas del crvix, se realizar tratamiento conservador en la clnica de
colposcopa o tratamiento quirrgico (histerectoma extrafascial) en el servicio que
corresponda.
9.5.4.4 Si la biopsia dirigida reporta cncer microinvasor o invasor, la paciente se
transferir a un Servicio o Centro Oncolgico para su tratamiento
correspondiente.
9.5.4.5 Si la citologa reporta LEIBG y la colposcopa es no satisfactoria, se
tomar cepillado endocervical (Apndice Normativo A)
9.6 En caso de colposcopa no satisfactoria, negativa a LEIBG y con cepillado
endocervical negativo, se continuar su control en la clnica de colposcopa en seis
meses, con colposcopa y citologa.
9.6.1.1 Si el cepillado endocervical reporta LEIBG se tratar a la paciente como
LEIAG, con mtodos conservadores escisionales.



Jueves 31 de mayo de 2007 DIARIO OFICIAL (Primera Seccin)
Modificacin a la Norma Oficial Mexicana NOM-014-SSA2-1994, Para la
prevencin, deteccin, diagnstico, tratamiento, control y vigilancia
epidemiolgica del cncer crvico uterino.
Al margen un sello con el Escudo Nacional, que dice: Estados Unidos Mexicanos.-
Secretara de Salud.
MODIFICACION A LA NORMA OFICIAL MEXICANA NOM-014-SSA2-1994,
PARA LA PREVENCION,
DETECCION, DIAGNOSTICO, TRATAMIENTO, CONTROL Y VIGILANCIA
EPIDEMIOLOGICA DEL CANCER CERVICO UTERINO.



7.3 El resultado del estudio citolgico es descriptivo y debe ser informado de la
siguiente manera:
a.- Negativo a cncer.
b.- Negativo con proceso inflamatorio.
c.- Displasia leve (NIC 1).
d.- Displasia moderada (NIC 2).
e.- Displasia grave (NIC 3).
f.- Cncer del cuello del tero in situ (NIC 3).
g.- Cncer microinvasor e invasor.
h.- Adenocarcinoma.
i.- Maligno no especificado.




6.- Se trata de mujer de 36 aos, nulpara tras 2 aos de relaciones sexuales sin
contracepcin, que desde hace 1 ao presenta dismenorrea, dispareunia y sangrado
vaginal intermenstrual. Cul sera la primera orientacin diagnstica?

a) Insuficiencia lutenica.
b) Endometriosis.
c) Enfermedad inflamatoria plvica.
d) Dismenorrea funcional.

La endometriosis consiste en la aparicin y crecimiento de tejido endometrial
fuera del tero, sobre todo en la cavidad plvica como en los ovarios, detrs del
tero, en los ligamentos uterinos, en la vejiga urinaria o en el intestino. Es menos
frecuente que la endometriosis aparezca fuera del abdomen como en los pulmones
o en otras partes del cuerpo.
La endometriosis es una enfermedad relativamente frecuente, que puede afectar a
cualquier mujer en edad frtil, desde la menarquia hasta la menopausia, aunque
algunas veces, la endometriosis puede durar hasta despus de la menopausia. La

endometriosis altera la calidad de vida de las mujeres que la padecen, afectando a
sus relaciones de pareja, familiares, laborales y de reproduccin.

Sntomas
Los sntomas clsicos son la dismenorrea, dolor plvico, dispareunia, sangrados
intermestruales y en muchos casos, esterilidad.
El dolor no tiene que ver con el tamao y la severidad de la lesin; generalmente
cuanto menor es la lesin mayor dolor produce. El dolor se agrava con las
menstruaciones y en los casos en que la lesin ocupa el fondo de saco de Douglas,
puede dar dispareunia. Existe un aumento de la PGF2 alfa y PGE2 y un aumento de
las contracciones uterinas que podra deberse a un depsito de endometrio en la
cavidad peritoneal.
La esterilidad debido a la endometriosis podra deberse a distintas causas de
acuerdo a la severidad de la patologa. En los casos de endometriosis severa puede
haber un factor tuboperitoneal con adherencias y alteracin en la anatoma de la
pelvis que interfiera con el transporte del esperma y el vulo. En los casos de
endometriosis leve hay varios mecanismos propuestos que justifican su relacin con
la infertilidad: foliculognesis alterada, fase ltea inadecuada, fagocitosis
espermtica, mala calidad ovocitaria, embriotoxicidad y alteracin a nivel de la
implantacin.. La produccin de prostaglandinas por el endometrio ectpico puede
afectar la motilidad tubaria, la foliculognesis y la funcin del cuerpo lteo. Puede
haber un aumento de la activacin de los macrfagos peritoneales en la
endometriosis que cause la fagocitosis de los espermas o la secrecin de citoquinas
que pueden ser txicas para el embrin. Segn algunos investigadores habra un
60% de las mujeres con endometriosis que presentan un sndrome de Folculo
Luteinizado no roto (LUF) en el cual el folculo no se rompe en la ovulacin y el
vulo queda atrapado.




Referencias bibliogrficas

1. Ruiz V. Endometriosis y fertilidad. Ed. Acosta y Warman, pp. 99
2. Lpes, VH. Palomo E. Incidencias de endometriosis en una poblacin infrtil. XXI
Congreso nacional de Ginecologa y Obtetricia. Guatemala, 1993.
3. El-Eoley, et al. Danazol but not ginadotropin releasing hormone agonists
suppresses autoantibodies in endomeriosis. Fertil Steril 1990; 54:725
4. Acosta AA. Buttram VC Jr. Besch PK, Malinak LR, Van Der Heyden J.
A.proposed classfication of pelvic endometriosis. Obstet Gynecol 1973;42:19.
5. Buttran VC Jr. Evolution of the revised American Fertility classification of
endometriosis. Fert. Steril 1985; 43: 347
6. Lpez VH. Tratamiento mdico-quirrgico de la endometriosis. Simposio El
rostro cambiante de la endometriosis panam 3. 12. 1993.
7. Steinleitner A. Heterolous transplation of activated murine peritonel
macrophages inhibitis gamete interaction in vivo; A paradigm fo endometriosis
associted subfertility. Fertil Steril 1990; 54:725.
8. Damewood M. Effect of serum from patients with minimal to mild endometriosis
on mouse embryo growth. Fertil Steril 1990; 54: 917
9. Proug S. Peritoneal fluid fracctions from patients with endometriosis do not
promote two-cell mouse embryo growth. Fertil Steril 1990; 54: 927.




7.- Mujer de 25 aos, cursando su 14 semana de gestacin, acude a consulta por
referir tenesmo vesical, disuria y escalofro. Por su estado actual, cul de los
siguientes antimicrobianos es el ms indicado?


a) Metronidazol
b) Tetraciclina
c) Ampicilina
d) Levofloxacina



El Metronidazol no ha mostrado efectos txicos en humanos, pero es teratognico
en modelo animal. Las tetraciclinas ocasionan coloracin anormal de los dientes,
hepatotoxicidad y alteracin en el desarrollo de huesos. Las sulfas podran tener
un efecto deletreo en el primer trimestre dada su actividad como
antimetabolitos, y en los ltimos meses pueden favorecer kernicterus en el recin
nacido si es que tiene alteraciones metablicas que favorezcan anemia hemoltica.
Las quinolonas se han asociado a malformaciones seas en modelos animales, y se
recomienda evitarlas si existen mejores opciones.

Kasper DL, Braunwald E, Fauci AS, Hauser SL, Longo DL, Jameson JL.
Harrisons Principles of Internal Medicine. McGraw Hill. 16 Ed. 789-806 pp.


8.- Femenino de 25 aos, con embarazo de trmino, sin antecedentes de control
prenatal. G 3. C-1. Se ingresa al servicio de obstetricia por presentar actividad
uterina regular y dolorosa. Ef.: Deambulante, tranquila, adecuada coloracin de
tegumentos, abdomen con fondo uterino a 32 cm. con producto nico vivo en
situacin transversa dorso inferior FCF 144, al tacto vaginal crvix dilatado a 3
cm. y membranas ntegras. Se realiza cesrea con retencin de placenta e invasin
a vejiga.
Para obtener el diagnstico, el estudio de gabinete de eleccin previo al evento
obsttrico es:

a) Prueba sin estrs
b) Perfil biofsico
c) Simple de abdomen
d) Ultrasonido doppler


PLACENTA:
Se divide en:
Placenta acreta.
Placenta increta.
Placenta percreta.

ACRETA: Las vellosidades se adhieren al miometrio.


INCRETA: Penetran ms de la mitad del espesor del
miometrio.

PERCRETA: Atraviesa todo el espesor del miometrio, llegando
a la serosa, incluso atravesndola y adhirindose a rganos
vecinos.



Factores:
Endometrisis previa.
Tumores submucosos. (Miomas)
Cicatrz uterina previa. (Cesrea, miomectoma)
Implantacin baja. (Placenta previa)
Malformaciones placentarias. (Placenta extracorial)
Legrado enrgico previo.
Extraccin manual previa de una placenta.
Diagnstico Pre-parto

Ultrasonografa Dppler.

Resonancia Magntica.

Diagnstico transparto-:
Placenta retenida por ms de 20 minutos.
Imposibilidad para encontrar un plano de separacin placentaria cuando se
intenta su extraccin manual.
Hemorragia incontrolable despus de la pseudoextraccin.
El diagnstico histopatolgico corrobora el diagnstico clnico.
Escenario menos deseable.

Tratamiento:
Histerectoma Obsttrica.
Constituye una ciruga no planeada y secundaria al hallazgo del
acretismo placentario con sangrado incohercible.

Cesrea-Histerectoma. (Con diagnstico previo)
Ciruga planificada ante un correcto diagnstico prenatal.

Recomendacin ACOG:
Maduracin pulmonar intrauterina.
Inyectar al cordn umbilical 50 mg de metrotexate.
Ligar el cordn en el nacimiento placentario y dejar la placenta in-
situ.

Embolizacin inmediata de arterias uterinas bilaterales, as como de
ramas de la divisin anterior de la arteria iliaca interna con alcohol
polivinlico.
Continuar con 5 dosis I.M. de 50 mg de metrotexate y cuantificar
niveles de hCG.
Programar Histerectoma Total Radical Abdominal y/o Cistectoma
parcial y/o reseccin pared anterior recto.


Lee et al. Conservative Management of Placenta Percreta. Obstet Gynecol,
112(2):421-424




9.- Femenino de 40 aos con DM acude a consulta quejndose de descarga vaginal
prurtica y blanquecina, cual de los siguientes exmenes de diagnsticos es mas
til para identificar el patgeno?

a) Wet prep.
b) KOH
c) Tincion de Gram
d) PH



Prueba del KOH es un procedimiento en el cual hidrxido del potasio (KOH) se
utiliza detectar hongos disolviendo las clulas humanas en una cultura. La
diferencia en la composicin de la pared de clula de clulas humanas y de clulas
fungicidas permite que este procedimiento ayude a distinguir las dos clulas. El
KOH desnaturaliza las protenas en la clula humana; solamente sigue habiendo las
clulas fungicidas ser considerado debajo del microscopio.
1. Bernal B. Fisiologa y ecologa de la vagina. Rev Chil Obstet Ginecol 1986; 51:56-
60.
2. Ibrcena E. Vaginosis bacteriana; diagnstico y prevalencia. XII Congreso
Peruano de Obstetricia y Ginecologa 1996; 204-6.
3. Scapini JC, Guzmn CA. Deteccin de bacilos Gram negativos curvos anaerobios
en pacientes con vaginosis. Obstet Ginecol Latinoam 1986; 44: 320-5.
4. Soihet S. El flujo vaginal en la consulta ginecolgica. Ginecol Obstet (Per) 1986;
30: 50-60.

5. Linaldi CAS, Urbina JR, Castaeda JL. Vaginitis por Gardnerella vaginalis en
nias y adolescentes. Bol Med Infant Mex 1988; 45: 101-3.
6. Sobel J. Vaginitis and vaginal flora. Controversies abound. Current Opin Infect
Dis 1996; 9: 42-7.
7. Faro S. Leucorrea. Causas infecciosas e imbalances en el ecosistema vaginal.
Tribuna Mdica. Cahners Healthcare ed 1996; 1 10.



10.-Paciente femenino de 26 aos que acude a consulta, cursa el tercer trimestre
del embarazo, presenta edema en miembros inferiores sin sintomatologa agregada,
usted indica a la paciente:


a) Restriccin Hdrica
b) Tiacidas
c) Elevacin de los miembros inferiores en decbito lateral
d) Diurtico de asa


DeCherney A. (1999) Diagnstico y tratamiento ginecoobsttricos. Mxico. Ed.
Manual Moderno. Pag 237. El edema de las partes bajas producido por la
impedancia del retorno venoso es comn en la parte final del embarazo, La paciente
debe tratarse solo si est molesta. La elevacin de los miembros inferiores
(especialmente en decbito lateral) mejora la circulacin. Estn contraindicados los
diurticos.


11.- Al encontrarse reparando una episiotoma media, realizada posterior a la
atencin de parto eutcico, una mujer de 24 aos de edad aprecia un marcado
incremento en el sangrado transvaginal. La causa ms probable de este fenmeno
es:

a) Atona uterina
b) Retencin de restos placentarios
c) Laceracin vaginal
d) Laceracin cervical



La mortalidad materna es un indicador de disparidad social y econmica. Cada ao
en todo el mundo mueren cerca de 600,000 mujeres, entre 15 y 49 aos. Como
resultado de complicaciones relacionadas con el embarazo, el parto y el puerperio.
Entre las causas principales destacan: hemorragia postparto (25 %), se psis (15%),
eclampsia (12 %) y labor prolongada o detenida (8 %).





DEFINICIN
Se define la hemorragia posparto (HPP) como la prdida sangunea de 500 mL. o
ms en las primeras 24 horas despus del parto o el descenso del hematocrito en
un 10 % o ms.
CLASIFICACIN
HPP Inmediata.- Prdida sangunea de 500 mL. o ms originada en el canal del
parto dentro de las 24 horas posteriores al parto.
HPP Tarda.- Sangrado anormal o excesivo originado en el canal del parto que se
presenta entre las 24 horas posteriores al parto y el final del puerperio (42 das).
FACTORES DE RIESGO
Se han descrito los siguientes factores de riesgo para la HPP:
Embarazo mltiple
Polihidramnios
Macrosoma
Trabajo de parto disfuncional
Gran multiparidad
Corioamnionitis
Uso inadecuado de oxitcicos
Endometritis en el puerperio
Prpura trombocitopnica
Anestesia general
Administracin previa y reciente de inhibidores uterinos
Placenta previa
Enfermedad de von Willebrand
Desprendimiento prematuro de placenta
Acretismo placentario
CAUSAS DE HEMORRAGIA POSPARTO
A continuacin se lisian las causas ms frecuentes de HPP. Segn su origen, se
dividen en uterinas y no uterinas:
2
Uterinas - No Uterinas
Hipotona o atona uterina
Retencin de placenta o restosplacentarios o membranas

Placentacin anormal (acretismo)
Inversin uterina
Traumatismo uterino (rotura uterina, desgarro cervical)
Laceraciones del canal del parto, incluyendo la episiotoma
Coagulopatas
Hematomas
CUADRO CLNICO
La HPP se caracteriza por los siguientes signos y sntomas:
Sangrado transvaginal de moderado a grave.
tero flcido (no contrado) o desgarros.
Alteraciones hemodinmicas que se manifiestan como: mareos, sudoracin,
nuseas, taquicardia y/o hipotensin arterial.
Oliguria.
tero nacido (no contrado).
Al considerar la HPP, deben diferenciarse dos tipos de situaciones: la prevencin
orientada a minimizar la probabilidad de que una mujer presente hemorragia tras
el parto y el manejo o tratamiento de la hemorragia, cuando sta ya se haya
producido

Referencias Bibliogrficas
1. AbdaRabbo SA: Stepwise uterine devascularization: A novel technique for
management of uncontrollable postpartum hemorrhage with preservation of
the uterus. Am J Obstet Gynecol 2003;171:694-700.
2. Bakri YN, Linjawi T: Angiographic embolization for control of pelvic genital
tract hemorrhage. Report of 14 cases. Acta Obstet Gynecol Scand
1992;71:17-21.
3. Bick RL: Disseminated intravascular coagulation. Objetive criteria for
diagnosis and manegement.- Med Clin N Am Vol 1994;78(3):511-43.
4. B-Lynch C, Coker A, Lawal A II, Abu J, Cowen MC: The B-Lynch surgical
technique for the control of massive postpartum haemorrhage: an
alternative to? Five cases reported. Br J Obstet Gynaecol 1997;104:372-5.




12.- Se trata de femenino 23 aos, G-1, cursa con embarazo de 37 semanas de
gestacin, presenta prdida del estado de alerta posterior a crisis convulsivas
tnico-clnicas, signos vitales con T-A 170.120mmhg Fc 95x, reflejos

osteotendinosos aumentados, se aprecia una Fc fetal de 132x y edema importante
de miembros inferiores, no se aprecian datos de trabajo de parto ni
modificaciones cervicales, El diagnstico ms probable es?



a) Pre eclampsia severa
b) Eclampsia
c) Crisis epileptica de gran mal
d) Hipertensin inducida por el embarazo



CUADRO 1. DIAGNSTICO*
Preeclampsia Leve: Se presenta despus de la semana 20 de gestacin, durante el
parto, o en las primeras 6 semanas despus de ste
Presin sistlica a 140 mm Hg o presin diastlica 90 mm Hg
Proteinuria a 300 mg / orina de 24 hrs o su equivalente en tira reactiva
Preeclampsia Severa: Se presenta despus de la semana 20 de gestacin, durante
el parto, o en las primeras 6 semanas despus de ste
Presin sistlica a 160 mm Hg o presin diastlica 110 mm Hg
Proteinuria a 2 gr en orina de 24 horas o su equivalente en tira reactiva
Creatinina srica > a 1.2 mg/dl
Trombocitopenia 150 000 cel/mm3
Incremento de la deshidrogenasa lctica a 600 UI
Elevacin al doble de los valores de TGO/AST o TGP/ALT
Cefalea, alteraciones visuales o auditivas
Epigastralgia
Oliguria a 500 ml en 24 horas 7
Edema agudo de pulmn
Dolor en hipocondrio derecho
Restriccin en el crecimiento intrauterino
Oligohidramnios
Eclampsia Preeclampsia mas convulsiones sin otra causa. Se presenta despus de la
semana 20 de gestacin, durante el parto, o en las primeras 6 semanas despus de
ste.
Sndrome de HELLP Criterios para establecer el diagnstico del sndrome de
HELLP:
Plaquetas < 100 000/mm3 TGO/AST 70U/L DHL 600U/LBilirrubina total > 1.2
mg/dl
Se presenta despus de la semana 20 de gestacin, durante el parto, o en las
primeras 6 semanas despus de ste.

Hipertensin Crnica: Se diagnostica cuando existe hipertensin arterial a
140/90 mm Hg antes de la semana 20 de gestacin o si persiste despus de doce
semanas posteriores al parto.
Las pacientes con hipertensin crnica deben ser evaluadas antes del embarazo
para determinar la severidad de la hipertensin y facilitar la planeacin de un
embarazo mediante el cambio de medicamentos y de hbitos higinicos y dietticos
para evitar complicaciones.
Hipertensin Gestacional: Presencia de hipertensin arterial a 140/90 mm Hg
despus de la semana 20 de gestacin y se mantiene hasta las doce semanas
despus del parto
Ausencia de proteinuria
Presencia o no de cefalea, acfenos y fosfenos
Despus de 12 semanas de la interrupcin del embarazo se revalorar la presencia
de hipertensin, si contina, se reclasifica como hipertensin crnica: es un
diagnstico retrospectivo. 8 Si no hay, se clasifica como hipertensin transitoria.

1. Aagaard-Tillery KM, Belfort MA. Eclampsia: morbidity, mortality, and
management. Clin Obstet Gynecol 48:12-23, 2005.

2. Atallah AN, Hofmeyr GJ, Duley L. Calcium supplementation during pregnancy for
preventing hypertensive disorders and related problems. Cochrane Database Syst
Rev 1:CD001059, 2001.

3. Barton JR, Sibai BM. Diagnosis and management of hemolysis, elevated liver
enzymes, and low platelets syndrome. Clin Perinatol 31:807-33, 2004.

4. Baxter JK, Weinstein L. HELLP syndrome: the state of the art. Obstet Gynecol
Surv 59:838-45, 2004.

5. Cetin A. Eclampsia. In Mohler III ER, Townsend RR. Advanced therapy in
hypertension and vascular disease. Ontario: B.C. Decker Inc. pp. 407-15, 2006.

6. Cetin A. Hemolysis, elevated liver enzymes, and low platelets (HELLP). In Mohler
III ER, Townsend RR. Advanced therapy in hypertension and vascular disease.
Ontario: B.C. Decker Inc. pp. 416-20, 2006.

7. Chappell LC, Seed PT, Briley AL, Kelly FJ, Lee R, Hunt BJ, Parmar K, Bewley SJ,
Shennan AH, Steer PJ, Poston L. Effect of antioxidants on the occurrence of pre-
eclampsia in women at increased risk: a randomised trial. Lancet 354:810-16, 1999.


13.- Mujer de 39 aos de edad acude a su consultorio refiriendo mastalgia que
es ms severa antes de la menstruacin. A la palpacin hay nodularidad excesiva,

hiperestesia y reas qusticas que la paciente refiere disminuyen en tamao
despus de la menstruacin. El diagnstico ms probable es:

a) Fibroadenomas
b) Papiloma intraductal
c) Cncer de mama
d) Mastopata fibroqustica



Es raro encontrar una mujer mayor de 35 aos a quien no le hayan dicho, en un
examen fsico mamario, ecogrfico o mamogrfico, que tiene quistes en la mama o
que su mama es mastoptica.


Es el trastorno benigno de la mama ms frecuente y consiste en un aumento del
tejido mamario, especialmente en las zonas superiores y externas de las mamas,
hacia las axilas, que las hace ms densas.
La mastopata fibroqustica suele presentarse en ambas mamas, aunque puede ser
de diferente intensidad en una que en otra.

Puede presentarse a cualquier edad despus del inicio de la menstruacin, pero es
ms probable que aparezca entre los 30 aos y la menopausia. Raramente se
presenta ms tarde de esa edad.

El origen de este trastorno es funcional y responde a desequilibrios de las
hormonas sexuales femeninas y puede condicionar la aparicin de quistes mamarios.

Los sntomas pueden fluctuar de leves a severos en una mastopata fibroquistica
mamaria, se acentan tpicamente antes de cada perodo menstrual y desaparecen
inmediatamente despus.
Los sntomas abarcan:

Consistencia de protuberancias (como de "guijarros"), irregular y densa del
tejido mamario
o generalmente ms notoria en la parte superior externa de la mama
Molestia en las mamas
o generalmente en ambas mamas
o puede ser persistente o puede aparecer y desaparecer
Sensacin de llenura en las mamas
Sensibilidad y dolor sordo e intenso
Sensibilidad y edema premenstrual
Secrecin ocasional del pezn




Bibliografa:
Jones III HW, Wentz AC. Tratado de Ginecologa de Novak. Editorial
Interamericana-McGraw Hill. Undcima Edicin 1994.
DiSaia-Creasman. Oncologa Ginecolgica Clnica. Editorial Mosby. Cuarta
Edicin 1994.
Van Dinh T. Sumario de Patologa Ginecolgica. Editorial La Prensa Mdica
Mexicana. 1992.
Pernoll ML. Diagnstico y Tratamiento Ginecoobsttricos. El Manual
Moderno. Mxico. Sexta Edicin 1991.
Alvarez-Bravo A. Diagnstico de los trastornos menstruales y
hemorrgicos. En: Alfonso Alvarez Bravo y su obra. Editorial Marketing y
Publicidad SA. Tomo I. 1993.
Vzquez E. Aspectos histoqumicos del endometrio humano despus del
tratamiento con progestgenos sintticos. Gac Md Mx 1966; 96: 1277-93.
Huerta MR, Malacara JM, Rivera-Cisneros A, Daz Cisneros FJ. Sntomas en
adolescentes de dos ciudades de Mxico y su asociacin con el ciclo
menstual. Ginec Obstet Mx 1994; 62: 146-50.


14.- Se trata de Femenino de 26 aos acude al servicio de consulta externa
refiriendo presentar baches amenorreicos, acn, hirsutismo y esterilidad de
aproximados 2 aos de evolucin E.F. con ndice de masa corporal 31 KG/M2, es
diagnstico ms probable es:



a) Hipotiroidismo.
b) Fallo ovrico precoz.
c) Sndrome del ovario poliqustico.
d) Amenorrea de causa uterina.


El sndrome de ovarios poliqusticos (SOPQ) afecta aproximadamente a un 4% de
mujeres en edad reproductiva y se caracteriza por anovulacin crnica e
hiperandrogenismo. Es la causa ms comn de infertilidad en mujeres.
Se caracteriza clnicamente por acn, alopecia, hirsutismo, irregularidades
menstruales e infertilidad.
Los hallazgos de laboratorio ms frecuentes son: aumento de la hormona
luteinizante (LH), aumento de la relacin LH/FSH (hormona folculoestimulante),
aumento de andrgenos (tanto ovricos como adrenales) y de estrgenos
circulantes. Otros hallazgos de laboratorio habituales son una prueba tolerancia
oral a la glucosa anormal y alteraciones en el perfil lipdico.
Todo esto junto con las imgenes ecocardiogrficas caractersticas definen al
sndrome.
La teraputica permite dos grandes enfoques que pueden superponerse: la
correccin de las manifestaciones de hiperandrogenismo y el tratamiento de las
alteraciones del eje reproductivo (anovulacin, esterilidad).
Los antiandrgenos estn fundamentalmente indicados para tratar los sntomas
virilizantes.
Las alternativas para inducir la ovulacin son numerosas: al citrato de clomifeno
y a la antigua reseccin en cua se agregan las gonadotrofinas humanas, pulsos de
GnRH (hormona liberadora de gonadotrofinas), medidas o frmacos para modificar
los niveles de insulina, y finalmente tcnicas quirrgicas endoscpicas para reducir
la masa ovrica.

Revista de Posgrado de la VIa Ctedra de Medicina - N 125 Marzo 2003
Pg. 37-40
SINDROME DE OVARIOS POLIQUISTICOS
Dra. Sandra Beneyto, Dra. Mara Andrea Ferreyra, Dr. Andrs Galfrascoli,
Dr. Andrs Gonzlez, Dra. Susana Sosa



15.- Se trata de femenino de 25 aos, Gesta 1, Para 1. Con dos citologas lesin
de alto grado, prueba de Schiller positiva y biopsia de crvix que demuestra
carcinoma In Situ. La conducta es:
a) Histerectoma total abdominal.
b) Histerectoma y salpingooforectoma bilateral.

c) Conizacin.
d) Electrocauterizacin del crvix.

La conizacin cervical es el tratamiento de eleccin en pacientes con cncer
cervicouterino microinvasor y ms si existedeseo de fertilidad. Asimismo, la
histerectoma extrafasciales un mtodo adecuado en lesiones de 0.5 a 3 mm de
invasin.Adems se propone que, para pacientes con lesiones de 3.1 a 5 mm de
invasin, a partir de la membrana inicial sinfactores de mal pronstico como
invasin vascular y linftica, sean tratadas con histerectoma extrafascial, ya que
en aquellasa las que se realiz linfadenectoma plvica, con este tipode lesin, no se
encontr metstasis a ganglios linfticos.


Resultados del tratamiento en cncer cervicouterino microinvasor en el Instituto
Nacional de Cancerologa de Mxico (1980-1999)
1.- Mestwerdt G. Fruhdiagnose des Kollumkarzinoms. Zentralb Gynaekol, 1947 ;69
:326.
2. - Morrow CP, Curtin JP. Surgery for cervical neoplasia. In Gynecologic Cancer
Surgery. New York, Churchill Livingstone, 1996, p 472.3.
3. - Burghardt E, Holzer E. Diagnosis and treatment of microinvasive carcinoma of
the cervix uteri. J Obstet and Gynecol 1977; 49:641-653.
4.- Sedlis A, Sall S, Tsukada Y, et al. Microinvasive carcinoma of the uterine
cervix: a clinical-pathologic study. Am J. Obstet Gynecol. 1979;133:64.




16.- A 6-year-old child presents with flesh-colored papules on the hand that are
not pruritic. Examination reveals lesions that are approximately 4 mm in diameter
with central umbilication. A halo is seen around those lesions undergoing
regression. Which of the following is the most likely diagnosis?


a) Verruca vulgaris
b) Keratoacanthoma
c) Herpetic whitlow
d) Molluscum contagiosum


El Molusco Contagioso es el nombre de una infeccin viral del grupo de los Poxvirus.
Es frecuente, transmisible, autoinoculable (uno mismo la puede trasmitir a
diferentes partes del cuerpo), se puede curar sola y es benigna.

Afecta principalmente a los nios menores de 10 aos, adultos sexualmente activos
y a pacientes inmunosuprimidos (con defensas bajas) como los pacientes con SIDA,
ocurriendo en un 5 18 por ciento. Afecta ms a hombres que a mujeres, y su
frecuencia aumenta tambin en climas tropicales y durante el verano.
CLINICA
El periodo de incubacin de la infeccin es de 14 a 50 das, aunque hay datos de
recin nacidos con lesiones al cabo de 7 das postparto.
Las lesiones se inician como neoformaciones que miden generalmente de 2 a 6
mm, aunque pueden llegar a medir 3 cm(13,14), son hemisfricas, cupuliformes,
lisas, del color de la piel o perladas, algunas (20%) tienen una umbilicacin
central(15); la base es levemente eritematosa y son de consistencia firme(13,14).
Se localizan en cualquier parte del cuerpo e incluso pueden afectar mucosas,
generalmente se agrupan en un rea especfica, pero pueden estar diseminadas en
personas infectadas con el virus de la inmunodeficiencia humana adquirida, siendo
un marcador de enfermedad avanzada (16). En el caso de los nios las lesiones se
localizan normalmente en cara, tronco, brazos y piernas a diferencia de los adultos
jvenes en quienes el molusco contagioso se adquiere por transmisin sexual, las
lesiones tienen predileccin por genitales, abdomen y cara interna de los muslos;
en otras series no hay diferencia (17).
Las lesiones del molusco suelen aparecer entre los 14 das a 6 meses despus de
la exposicin, hay datos de recin nacidos con lesiones al cabo de 7 das
postparto. Se pueden propagar por auto inoculacin, pero es tpico que se resuelvan
espontneamente en pocos meses.
Las lesiones son asintomticas en la mayora de los pacientes (13,18), aunque en el
10% de los casos puede haber prurito y desarrollarse una reaccin eccematosa
(14).
CRITERIO DIAGNSTICO
El diagnstico se hace clnicamente y en algunos casos dudosos puede efectuarse
biopsia con tincin H-E donde se encuentran los cuerpos de molusco ( inclusiones
intracitoplasmticas grandes) o de Hendersen-Paterson; el 90% de los pacientes
posee Ac tipo Ig G. Puede realizarse microscopa electrnica, PCR, Elisa, e
inmunohistoqumica.
REFERENCIAS
1. Schotz J, Rosen-Wolft A, Bugert J et al. Molecular epidemiology of molluscum
contagiosum. J Infect Dis 1988; 158: 898-900.

2. Porter CD, Nlake NW, Archard LC et al. Molluscum contagiosum virus type in
genital and non genital lesions. Br J Dermatol 1989; 120: 37-41.


3. Overfield TM, Briody JA. An epidemiologic study of molluscum contagiosum in
Achorage, Alaska. J Pediatr 1966; 69: 640-42.

4. Gottlieb SL, Myskowwski PL. Moluscum contagiosum. Intern J Dermatol 1994;
33: 453-61.

5. Telner P, Solomon LM. Eruptive molluscum contagiosum in atopic dermatitis. Can
Med Assoc J 1966; 95: 978-79.

6. Pauly CR, Artis WM, Jones HE. Atopic dermatitis, impaired cellular immunity
and molluscum contagiosum. Arch Dermatol 1978; 114: 391-93.

7. Dohil, P. Lin, J. Lee, A. Lucky, A. Paller, L The epidemiology of molluscum
contagiosum in children. J Am Acad Dermatol. 2006;54:47-54.

8 Rosenberg EW, Yusk JW. Molluscum contagiosum. Eruption following treatment
with prednisone and methotrexate.Arch Dermatol 1970; 101: 439-41.


9. Koopman JJ, Van Merrienboer FCJ, Vreden SGS, Dolmans WMV. Molluscum
contagiosum: a marker for advanced HIV infection. Br J Dermatol 1992; 126: 528-
29.
10. Schwartz JJ, Myskowski PL. HIV-related molluscum contagiosum presenting as
a cutaneous hom. Int J Dermatol1992; 31: 142-44.


17.- Se trata de paciente femenino de 34 aos con referencia por diagnstico de
lupus eritematoso generalizado, Cul es el sntoma dentro de los criterios de
clasificacin de de ste padecimiento?

a) Alopecia
b) Fotosensibilidad
c) Artralgias
d) Prdida de peso

Criterios de Clasificacin para el Diagnstico de Lupus Eritematoso Sistmico
(LES)
Erupcin malar: Eritema fijo, plano o alto, sobre las eminencias malares, que no
suele afectar los surcos nasogenianos.
Erupcin discoide: Placas eritematosas altas, con descamacin queratsica
adherente y tapones foliculares; puede haber cicatrices atrficas en las lesiones
ms antiguas.

Fotosensibilidad: Erupcin cutnea a causa de una reaccin inslita a la luz
solar, referida por el paciente u observada por el mdico.
lceras bucales: Ulceracin nasofarngea, por lo comn indolora, observada por un
mdico.
Artritis: Artritis no erosiva que afecta dos o ms articulaciones perifricas,
caracterizada por dolor a la palpacin, tumefaccin o derrame. Serositis: Pleuritis
o pericarditis documentada por electrocardiograma o frote o evidencia de derrame
pericrdico.
Enfermedad renal: Proteinuria persistente mayor a 0,5g/da o 3+ o cilindros
celulares.
Transtorno neurolgico: Convulsiones o psicosis en ausencia de otra causa conocida.
Transtorno hematolgico: Anemia hemoltica o leucopenia (< 4.000/mm3) o
linfopenia: (<1.500/mm3) o trombocitopenia (< 100.000/mm3) en ausencia de
frmacos que produzcan esta alteracin.
Trastorno inmunolgico: Anti-DNA, anti-Sm, y/o Anticuerpos antifosofolipdicos
(AFL).
Anticuerpo antinuclear: Un ttulo anormal de ANA por inmunofluorescencia o
anlisis equivalente en cualquier momento y en ausencia de medicamentos
relacionados con el sndrome de lupus de origen farmacolgico.
Cualquier combinacin de 4 o ms de los 11 criterios, bien documentado durante
cualquier intervalo de la historia del paciente, hace el diagnsticos de LES
(especificidad y sensibilidad son del 95% y 75%, respectivamente).
Petri M. Review of classification criteria for systemic lupus erythematosus.
Rheum
Dis Clin North Am. 2005 May;31(2):245-54.



18.- Al encontrarse realizando exploracin fsica por dolor abdominal, usted
identifica positividad a la maniobra de Murphy, esto se traduce en:

a) Dolor a la palpacin profunda en el punto cstico
b) Dolor que interrumpe la inspiracin a la palpacin profunda en el punto
cstico
c) Dolor a la palpacin en el punto cstico durante la inspiracin
d) Dolor a la palpacin profunda en el punto cstico durante la espiracin



Maniobra de Murphy, el enfermo sentado y el examinador colocado atrs
engancha el punto cistico mientras el enfermo trata de realizar una inspiracin
profunda, Es positiva en la cocolecistitis aguda.

Apuntes de Semiologa

Semiologa mdica y tcnica exploratoria Escrito por Antonio Surs Batll,Juan
Surs Batll


19.- Masculino de 54 aos que ingresa al servicio de urgencia politraumatizado abre
los ojos dbilmente a la orden verbal, tiene lenguaje coherente y apropiado, y
mueve las cuatro extremidades Que puntuacin tiene de acuerdo a la escala de
Glasgow?

a) 15
b) 9
c) 13
d) 10


BIBLIOGRAFA
1. E. Bermejo Pareja, J. Daz Guzmn, J. Porta Etessam. Cin escalas de inters
en
Neurologa. Prous Science,2001.
2. Belinda J Gabbe, Peter A Cameron, Caroline F Finche. The status of the Gasgow
Coma Scale. Emergency Medicine 2003; 15: 353-360.

3. Lynne Moore, Andr Lavoie, Stphanie Camden. Stadistical Validation of the
Glasgow
Coma Score. J Trauma 2006;60:1238-1244.
4. Kameshwar Prasad. The Glasgow Coma Scale: A Critical Appraisal of Its
Clinimetric
Properties. J. Clin. Epidemiol. Vol. 49, N 7, pp. 755-763, 1996.



20.- Ante un probable infarto agudo al miocardio, Cul de los siguientes
marcadores tiene mayor sensibilidad para realizar ste diagnstico?

a) CPK
b) Transaminasa
c) Troponina
d) Deshidrogenada lctica

Es muy conocida la falta de especificidad de la CPK. La elevacin de la actividad de
esta enzima se produce tanto en el infarto de miocardio (IAM) como en afecciones
caracterizadas por un grado variable de necrosis muscular. Por ello en los ltimos
aos se han realizado, y se siguen realizando, considerables esfuerzos para
encontrar nuevos marcadores analticos que sean capaces de diferenciar estos
cuadros. Entre ellos, la determinacin de la isoenzima miocrdica de la CPK de
forma cuantificada (CPK-MB-Masa) resulta tambin poco especfica ya que se eleva
tambin en caso de necrosis de msculo estriado; est descrita su elevacin en
traumatismos, rabdomiolisis, convulsiones
1
, miopatas agudas y crnicas
2
,
insuficiencia renal en dilisis
3
e incluso en el ejercicio intenso
4,5
; y lo mismo
podemos afirmar de la mioglobina
6
.
Ms recientemente se han determinado las troponinas T e I en sus isoformas
especficas de msculo cardiaco, que tienen una secuencia diferente de
aminocidos a las de msculo estriado no cardiaco, lo que permite el desarrollo de
inmunoensayos especficos. El complejo de las troponinas T, C, e I est
estrechamente unido al filamento de tropomiosina. La T se encarga de la unin a
tropomiosina; la C es iniciadora de la contraccin tras unirse al calcio, y la I se
llama as por ser inhibidora de la contraccin en reposo
7
. Esta especificidad ha sido
demostrada en varios trabajos que muestran valores normales de troponina-I en
procesos que cursan con necrosis de msculo estriado
2,8
.
Adems de ser muy especficas, las troponinas son altamente sensibles en el
infarto de miocardio. Mair y col encuentran una sensibilidad del 100% para el
diagnstico si se hace la determinacin de troponina-I pasadas 6 horas del
comienzo del dolor
9
. La cintica de estos marcadores en pacientes con IAM puede
resumirse de esta forma:

1. La CPK total comienza a elevarse a las 4-8 horas del comienzo de los sntomas, y
permanece elevada hasta que se normaliza a las 48-72 horas
10
.
2. La isoenzima MB de la CPK (CPK-MB) aparece en el suero tres horas despus del
comienzo del IAM, con un pico mximo a las 18-20 horas, alcanzando valores 16
veces superiores al normal. A partir de este punto desciende lentamente y persiste
elevada al menos 2 das
11
.
3. La troponina-I se eleva a partir de las 2-3 horas del comienzo de los sntomas,
con un valor mximo a las 16 horas. Desciende bruscamente hasta las 48 horas, y a
partir de entonces se produce un lento descenso; puede detectarse todava el 7-
8 da
11
.
4. La mioglobina es la primera que se eleva. Da las cifras ms altas de sensibilidad
en el plazo de dos horas del comienzo del dolor10 con respecto a la troponina-I y la
CPK-MB. Alcanza su pico a las 8 horas y desciende bruscamente a valores normales
a las 18 horas
11

Utilidad de la troponina-I, CPK-MB y mioglobina en el diagnstico del infarto de
miocardio y de los procesos de necrosis muscular de origen no cardiaco
Use of troponin-I, CPK-MB and myoglobin in the diagnosis of myocardial infarct
and processes of muscular necrosis of non-cardiac origin J.I. Ibez
1
, R. Sobrado
1
,
M. Rivero
2
, J.M. Olite
3
, I. Idoate
3
, I. Berrozpe
1
, E. Arina
1
, L. Metola
1
, J. Sesma
1

1. Unidad de Urgencias.

2. Servicio de Medicina Interna.

3. Servicio de Bioqumica Clnica. Hospital Virgen del Camino Pamplona.



21.- Femenino de 27 aos acude por amenorrea de 7 semanas asintomtica, con
antecedentes de enfermedad plvica inflamatoria tratada anteriormente, sin
evidencia mediante ecografa de tero ocupado. Al determinar la B-HCG, se
obtiene cifra de 2,500UI/l. Ante estos datos, debemos sospechar de:

a) Gestacin de evolucin normal correspondiente a amenorrea.
b) Gestacin ectpica asintomtica.
c) Aborto precoz completo con expulsin total de restos ovulares
intrauterinos.
d) Embarazo molar de inicio, sin signos ecogrficos intrauterinos.




FACTORES DE RIESGO
Anomalas tubarias

Enfermedad plvica inflamatoria 30-50%
Adherencias (endometriosis)
Antecedente de salpingoclasia
Alteraciones anatmicas

EMBARAZO ECTPICO
Descenso
Cuantificacin seriada
de HGC
USG normal
<6500mUI
Repetir USG
Laparoscopia Dx
Dilatacin y legrado
Estabilizacin o
incremento
Aumento
02/07/2013 64


Diagnstico:

HGC > 1500 U/L
Sin evidencia de saco
Lquido libre en fondo de
saco
Sensibilidad 63%
Especificidad 100%

Ankum WM., Mol Bw.,Van der Veen F.,Bossuyt PM.
Risk factors for ectopic pregnancy:a meta-analysis. Fertil Steril 1996:65:1093-9.


22.- Se trata de mujer de 23 aos de edad refiere irregularidades menstruales
tipo hipo-opso-oligomenorrea desde hace 3 aos. Niega tener vida sexual activa y
no recuerda su fecha de ltima menstruacin. No hay antecedente de galactorrea
ni de uso de hormonales exgenos. Mide 164cm y pesa 60kg. Sin datos de
hirsutismo, las mamas, tero y anexos son normales. El ultrasonido plvico es
normal, as como el perfil hormonal. El diagnstico clnico ms probable es:


a) Sangrado uterino disfuncional
b) Hiperplasia del endometrio

c) Alteracin menstrual fisiolgica
d) Endometriosis





El ciclo menstrual normal ha cambiado en el curso de los siglos, dependiendo de las
modificaciones en el patrn reproductivo y el estado nutricional de la poblacin.
El debut del sangrado menstrual, definido como menarquia, marca una etapa
importante en la madurez biolgica durante la pubertad.
En el Programa Nacional de Atencin Integral a la Salud de los Adolescentes se
mencionan los trastornos menstruales como una de las primeras causas de consulta
o de urgencia en servicios clnicos de primero y segundo nivel.
Diversas alteraciones menstruales, ya sea las que inducen aumento en la cantidad o
frecuencia o las que se presentan como episodios infrecuentes e irregulares se
observan en la adolescencia y pueden impactar la calidad de vida de las pacientes,
con repercusiones significativas en el mbito reproductivo y metablico.
Por esas razones podemos afirmar que los trastornos menstruales constituyen un
problema de salud pblica y requieren un enfoque ms integral en aspectos
preventivos y curativos en la etapa de la adolescencia.
Aproximadamente la mitad de todas las adolescentes tienen perodos irregulares
durante el primer ao despus de la menarquia. Estos periodos irregulares pueden
persistir hasta cinco aos despus de la menarquia en 20 % de estas adolescentes.
La principal complicacin es la anemia, que puede ser severa y raramente tiene
consecuencias fatales.
PATRN MENSTRUAL NORMAL

La Federacin Internacional de Ginecologa y Obstetricia considera el patrn
menstrual normal entre tres y cuatro das, aunque flucta entre dos y siete das. El
intervalo entre menstruaciones es de veintiocho das, considerndose como lmites
de veintiuno a treinta y cinco das.

El volumen de sangre menstrual es de sesenta a ochenta mililitros; no obstante, se
consideran normales los rangos de sangrado entre cincuenta y ciento cincuenta
mililitros. El aspecto de la sangre es rojo oscuro incoagulable.
En la prctica mdica se utiliza un grupo de trminos para hacer referencia a las
diversas alteraciones del ciclo menstrual, que requieren precisin por la frecuencia
en que son diagnosticados.
Segn Schiavon (2000), las alteraciones menstruales ms frecuentes son:
oligoamenorrea: episodios de sangrado infrecuentes, irregulares, con intervalo de
ms de cuarenta das;

polimenorrea: episodios frecuentes pero regulares de sangrado uterino, que
ocurren a intervalos menores de veintin das;
menorragia: sangrado excesivo, tanto en cantidad como en duracin, que ocurre
con regularidad y es sinnimo de hipermenorrea;
metrorragia: sangrado generalmente no excesivo, que ocurre a intervalos
irregulares;
menometrorragia: sangramiento generalmente excesivo y prolongado, que ocurre
a intervalos frecuentes e irregulares;
hipomenorrea: sangrado uterino regular, pero disminuido en cantidad;
sangrado intermenstrual: sangrado uterino generalmente no excesivo, que ocurre
entre perodos menstruales regulares.
Las irregularidades menstruales son causa frecuente de consulta en las
adolescentes, siendo 95 % de las veces de naturaleza disfuncional, por inmadurez
del eje hipotlamohipofisoovrico (HHO).
Uno de los primeros problemas que hay que plantear ante estas irregularidades
menstruales, es la hemorragia uterina disfuncional (HUD). Su definicin guarda
relacin con las caractersticas en cantidad y frecuencia que difieren del sangrado
menstrual normal.
Con mayor frecuencia se encuentra en forma de sangrados excesivos y
prolongados, asociados a ciclos anaovulatorios, en ausencia de una patologa o
enfermedad existente, aunque raramente la HUD puede presentarse con ciclos
ovulatorios.
De forma prctica, consideramos una hemorragia uterina (HU) como anormal
cuando el sangrado es excesivo, con cualquier desvo o alteracin de su duracin,
cantidad o intervalo.

El diagnstico de HUD supone una alteracin de origen endocrino (eje HHO); por lo
tanto, su diagnstico impone haber descartado cualquier patologa orgnica y
sistmica que produzca hemorragia genital. Es un diagnstico por exclusin.
ETIOPATOGENIA DE LA HUD EN LA ADOLESCENCIA
Los ciclos anovulatorios son ms frecuentes en las adolescentes por la inmadurez
del eje HHO en el primer ao tras la menarquia.
En esos casos de HUD, como ya se mencion, se producen ciclos anovulatorios que
se traducen en una proliferacin desorganizada del endometrio por falta de efecto
progestagnico. Una vez que el endometrio alcanza un grosor crtico, comienza a
descamarse en forma irregular, traducindose en un sangrado permanente de
cuanta variable.
Las manifestaciones clnicas de la HUD son:
fases de amenorrea de dos a cuatro meses, seguidas de salida de sangre
abundante durante tres o cuatro semanas; en oportunidades existe irregularidad
completa en el sangrado;
sangrado de ms de seis compresas (bien empapadas) al da;
presencia de cogulos;
suele ser indolora;
menstruaciones de ms de siete das de duracin;
ciclos de menos de veintin das.
La gravedad de esta hemorragia se clasifica, de acuerdo con el grado de anemia
que produzca, en metrorragia leve, moderada o grave:
Leve:
metrorragia leve y prolongada,
ciclo menstrual acortado,
hemoglobina y hematocrito normales.
Moderada:
metrorragia copiosa prolongada,
ciclo menstrual acortado,
anemia leve (cifras de hemoglobina inferior a diez gramos por litro).
Grave:
metrorragia copiosa prolongada,
ciclo acortado e irregular,
anemia grave (cifras de hemoglobina de ocho gramos por litro o menos).
Ante un sangramiento uterino en estas edades se debe realizar el diagnstico
diferencial con:

a) gestacin y problemas relacionados con sta, tales como abortos y gravidez

ectpica;
b) coagulopata: 20 % de las adolescentes con hemorragia uterina tienen un
defecto de la coagulacin. La manifestacin ms precoz de alteraciones de la
coagulacin sangunea puede ser evidenciada por un sangramiento genital anormal,
lo cual puede estar relacionado con deficiencias de plaquetas, leucemias, prpuras,
enfermedad de Von Willebrand, deficiencia de protrombina u otros factores de la
coagulacin;
c) malformaciones del aparato genital, traumatismosgenitales, presencia de
cuerpos extraos;
d) dispositivos intrauterinos;
e) tumores uterinos, sarcoma botroides o tumores anexiales;
f) hipo o hipertiroidismo;
g) insuficiencia renal o heptica.
Como el diagnstico de HUD es de exclusin, hay que hacer una historia clnica
minuciosa, exmenes complementarios y sin falta descartar las otras causas de
sangramiento transvaginal.
Se debe precisar con detalle el nivel de desarrollo puberal, la actividad sexual y la
presencia de situaciones concomitantes como: a) contacto sexual sin proteccin
contraceptiva;
b) uso irregular de anticonceptivos orales o antecedentes de insercin de
dispositivos intrauterinos;
c) ejercicios fsicos extenuantes;
d) historia previa de sangrado excesivo, asociado a extracciones dentarias,
pequeas heridas, epistaxis y otras;
e) dolencias renales u hepticas preexistentes.
Ante cualquier demanda de atencin por adolescentes con sangramiento genital con
las caractersticas descritas, se requiere de un examen fsico general que incluya
exploracin general completa, toma de tensin arterial y pulso, bsqueda de
exoftalmia, fascie Cushing, visceromegalias o presencia de masas abdominales
palpables, edemas parpebrales y de miembros superiores, as como puntos
hemorrgicos en epidermis y otros signos de coagulopata.
El examen ginecolgico debe realizarse en todas las adolescentes, con excepcin
de las que no han tenido actividad sexual y presentan sangramiento leve. Adems
del examen de sus genitales, hay que efectuar una valoracin citolgica y
microbiolgica en particular en quienes presenten manifestaciones clnicas.
En la inspeccin de los genitales durante el examen de la paciente, es importante
evaluar que el sangramiento se origine en lesiones ubicadas en los genitales
externos, uretra u hemorroides, as como indagar acerca de la posibilidad de abuso
sexual. Si despus del tratamiento de la HUD leve contina el sangrado, se
recomienda la realizacin de ecografa abdominal para precisar el diagnstico.

En pacientes que ya han tenido relaciones sexuales se debe buscar si el tero tiene
caractersticas gravdicas, la posibilidad de un aborto en curso, as como la
presencia de una masa anexial que permita corroborar la existencia de embarazo o
alguna neoplasia benigna o maligna. El examen con espculo podr demostrar un
cuello hipermico, sangrante o gravdico; de encontrarse estos hallazgos, se
descartara la etiologa disfuncional del sangrado.
Exmenes complementarios bsicos que no pueden faltar:
hemograma completo,
coagulograma completo,
orina,
ultrasonido ginecolgico abdominal, transvaginal o transrectal segn proceda,
ecografa abdominal. Si fuese necesario por los signos identificados en el examen
de la paciente, se deben realizar:
dosificacin de FSH, LH, T3, T4, TSH y prolactina si hay sospechas clnicas de
otras enfermedades endocrinas concomitantes,
laparoscopia en casos seleccionados por patologa de base,
otros, segn hallazgos de la historia clnica y la exploracin.

23.- Femenino 40 aos, G-3 P-2 A-1, se detecta anemia ferropnica de 9.5 g/dl,
refiere ciclos menstruales de 31,32 x 8,9 das de duracin, acompaados de
cogulos, los cuales aparecieron despus del nacimiento de su segundo hijo hace 12
aos. e.f.: con ligera palidez de tegumentos, S/V dentro de los parmetros
normales, genitales con evidencia de sangrado activo, al tacto vaginal bimanual se
detecta tero de consistencia firme voluminoso, irregular, aproximadamente de 12
cm. anexos libres.
El diagnstico ms probable es:


a) Adenomiosis uterina.
b) Cncer cervicouterino.
c) Hiperpalsia adenomatosa de endometrio.
d) Miomatosis uterina.



Definicin
Los leiomiomas uterinos (mioma, miofibroma, fibromioma, leiomiofibroma,
fibroliomioma, fibroma y fibroide) son tumores benignos conformados por msculo
liso del miometrio con cantidades variables de tejido conectivo fibroso.


Factores de riesgo y epidemiologia:

Son los tumores ms comunes del tero y la pelvis femenina. La incidencia citada es
del 50% hallada en estudios post mortem. Sabemos que es ms frecuente en
mujeres de raza negra (3.75 a 1), con predominio entre los 25 a 45 aos, sin
embargo, estudios recientes con anticonceptivos con dosis bajas de estrgeno y
progesterona, aparentemente no favorecen el desarrollo de los mismos. los
leiomiomas pueden ser nicos, aunque la mayora son mltiples. Se desarrollan
frecuentemente en el cuerpo del tero y con menor frecuencia en el cervix. El
cambio ms comn es la degeneracin hialina.


Clasificacin:

Por tamao, grandes elementos, pequeos elementos. Por localizacin, intra
murales, submucosos, subserosos, parasitario.


Cuadro clinico:
La mayora de los miomas son asintomticos; sensacin de plenitud o presin en el
bajo abdomen, compresin de vsceras pelvianas, hemorragia o flujo no
relacionados con la menstruacin, dificultad o dolor al orinar, aumento del
permetro, abdomen agudo en torsin, aborto espontneo.


Laboratorio: biometra hemtica, qumica sangunea, examen general de orina y
tiempos de coagulacin. un papanicolaou es vital antes del tratamiento quirrgico,
en algunos casos puede influir en la extensin de la ciruga, o preparacin vaginal
adecuada en caso de infeccin. la prueba inmunolgica de embarazo, es importante,
por cuestiones legales en las que se pudiera ver inmiscuido el mdico, en caso de no
detectar dicho estado.
Gabinete: el ultrasonido; continua siendo el estudio de gabinete ms confiable,
para el diagnstico de miomatosis, con sensibilidad y especificidad cercanos al 97%
cuando se realiza a travs del abdomen y hasta de 99% cuando se realiza en forma
intravaginal.



24.- Femenino de 55 aos, se queja de presin plvica y una masa en la entrada
vaginal. Anetecedentes: G.3 P.3, el ltimo con peso al nacer de 4,500 grs. FUR hace
tres aos. Sin terapia de reemplazo. Historia de tabaquismo positivo a razn de 40
cajetillas ao. Actualmente refiere dificultad para evacuar, tos crnica, presenta
una orina de 60 cc. En la exploracin plvica, cual ser el hallazgo ms probable?:

a) Cistocele
b) Enterocele

c) Rectocele
d) Uretrocele

El Rectocele es una hernia de la pared anterior del Recto hacia la porcin posterior
de la vagina. La incidencia real de esta deficiencia anatmica es desconocida y en
muchas ocasiones es un resultado del paso del tiempo. Es un hallazgo muy
frecuente del examen perineal, siendo en mltiples ocasiones asintomtico.
El rectocele puede ser un hallazgo importante del sndrome de Obstruccin
Defecatoria
(SOD). No debe tomarse como una deficiencia anatmica nica, sino como parte
importante de un problema anatomo-fisiolgico complejo.

Un principio importante es la etiologa y anatomo-patologa del rectocele. Existen
varias teoras sin consenso principal. La existencia, deficiencia o alteraciones del
septo rectovaginal son controversias importantes. No existe una fascia visceral
que separe el recto de la vagina o que forme un septo especfico. Existe frontera
entre donde termina la pared anterior del recto y donde comienza la pared de la
vagina, pero mltiples estudios no han encontrado un septo rectovaginal especfico.
El septo puede estar formado de una pelcula casi transparente hasta una pared de
consistencia fibromuscular fuerte.
El rectocele es un hallazgo comn. Se presenta en el 80% de las pacientes
femeninas y
13% de los masculinos en una defeco grafa (> a 1cm.) Entre mas grande es el
rectocele, mayor son los sntomas asociados, Dificultad en la evacuacin,
constipacin crnica, dolor rectal y perineal, sensacin de masa y en ocasiones
sangrado. La necesidad de presin manual para ayudar a la evacuacin o para vaciar
el rectocele es comn en ms del 50% de los pacientes.
El diagnostico se hace con un simple tacto rectal e inspeccin vaginal, pero debe
recordarse que rara vez es el rectocele un hallazgo aislado. La presencia cistocele,
peritoneocele, enterocele u otros prolapsos perineales y problemas funcionales
deben ser descartados antes de proponer la reparacin quirrgica del rectocele.
Defeco grafa,
Pruebas de funcin fisiolgica del piso plvico, pruebas de funcin urinaria,
evaluacin del esfnter anorectal por ultrasonido y hasta la resonancia magntica
han sido propuestos antes de la ciruga.


Bibliografa:
Rectocele: Pathogenesis and surgical managment. Zbar AP, Linemann A, Fritsch H,
Beer-Gabel M, Pescatori M. Int J Colorectal Dis. (2003) 18:369-384.
Evaluation and Treatment of Women with rectocele. Cundiff GW, Fenner D,
Obstetrics
and Ginecology 104(6): 1403-1416

Stapled transanal rectal resection to treat obstructed defecation caused by
rectal
intussusseption and rectocele. Renzi A, Izzo D, Di Sanrno (26) 21:661-667
Rectocele repair using biomaterial augmentation. Altman D, Melgren A,
Zetterstrom J.
Obstet Gynecol (2005) 60(11)753-760.


25.- Se trata de femenino de 30 aos de edad es llevada a quirfano por una
laparoscopa diagnstica debido a dolor plvico y en cuadrante superior izquierdo
crnicos (durante los ltimos 2 aos). No tiene alteraciones funcionales vesicales o
intestinales. Tiene antecedente de 2 episodios de gonorrea previos. Bebe una
cerveza al da. Labs: HCG urinaria negativa; Hto 39%; Leuc. T 8 000; Plt 200 000;
AST 12; ALT 14. Intraoperatoriamente se observan adhesiones densas que
involucran los oviductos, ovarios y tero. Tambin se observan adhesiones
perihepticas que se extienden desde la superficie heptica hacia el diafragma. El
diagnstico ms probable es:

a) Hepatitis
b) Sx de Fitz-Hugh-Curtis
c) Carcinoma hepatocelular
d) Sx Wolff-Parkinson-White


El sndrome de Fitz-Hugh-Curtis se define como la presencia de una perihepatitis
asociada a salpingitis. Los agentes etiolgicos reconocidos hasta la fecha son
Chlamydia trachomatis y Neisseria gonorrhoeae. El cuadro clnico de este sndrome
es inespecfico y puede ser confundido con procesos inflamatorios o infecciosos
del tubo digestivo, aparato urinario y respiratorio, en los cuales la manifestacin
sintomtica fundamental es el dolor en hipocondrio derecho. El diagnstico debe de
sospecharse en aquella mujer joven con vida sexual activa que tenga antecedentes
de promiscuidad en ella o en su pareja, que se queje de dolor subcostal derecho. Es
ms probable el diagnstico si se cuenta con el antecedente de enfermedad plvica
inflamatoria y ms an, si se tiene evidencia de que sta sea causada por Neisseria
gonorrehoeae y/o Chlamydia trachomatis. El diagnstico definitivo se realiza con la
visualizacin directa de la adherencia periheptica por laparoscopa o laparotoma.
Se recomienda la primera. El tratamiento mdico es a base de cefalosporinas y
dicloxacilina y en algunos casos se requiere de la extirpacin quirrgica del proceso
adherencial para mitigar el dolor.

El Sndrome de Fitz-Hugh-Curtis. Causa frecuente de error de diagnstico en
hepatologa y gastroenterologa / The Fitz-Hugh-Curtis Syndrome. a frequent
misdiagnosis in hepatology and gastroenterology
Rev. gastroenterol. Mx;60(4):223-8, oct.-dic. 1995.



26.- Femenino de 32 aos que inicia tratamiento con sulfato de magnesio por
presentar eclampsia, se presentan signos de sobre dosificacin qu antdoto se
debe emplear?:


a) Nitroprusiato.
b) Simpaticomimticos.
c) Gluconato clcico
d) Carbonato sdico.


NIVEL DE PRIMER CONTACTO (ATENCION PRIMARIA)
Se debe instruir a todas las embarazadas que deben acudir inmediatamente a un
centro de salud en cualquiera de los siguientes casos:
Edema que se desarrolla rpidamente (en pocos das)
Cefalea severa y persistente
Dolor en la regin abdominal superior
Visin borrosa
Se debe realizar la medicin de la presin arterial y un anlisis de orina para la
deteccin de proteinuria a las mujeres que acudan a centros de salud presentando
estos sntomas.
Convulsiones
Si se asiste a una mujer con eclampsia en un centro de atencin primaria,
1. Deben mantenerse las vas respiratorias permeables;
2. Se debe colocar a la mujer de costado (posicin decbito lateral izquierda) para
evitar la aspiracin del vmito u otras secreciones;
3. Si es posible, se debe establecer una va intravenosa;
4. Se debe administrar sulfato de magnesio.
Monitoreo de la administracin de sulfato de magnesio: Durante el tratamiento con
sulfato de magnesio, se recomienda realizar un control cada 4 horas, como mnimo,
para detectar la presencia de: Reflejo rotuliano, frecuencia respiratoria superior a
16 por minuto, volumen de orina >100 ml en las 4 horas previas.

- Sobredosis de sulfato de magnesio: Todo centro de salud que utilice sulfato de
magnesio debe disponer de ampollas de gluconato de calcio (1 g) como antdoto para
la sobredosis de dicho frmaco.
Se sugiere medir la presin arterial y administrar antihipertensivos segn
corresponda.
Convulsiones recurrentes: en caso de convulsiones recurrentes, se
administran otros 2 a 4 g de sulfato de magnesio por va IV en el lapso de 5
minutos, tanto para el rgimen IM como el IV; la dosis se determina en
funcin del peso de la paciente.
El sulfato de magnesio es un frmaco usado en el control de las convulsiones
eclmpticas, para suprimir o controlar las contracciones uterinas sean estas
espontneas o inducidas, y como broncodilatador luego del uso de beta agonistas y
agentes anticolinergicos. Tambin tiene indicacin como terapia de reemplazo en
la deficiencia de magnesio, como laxante para reducir la absorcin de txicos del
tracto gastrointestinal. El sulfato de magnesio esta ganando popularidad como
tratamiento de inicio en el manejo de algunas arritmias, particularmente en
Torsades de Pointes, y en arritmias secundarias a sobredosis de antidepresivos
tricclicos o toxicidad digitlica. Esta tambin considerado clase Ila (probable
beneficio) para la fibrilacin ventricular refractaria y la taquicardia ventricular,
luego de la administracin de dosis de lidocaina y bretilio.


FARMACODINAMIA
El sulfato de magnesio tiene la capacidad de alterar la excitabilidad de la fibra
miometrial, afecta el acoplamiento excitacin contraccin y el proceso mismo de
contraccin, inhibe la entrada de calcio al sarcoplasma y reduce la frecuencia de
los potenciales de accin. Inhibe tambin la liberacin de acetilcolina. Por ser
estas acciones comunes en las fibras musculares se pueden ver afectadas tambin
la musculatura voluntaria e incluso las fibras miocrdicas.
(1)

Bibliografa.
Graves C. Frmacos que contraen o relajan el tero. En: Hardman J, Limbird L,
Molinoff P, Ruddon R, Goodman A, eds. Goodman & Gilman. Las Bases
Farmacolgicas de la Teraputica. 9 ed. Mxico DF: McGraw-Hill Interamericana;
1996. pp. 1012-3.



27.- Femenino que cursa con 9 semanas de gestacin, la cual tiene contacto con un
menor que, 6 das ms tarde, desarrolla un cuadro de exantema y sndrome general
infeccioso sugerente de infeccin por virus de rubola. En el primer control
serolgico gestacional se detect la negatividad de la IgG especfica. Cul de las
siguientes afirmaciones es la correcta?:


a) No existe posibilidad de contagio dado que el nio ya no se hallaba en fase
de eliminacin viral.

b) En este perodo de la gestacin, el riesgo de la infeccin y de afectacin
embrionaria es mnimo.

c) La aparicin de IgM materna positiva constituye indicacin para la
determinacin de la IgM fetal.

d) La existencia de IgM fetal negativa excluye en este caso la posibilidad de
transmisin transplacentaria.



INFECCIONES EN EMBARAZADA

VACUNACIONES:
CONTRAINDICADAS:
- Parotiditis
- Rubola
- Sarampin
- Fiebre amarilla
- NO se RECOMIENDA: gripe, poliomielitis, hepatitis B.
S SE PUEDEN DAR (cuando estn indicadas):
- Fiebre tifoidea
- Rabia
- Ttanos
- Tos ferina (ver pregunta mir, que pone que no)

TOXOPLASMOSIS:
50% de transmisin. Si grave (en 1er T, pero es menos frecuente): aborto,
parto pretrmino, muerte fetal intratero.
Dx: lo de siempre. Calcificaciones cerebrales.
TTO:
- En casos de seroconversin ESPIRAMICINA (depresin medular,
ac. folnico) ht final del embarazo.

- Si infeccin fetal: PIRIMETAMINA Y SULFADIAZINA en ciclos
de 3 semanas alternando con la espiramicina ht final del
embarazo.
RUBEOLA:
Contagio >80% si la madre se contagia en 8 primeras semanas. SORDERA
CONGNITA.
Acs <1/16 susceptibilidad para infeccin. La aparicin de IgM materna
positiva es indicacin de determinacin de IgM fetal.
La madre no debe quedarse embarazada en los 3 meses siguientes a la
vacunacin.
Profilaxis de EXPOSICIN: gammaglobulina, solo eficaz en primeros 7-8d.

CITOMEGALOVIRUS:
Insospechada. Inclusiones citomeglicas en OJO DE BHO en clas de
tejidos afectados.

SFILIS:
Prueba sexolgica a todas las gestantes. Si se infecta, en los 3 meses
neonatales: lesiones CUTNEOMUCOSAS, OSTEOCONDRITIS Y
HEPATOESPLENOMEGALIA.
DX: screening VDRL o RPR (no treponmicas, pero el embarazo es la
primera causa de falso positivo)
certeza FTA-Abs o MHA-TP (treponmicas)

VARICELA:
Si en 1er T 2%; gravemente teratognica. En perodos ms avanzados o
periparto enfermedad sistmica generalizada o SNC.


HEPATITIS B:
Se infecta el hijo si:
- Madre PORTADORA CRNICA
- INFECCIN ACTIVA durante la gestacin
- Madre con HEPATITIS CRNICA ACTIVA
HBsAg positivo (slo) riesgo bajo de transmisin placentaria
HBsAg + HBeAg 90% de transmisin
*El riesgo de cronificacin ser muy alto si se adquiere en el perodo perinatal.
*Especial vigilancia del crecimiento fetal.
*Se realizar profilaxis ACTIVA y PASIVA
*La gestacin NO aumenta el riesgo de curso clnico grave.


RUBOLA Y EMBARAZO

Existen tres situaciones claramente diferenciadas que exigen planteamientos
diagnsticos distintos:
Determinacin de la inmunidad frente a rubola en la gestante, sin sospecha
clnica ni epidemiolgica de padecer la enfermedad: El objetivo de este estudio
es conocer si la gestante est protegida, de una posible infeccin por el virus de la
rubola, durante el embarazo. Se recomienda la determinacin cualitativa de
anticuerpos totales o de IgG especfica, en la primera consulta de control del
embarazo. Se desaconseja expresamente la evaluacin cuantitativa de los
resultados, ya que no proporciona ninguna informacin til. La presencia de
anticuerpos refleja contacto previo con el virus, y por tanto inmunidad, haciendo
innecesaria la realizacin de nuevos controles en embarazos sucesivos.
A pesar de que se describe que la rubola puede cursar de manera asintomtica, F.
de Ory et al estudian 185 sueros de 101 mujeres embarazadas con presencia de
IgM y slo confirman la existencia de primoinfeccin en tres de las mismas,
asocindose siempre a datos clnicos o epidemiolgicos compatibles, por lo que es
desaconsejable la realizacin sistemtica de IgM a las embarazadas.
Si la mujer embarazada es seronegativa, deber adoptar las precauciones
necesarias para evitar la exposicin al virus y debe ser vacunada frente a la
rubola en el post-parto inmediato.
Sospecha clnica de infeccin aguda durante el embarazo: Este caso puede
plantearse ante la existencia de una clnica compatible en la embarazada, o por
exposicin a un sujeto con infeccin aguda por rubola.
La presencia de IgG en ausencia de IgM indica que la mujer est protegida, por
vacunacin o por infeccin antigua y por tanto no deben realizarse ms
determinaciones.
La demostracin de seroconversin, con ausencia de anticuerpos en el primer suero
y presencia de stos en el segundo, obtenido 15-21 das despus, es la forma ms
segura de diagnosticar una primoinfeccin por este agente. Sin embargo, si el
primer suero de la enferma presenta anticuerpos, aunque se produzca un
incremento del ttulo de estos en el segundo suero, puede ser debido a una
reinfeccin.
La presencia de IgG y de IgM especfica en una paciente, nos hace sospechar la
presencia de primoinfeccin, sin embargo, debemos tener en cuenta varios
aspectos:
a.- La IgM puede tener reacciones heterlogas entre rubola y otros virus como
EBV, CMV, Parvovirus B 19 y virus del sarampin (por reacciones cruzadas o por
estimulacin policlonal de linfocitos de memoria); por tanto es necesario confirmar

su presencia, siendo la tcnica de ELISA de captura la que presenta mejor
especificidad y sensibilidad.
b.- La IgM puede aparecer durante las reinfecciones, pero a ttulos bajos y
durante poco tiempo.
c.- En un pequeo porcentaje de personas, la IgM puede mantenerse positiva en
suero hasta 6 meses. Thomas et al, detectan la presencia de IgM en el 9% de los
casos a los 3 4 meses de la infeccin aguda.
El estudio de la avidez de la IgG diferencia si la IgG es de aparicin reciente (baja
avidez se asocia a infeccin primaria aguda) o si hay ausencia de infeccin primaria
(IgG de alta avidez), puede ser una tcnica que ayude a valorar la presencia de IgM
y puede colaborar en la diferenciacin entre primoinfeccin y reinfeccin. F. de
Ory et al. estudian mltiples patgenos y comunican que esta tcnica presenta una
sensibilidad entre el 81 y el 100% y una especificidad del 100%. Tambin se est
valorando la utilidad en el diagnstico de la IgA, aunque los datos no son an
concluyentes.
Todos estos datos serolgicos deben ser interpretados junto con los datos clnicos
de la embarazada, en el caso de que los haya y junto con los datos que podamos
obtener de la posible fuente de infeccin

BOSMA TJ, CORBETT KM, OSHEA S, BANATVALA JE, BEST JM. PCR detection
of rubella virus in clinical samples. J Clin Microbiol 1995; 33:1075-1079.
BOSMA TJ, CORBETT KM, OSHEA S et al. Use of PCR for prenatal and
postnatal diagnosis of congenital rubella. J Clin Microbiol 1995; 33:2881-2887.
DE ORY F, CASAS I, DOMINGO CJ, ECHEVARRA JM. Application of
fluoroimmunoassay to the identification of low avidity specific IgG against
pathogenic human viruses and Toxoplasma gondii. Clin Diagn Virol 1995; 3:323-332.
DE ORY F, DOMINGO CJ. Los anlisis de avidez de la IgG especfica en el
diagnstico de la infeccin por el virus de la rubola. Med Clin (Barc) 1996;
107:118.
DE ORY F, ECHEVERRA JM, DOMINGO CJ. Cribado rutinario de IgM especfica
antirrubola en mujeres embarazadas: una prctica desaconsejable. Prog Obstr
Ginec. Dic 1998; 41:574-578.
ENGLUND J, GLEZEN WP, PIEDRA PA. Maternal immunization against viral
disease. Vaccine 1998; 16:1456-1463.

FREY TK, ABERNATHY ES, BOSMA TJ et al. Molecular analysis of rubella virus
epidemiology across three continents: North America, Europe and Asia, 1961-1997.
J Infect Dis 1998; 178:642-650.


28.- Mujer de 35 aos que acude a consulta con antecedentes de G4 C2 A1,
refiere que ha presentado durante el primer y segundo trimestres de su
embarazo un manchado con frecuencia intermitente, a las 34 SDG inicia con
hemorragia abundante, repentina e indolora, su principal sospecha es:


a) Coriocarcinoma
b) Ruptura uterina
c) Placenta previa
d) Desprendimiento grave de placenta normoinserta


PLACENTA PREVIA
DEFINICIN:
Es cuando la placenta se implanta sobre o muy cerca del orificio cervical interno y
una parte de la placenta precede a la parte fetal que se presenta.
INCIDENCIA:
Esta es difcil determinar ya que muchos casos pasan desapercibidos, sobre todo
cuando ocurren los abortos en embarazos tempranos.
La prevalencia vara de 1 en 100 a 1 en 850 nacidos vivos, pero solo el 20% total.
Etiologa:
Edad avanzada, multparas, paciente con cesreas previas, paciente con aborto de
repeticin, esto debido a las gestaciones previas.
TIPOS:
Insercin baja.- Es cuando el borde placentario se encuentra en el segmento
inferior a menos de 6 cm del orifico cervical interno.
Marginal.- Es cuando el borde placentario alcanza los mrgenes del orificio cervical
interno.
Parcial.- Es esta la placenta cubre parcialmente el orificio cervical interno.
Total.- La placenta cubre la totalidad del orificio cervical interno an con
dilatacin cervical avanzada.
DIAGNOSTICO:
La caracterstica es el STV de aparicin brusca en forma indolora en el segundo o
tercer trimestre. Frecuentemente hay ausencia de dolor a actividad uterina que
son parmetros para hacer el diagnstico.
La mayor incidencia de sangrado aparece a las 33-34 sdg.
ESTUDIOS DE GABINETE:
El estudio mas utilizado es la ultrasonografa obsttrica.
TRATAMIENTO:

El manejo va a depender de factores como son:
Edad gestacional, magnitud del sangrado, si hay trabajo de parto, variedad de
placenta previa y complicaciones materna.
BIBLIOGRAFA:
1.- Waxler P, Gottesfeld KR. Early diagnosis of placenta previa. Obstet Gynecol
1979;54:231-32.
2.- Cabrero-Roura L. Riesgo elevado obsttrico. Ed. Masson 1996; pp; 109-118.
3.- Patrick J, Placenta Previa, Clinical Obst and Gynecology 1990;33(3): 414-421.
4.- Chapman M, Furtenes ET, Significance of ultrasound in location of placenta in
early pregnancy Br J Obst Gynecol 197;86: 846.
57



29.- Femenino de 30 aos con sospecha diagnstica de endometriosis, el sntoma
caracterstico de esta patologa es:

a) Dismenorrea
b) Dispareunia
c) Metrorragia
d) Leucorrea






ENDOMETRIOSIS:
Es relativamente frecuente en personas jvenes, 25-30 aos. Se define como la
localizacin de la mucosa endometrial en un lugar ectpico.
La localizacin normal de endometrio es el tero. La mucosa endometrial sufre una
serie de cambios durante el ciclo. Cuando la mucosa endometrial penetra en el
msculo uterino (miometrio) se denomina: Adenomiosis.
Patogenia:
No se conoce realmente por qu se produce y tampoco existe un tratamiento
definitivo para ella.
Teoras: a) Teora Metaplsica celmica (T. De Meyer): El celoma es un epitelio que
recubre la cavidad abdominal (metaplasia).

b) Teora Transplantativa (Sampson): La mucosa endometrial a travs de las
trompas, cuando se produce una menstruacin si el orifico cervical est cerrado, la
sangre refluye (por mecanismo retrgrado), y va a cavidad abdominal donde se
producen implantes de clulas endometriales (en fondos de saco de Douglas,
peritoneo...). Tambin por intervenciones quirrgicas. Esta teora es la que parece
ms veraz.
c) Teora embrionaria: Es poco importante. Dice que la endometriosis se produce
por los restos embrionarios de Wolf o Mller.
d) Teora inmunolgica: Suele influir sobretodo en las personas estriles, con
disminucin de la respuesta inmunitaria por: - Disminucin de la citotoxicidad de
las clulas NK// - Aumento de resistencia de las clulas endometriales a la
destruccin// - Aumento de la actividad de los Macrfagos.
Clnica:
Disfuncin menstrual: Aparicin de dismenorrea progresiva, que no aparece
desde el principio (es tarda), es intermenstrual y aumenta al final ms
intensa. Va aumentando con los aos (al contrario que la Dismenorrea
normal).
Exploracin: Retraccin de los ligamentos uterosacros, nodulaciones,
rugosidades y dolorosos al tacto. Dispareunia (dolor en las relaciones
sexuales). Dolor en la defecacin (porque la reaccin inflamatoria produce
adherencias y dolor en la movilizacin del tero). Dismenorrea.
Disfuncin ovrica: Se hace quistectoma, son ms conservadores en el
tratamiento quirrgico. Insuficiencia ltea. Hiperprolactinemia.
Amenorrea// No son muy frecuentes ni especficas puede que no se den las
tres.
Disfuncin reproductiva: Es causa de esterilidad (estadios III y IV de la
clasificacin americana: Reaccin inflamatoria que afecta a anejos, recto,
sigma, apndice, ureter...)// Se producen abortos de repeticin e
insuficiencia ltea.


Bibliografa
1. Stenchever A. Comprehensive Gynecology. 4th ed. St. Louis, Mo: Mosby;
2001:1065-1070.

2. Noble J. Textbook of Primary Care Medicine. 3rd ed. St. Louis, Mo: Mosby;
2001:325.
3. Chen C, Cho S, Damoskosh AI. Prospective study of exposure to
environmental tobacco smoke and dysmenorrhea. Environ Health Perspect
2000; 108(11): 1019-1022.
4. Wilson ML, Farquhar CM, Sinclair OJ. Surgical interruption of pelvic nerve
Pathways for primary and secondary dysmenorrhea. Cochrane Database
Syst Rev 2000; (2): CD001896.
5. Morrison B, Daniel S, Kotey P, et al. Rofecoxib, a specific cyclooxygenase-2
inhibitor, in primary dysmenorrhea: A Randomized controlled trial. Obstet
Gynecol 1999; 94(4): 504-508.
6. Schroeder B, Sanfilippo J. Dysmenorrhea and pelvic pain in adolescents.
Pediatrics Clinics of North America. 1999; 46 (3): 555-571.
Dawood MY. Dismenorrea. Clnicas Obsttricas y Ginecologa.


30.- Femenino de 34 aos con antecedentes patolgicos de hipertensin arterial
crnica bien controlada tratada con IECAS, actualmente cursa con 7 semanas de
gestacin, signos vitales dentro del parmetro normal y exmenes de laboratorio
sin alteraciones, se refiere asintomtica. La conducta ideal a seguir es?

a) Mantener el tratamiento y asociar alfametildopa para disminuir los riesgos
fetales de los IECAs
b) Mantener el tratamiento y asociar hidralacina para disminuir los riesgos
maternos de los IECAs.
c) Mantener el tratamiento dado el buen control tensional.
d) Suspender los IECAs dado el riesgo que presentan para el feto.

El uso de IECA y ARAII durante el segundo y tercer trimestre de embarazo
est contraindicado, debido a que estos medicamentos inducen toxicidad fetal
(descenso de la funcin renal, oligohidramnios, retraso en la osificacin del
crneo) y toxicidad neonatal (insufi ciencia renal, hipotensin, hiperpotasemia).
En cuanto a su uso durante el primer trimestre de embarazo, un estudio
publicado en el ao 20061 mostraba un incremento de la incidencia de
malformaciones congnitas, en particular malformaciones cardiacas, en nios
nacidos de madres expuestas a IECA durante el primer trimestre de embarazo en
comparacin con las mujeres que no recibieron tratamiento antihipertensivo o que
recibieron tratamiento con otros medicamentos antihipertensivos. Estudios
posteriores realizados no han confirmado a da de hoy los resultados de este
estudio. En lo referente a los ARAII, no se dispone de estudios epidemiolgicos
analticos apropiados, por lo que no se puede descartar que exista el mismo riesgo
que para los IECA.

A pesar de estas incertidumbres, el Comit de Medicamentos de Uso Humano
(CHMP) de la Agencia
Europea de Medicamentos (EMEA) ha recomendado prudencialmente evitar el uso
de IECA y ARAII durante el primer trimestre del embarazo.

Cooper WO et al. Major congenital malformations after fi rst-trimester exposure
to ACE inhibitors. N Engl J Med 2006; 354 (23): 243- 51.
(ref.: 2008/10, junio


31.- Femenino de 23 aos, G1 en trabajo de parto prematuro con embarazo de 30
semanas de gestacin. A pesar del uso de agentes tocolticos, estos no han dado
resultado. Se puede inducir la maduracin pulmonar del producto por medio de:
a) b) Sulfato de magnesio
b) Hidroxiprogesterona
c) Betametasona
d) Clorprocana

La utilizacin de betametasona como inductor de madurez pulmonar fetal (IMPF)
disminuye la morbilidad neonatal relacionada con prematurez pero su efecto
diabetgeno materno ha sido poco estudiado.
La revisin Cochrane de un ciclo nico de corticosteroides se actualiz en 2006. En
esta actualizacin se incluyeron 21 estudios con un total de 3885 mujeres y 4269
lactantes.
En la revisin se descubri que la administracin de determinados corticosteroides
a mujeres con riesgo de tener un parto prematuro reduce considerable los riesgos
de complicaciones relacionadas con la prematurez como muerte fetal y neonatal
combinada, sndrome de dificultad respiratoria, hemorragia cerebroventricular,
enterocolitis necrotizante, infecciones sistmicas y retraso en el desarrollo
durante la niez. Los beneficios estaban presentes cuando el tratamiento se
iniciaba entre las 26 y las 35 semanas de gestacin y en los nios que nacan entre
1 y 7 das despus de haber comenzado el tratamiento; tambin se observaron
beneficios en los subgrupos de mujeres con rotura prematura de membranas y
trastornos hipertensivos. La muerte fetal y neonatal combinada se redujo incluso
en neonatos que nacieron a menos de las 24 horas de haber administrado la
primera dosis.

No se demostraron beneficios cuando el tratamiento comenz antes de las 26
semanas de gestacin, tampoco se observaron beneficios en los recin nacidos
antes de las 26 semanas de gestacin ni en los que nacieron despus de 7 das o
ms de la administracin del tratamiento.
En el caso de los neonatos que nacieron despus de las 36 semanas hubo una
tendencia a aumentar la muerte fetal y neonatal combinada.
Se observ una reduccin en el peso al nacer en los neonatos que nacieron entre los
das 1 y 7, al igual que en los que nacieron ms de 7 das despus del primer
tratamiento.
Un estudio que reclut mujeres con preeclampsia severa sugiri que las mujeres
tratadas tenan un mayor riesgo de sufrir diabetes gestacional.
La evidencia epidemiolgica y en animales sugiere que pueden haber efectos
adversos a largo plazo por la exposicin prenatal a los corticosteroides, entre ellos
la alteracin de la tolerancia a la glucosa y la hipertensin. Los estudios en animales
tambin han sugerido que afecta el crecimiento del cerebro.
1. National Institute of Health (NIH). Consensus Conference; Effect of
corticos-teroide for fetal maturation on perinatal outcomes. JAMA
1994;(12):1-19.
2. White A, Marcucci G, Andrews E, Edwards K. Antenatal steroids and
neonatal outcomes in controlled clinical trials of surfactant replacement.
Am J Obstet Gynecol 1995; (173):286-90.
3. Klauss MH, Fanaroff AA, Martin RJ. Problemas respiratorios. En:
Asistencia del recin nacido de alto riesgo. 2 ed. La Habana: Editorial
Cientfico-Tcnica, 1981:194.
4. Avery M, Frank N, Gribetz I. The inflationary force produced by pulmonary
vascular distention in excised lungs. The possible relation of this force to
that needed to inflatc the lungs at birth. J Clin Invest 1959;38:456.
5. Chu J, Clements J, Cotton E. Neonatal pulmonary ischemia. Pediatrics
1965;40:733.
6. Liggins GC, Howle RN. A controlled trial of antepartum glucocorticoid
treatment for prevention of respiratory distress syndrome in premature
infants. Pediatrics 1972;50: 515-25.
7. Wright LL, Verter J, Younes N. Antenatal corticosteroids administration
and neonatal outcome in infants 501 to 1500 g. Am J Obstet Gynecol 1995;
(173):263.



32.- Se trata de mujer de 33 aos se enva de alta con diagnstico de enfermedad
inflamatoria plvica, regresa a los 15 das con temperatura de 38.5 c, mal estado
general y datos de irritacin peritoneal, El diagnstico ms probable es?



a) Absceso tubo-ovrico
b) Endometritis
c) Hidrosalpinx
d) Ooforitis aguda


La enfermedad inflamatoria plvica (EIPA) es un sndrome clnico caracterizado
por la infeccin del tracto genital superior que se produce casi siempre por va
ascendente desde el cuello uterino. El impacto que la infeccin plvica ejerce sobre
la condicin fsica de la mujer va desde la infeccin asintomtica o silente a una
mayor morbilidad que en algunos casos puede llegar hasta la muerte. Incluye una
variedad de condiciones inflamatorias que afectan el tracto genital superior. Los
Centros de Control de Enfermedades (C .D. E.) la definen como un sndrome agudo
debido al ascenso de microorganismos de la vagina o el cuello uterino al
endometrio, trompas uterinas y en ocasiones a las estructuras vecinas (ovarios,
peritoneo y cavidad pelvianas).

En el momento actual se incluyen como principales agentes etiolgicos de la E.I.P.A
la Neisseria gonorrhedae, las clamydias y los anaerobios. Otros microorganismos
como los microplasmas y los actinomices se estn observando con frecuencia.
La presencia de anaerobios as como de bacterias aerobias puede deberse a un
fenmeno de sobre infeccin secundaria. Hay autores que sealan que excepto para
el gonococo y la Clamydia trachormatis, no existen datos suficientes que permitan
afirmar que otras bacterias tengan un papel primario en la infeccin de unas
trompas sanas. Una vez alterada la integridad anatmica de la trompa, se
producira la infeccin mixta o poli microbiana.

El absceso tubo ovrico es una formacin inflamatoria que compromete el ovario
y la trompa y puede ser uni o bilateral. En este absceso las estructuras
comprometidas estn infectadas y contienen pus. Este proceso inflamatorio es
secundario a un proceso infeccioso de la pelvis, habitualmente producido por
grmenes muy patgenos, que llegan al tracto genital a travs de una relacin
sexual, es decir corresponde a una complicacin severa de una enfermedad de
transmisin sexual.

Se caracteriza por aumento de volumen del ovario y trompa, los que se
encuentran adheridos entre s producto de esta infeccin, adems el proceso
infeccioso se extiende habitualmente a otras estructuras y rganos pelvianos, los
que estn muy inflamados y adheridos formando lo que se denomina plastrn.
El tratamiento se inicia mdicamente con antibiticos de amplio espectro para
cubrir tanto grmenes aerbicos como anaerbicos, generalmente requiere de
hospitalizacin para iniciar una terapia agresiva endovenosa con los antibiticos y
para monitorizar adecuadamente a la paciente, pues la infeccin produce
compromiso del estado general pudiendo llegar hasta la sepsis generalizada.
Diagnostico
El cuadro clnico se sospecha cunado una paciente consulta por dolor abdominal
intenso, progresivo, fiebre y compromiso de su estado general, habitualmente en
el examen se encuentra un distensin abdominal y a la palpacin del abdomen
hay dolor, y signo de blumberg positivo o irritacin peritoneal. El Tacto vaginal
demuestra fondos de saco vaginales abombados y dolorosos y habitualmente el
cuello del tero lateralizado y doloroso a la movilizacin si el compromiso es
unilateral, adems de palpar una masa para uterina irregular y sensible.









BIBLIOGRAFA.
1. Botella Llusi,J.Clavero Nez,J.A:Tratado de Ginecologa.14
edicin.Ed.Diaz de Santos.pg 833-844.Madrid,1993.
2. Brunham,R,C:Infectionin woman and ectopic pregnancy. Am J Obstet
Gynecol.67:722,1999.

3. Cates,W,Wasserheit,J,N:Genital Infection Epidemiology and sequeale.Am J
Obstet Gynecol 164-1771,1998.
4. Keit,L,G; Berger,G,S:On the causation of pelvic inflammatory disease. Am J
Obstet Gynecol 149-215,2002.
5. Muller,B,R;Allen,J,et al.Pelvic Inflamatory disease after
histerosalpingography.Brit J Obstet Gynecol,91-1181,1999.
6. Toth,A,O Leary,W,M: Evidence of microbial transfer by espermatozoo.Am
J O bstet Gynecol 59-556,2003.
7. Varela,R,et col:Abceso Tuboovrico,Acta mdica Portuguesa ,p:537-542,Vol.
8,2001.
8. Sopper,D,E:Pelvic Inflamatory disease.Infections disease.Clin of North
America .831-840,vol 8;n 4.Dec 2003.



33.- Se trata de paciente femenino de 64 aos de edad con la siguiente
sintomatologa: plenitud, estreimiento, imagen qustica en el ultrasonido plvico en
ovario derecho de 15 por 15 cms. El diagnstico ms probable es:
a) Teratoma qustico.
b) Cistadenoma seroso.
c) Disgerminoma.
d) Endometrioma.

Los Tumores de Ovario son una patologa frecuente dentro del contexto de la
patologa femenina. Por esta causa consultan un grupo elevado de mujeres, tanto
las consultas de ginecologa como las de Ciruga propiamente dicha. Las edades
oscilan desde las tempranas hasta las ya avanzadas, siendo el riesgo de
degeneracin maligna muy variable y relacionado con le edad. La experiencia de la
clnica revela la alta incidencia de tumores de ovario en la etapa del climaterio,
comprendida entre los 35 y 65 aos de edad
1
.
El cistoadenoma seroso de ovario (CSO) es un tipo de tumor derivado del epitelio
superficial (celmico), formado por reas qusticas. El cistoadenoma seroso de
ovario es el tumor ms frecuente de aquellos que provienen del epitelio celmico
superficial. Hay tumores pequeos macroscpicamente y tumores masivos que
ocupan toda la pelvis e incluso la cavidad abdominal. Estas frecuentes neoplasias
qusticas uniloculares estn tapizadas por clulas epiteliales altas, cilndricas y
ciliadas, llenas de un lquido seroso claro y de superficie lisa con abundantes vasos.
Las variedades benigna, limtrofe y maligna representan, en conjunto, 30%
aproximadamente de todos los tumores del ovario. El riesgo de presentar tumores
epiteliales se incrementa con el paso de la edad, ya que pese a que la declinacin de
la funcin ovrica marca el envejecimiento gonadal progresivo, el ovario humano

nunca pierde su capacidad para generar tumores. Por lo general, cuando es
detectado, su tamao es grande, en donde la imagenologa puede ayudarnos a
considerar su diagnstico.

1. Captulo 22 Tumores Benignos de Ovario. En: Novak ER, Jones G., Jokes
HW. Tratado de Ginecologa. 9 ed. Ciudad de la Habana. Editorial Cientfico
Tcnica; 1977.p.432 66.
2. MedlinePlus Enciclopedia Mdica en Espaol: Quistes Ovricos. Disponible
en:
http://vsearch.nlm.nih.gov/vivisimo/cgibin/querymeta?v%3Aproject=medlineplussp
anish&spell=spell&query=Quistes+Ov%C3%A1ricos Acceso: Actualizado 20/6/06.
Captulo XL Tumores Ovricos En: Llusi Botella J, Nez Clavero JA. Tratado de
Ginecologa. Ciudad de la Habana. Editorial Cientfico Tcnica. 1983; T 3.1; p. 751
803.



34.- Femenino que cursa con 36. 5 semanas de gestacin acude al servicio por
referir malestar general, fosfenos, nausea y vmito, aprecia moderada ictericia,
usted sospecha de un sndrome de HELLP Que alteraciones de laboratorio
espera encontrar al confirmar el diagnstico?:

a) Trombocitosis, Enzimas heptica elevadas, Anemia hemoltica.
b) Anemia hemolitica, Trombocitosis, Fosfatasa Alcalina elevada.
c) Anemia Hemoltica, trombocitopenia, enzimas hepticas elevadas.
d) Trombocitopenia, Leucopenia, Hipertensin Arterial.




DEFINICIN:

Es una complicacin de la preeclampsia en la cual adems de la Hipertensin
Arterial y proteinuria hay presencia de anemia hemoltica, enzimas
hepticas elevadas y recuento bajo de plaquetas



EPIDEMIOLOGIA:


Se presenta en un 4 a 10% de las preeclmpticas, diagnosticndose anteparto
en un 70% de los casos preferentemente antes de las 37 semanas, mientras
que el 30% de los casos restantes enferma en los primeros 7 das del
puerperio, sobre todo en las 48 h iniciales.

La proteinuria e hipertensin pueden estar ausentes en un 15 al 20% de los
casos.

Incidencia mayor en multigestantes y en edades avanzadas.

Ocurre ms frecuentemente cuando se demora la salida del feto y cuando se
presenta desprendimiento de la placenta

Mortalidad materna del 24% y mortalidad perinatal del 30-40%.






CLASIFICACION:
Sndrome de HELLP. Clasificacin de Mississipi.
CLASE Plaquetopenia LDH AST-ALT
1 Severa
<50000
>600 IU/L >70 IU/L
2 Moderada
>50000
<100000
>600 IU/L >70 IU/L
3 Ligera
>100000
<150000
>600 IU/L >40 IU/L
<70 IU/L
PE severa
Eclampsia (sin
HELLP)
>150000 <400 IU/L <40IU/L


MANIFESTACIONES CLINICAS:

Malestar general, fatiga y molestias inespecficas 90%

Cefalea 70%

Epigastralgia 64%


Vmito 22%

Fosfenos 15%

Visin Borrosa 11%

Acfenos 3%

Ictericia

Anemia no explicada

Oliguria



Si se aade una HEMORRAGI A HEPTI CA, el pacient e puede quej ar se de
dolor en el HOMBRO DERECHO y EL CUELLO, adems de las molestias
abdominales.

Equimosis en los sitios de punciones venosas, petequias en los sitios de presin
del brazo, pero pueden tener pruebas de Rumpel Leed negativas.

En casos severos se pude presentar ascitis como causa de hipertensin portal.


DIAGNOSTICO:
El diagnstico clnico del sndrome de HELLP se plantea en gestantes o purperas
con preeclampsia severa-eclampsia, excepto en el 15-20%, en las cuales esta
asociacin no puede ser demostrada, en tanto se cumplan los criterios de Sibai:


MANIFESTACIONES CLINICAS:

Malestar general, fatiga y molestias inespecficas 90%

Cefalea 70%

Epigastralgia 64%

Vmito 22%

Fosfenos 15%


Visin Borrosa 11%

Acfenos 3%

Ictericia

Anemia no explicada

Oliguria



Si se aade una HEMORRAGIA HEPTICA, el paciente puede quejarse de
dolor en el HOMBRO DERECHO y EL CUELLO, adems de las molestias
abdominales.

Equimosis en los sitios de punciones venosas, petequias en los sitios de presin
del brazo, pero pueden tener pruebas de Rumpel Leed negativas.

En casos severos se pude presentar ascitis como causa de hipertensin portal.


DIAGNOSTICO:
El diagnstico clnico del sndrome de HELLP se plantea en gestantes o purperas
con preeclampsia severa-eclampsia, excepto en el 15-20%, en las cuales esta
asociacin no puede ser demostrada, en tanto se cumplan los criterios de Sibai:


HEMOLISIS
Frotis perifrico anormal (eritrocitos fragmentados)

Hematocrito (>24%)

Bilirrubina indirecta (>1.2mg/dL)

Deshidrogenasa lctica (>218 UI/L)


ENZIMAS HEPTICAS ELEVADAS


LDH >218UI/L

AST >30UI/L


ALT >37UI/L


PLAQUETAS BAJAS

<100.000/mm3



BIBLIOGRAFIA:

Sibai baha, El sndrome HELLP. Universidad de Valencia , revista quincenal de
Obstetricia clnica y ginecologa, Octubre 2003.
V. Cararach, Sndrome de HELLP y Repercusiones maternas. X curso intensivo de
formacin continuada materno fetal. Enero de 2003.

Toirac, Abelardo. Sndrome de Weistein HELLP Hospital Ginecoobstetrico Tamara
Bunke. Junio 2002

De la Fuente, David. Sndrome HELLP. Medicina Universitria 2003; 5 (19): 101 -9
Andrea G. Witlin, DO, Baha M. Sibai, MD. Diagnosis and Management of women
with Hemolysis Elevate Liver Enzymes, and Pletelet Count (HELLP) syndrome.
Hospital Physician. Febrero 1999.
CIFUENTES B, Rodrigo. Ginecologa y obstetricia basadas en las evidencias.
Bogot: Distribuna, 2006. Sexta edicin. 447 - 283 p.


35.-Femenino de 19 aos, atendida en sala de urgencias ginecoobsttricas,
Antecedente: cursa embarazo de 38 SDG. Exploracin Fsica: en trabajo de parto.
Repentinamente presenta sangrado profuso transvaginal y dolor abdominal. La
causa ms probable de la sintomatologa de esta paciente es:

a) Abruptio placentae
b) Laceracin vaginal por coito
c) Cervicitis
d) Placenta previa



DESPRENDIMIENTO PREMATURO DE PLACENTA NORMOINSERTA (DPPNI):
Constituye la separacin de la placenta de su rea de insercin antes del 3 perodo
del parto. La mortalidad fetal es muy alta (superior al 15%) y la materna es tres
veces superior a la esperada.
La aparicin frecuente de SFA, prematuridad, anemia, etc., hace que el nmero de
secuelas tanto sensitivas como motoras sea alto. Desde el punto de vista materno,
complicaciones secundarias a la hemorragia, a las alteraciones de la coagulacin o
bien a la embolia pulmonar tambin tienen una tasa muy alta.
Desde la antigedad se identific al cuadro clnico caracterizado con la trada
sintomtica de hipertona, metrorragia y muerte fetal, con pronstico materno
comprometido. En 1775, Ricci diferenci la separacin prematura de una placenta
de insercin normal, de aquella de insercin baja; a la primera llam hemorragia
accidental, y a la segunda hemorragia imprevisible. Couvelaire introdujo el
trmino de desprendimiento prematuro de placenta y describi la aparicin de la
apopleja uterina
(tero de Couvelaire)
Obstetricia. Scwarcz, Sala, Duverges. 7 edic. Edit. El Ateneo. (Biblioteca Fac.
Med. UNNE

36.- A 17-year-old patient presents with severe pruritus that is worse at night.
Upon examination of the skin, areas of excoriated papules are observed in the
interdigital area. Family members report similar symptoms. Which of the following
is the most likely diagnosis?


a) Cutaneos larva migrans
b) Contact dermatitis
c) Scabies
d) Dermatitis herpetiformis

Es un padecimiento parasitario de la piel, sumamente pruriginoso, producido por el
caro Sarcoptes Scabiei Hominis. Suele afectar la piel, en forma generalizada con
predominio en pliegues. La incidencia es mundial, sin embargo es mas frecuente en
pases con niveles socioeconmicos bajos debido al hacinamiento y la falta de
higiene. Se puede trasmitir por contacto directo de piel a piel o por ropas o reas
infestadas por el parsito, de ah que con frecuencia sea un padecimiento familiar.
Caractersticas Clnicas
Los sitios de predileccin son los pliegues interdigitales en manos, caras laterales
de dedos, cara anterior de mueca, pliegues axilares anteriores, regin

submamaria, areola, pezones, brazos, antebrazos, caras internas de muslos, pene,
escroto y regin nter gltea.
Las lesiones son ppulas con costras hematicas debidas al rascado intenso de
predominio nocturno. Rara vez se observan los tneles que produce el parsito. En
adultos se respetan las lneas de hebra, mientras que en lactantes o nios pequeos
no sucede esto, siendo en la edad peditrica un cuadro es generalizado.
Debido al prurito intenso y el rascado que este produce, una complicacin
frecuente es la impetginizacin, mientras que la automedicacin es la causa de la
dermatitis por contacto que en ocasiones tambin complica a esta enfermedad.
Executive Committee of Guideline for the Diagnosis, Ishii N. Guideline for the
diagnosis and treatment of scabies in Japan (second edition). J Dermatol. 2008
Jun;35(6):378-93.
- Walton SF and Currie BJ. Problems in Diagnosing Scabies, a Global Disease in
Human and Animal Populations. Clin. Microbiol. Rev. 2007 20: 268-279.
doi:10.1128/CMR.00042-06
- Ulrich R Hengge, Bart J Currie, Gerold Jger, Omar Lupi, Robert A Schwartz.
Scabies: a ubiquitous neglected skin disease. The Lancet Infectious Diseases, Dec
2006;6(12):769-779. doi:10.1016/S1473-3099(06)70654-5



37.-En nuestro pas, la Secretara de Salud ha lanzado una campaa para fomentar
entre el personal de las instituciones gubernamentales, una rutina de ejercicios con
duracin de cinco minutos diarios, esta es una accin especfica de:

a) Restauracin de la salud
b) Promocin de la salud.
c) Medicina preventiva
d) Proteccin de la salud


Las acciones de Promocin de la Salud pretenden fomentar la salud de los
individuos y la colectividad, promoviendo que adopten estilos de vida saludables.
Ello se consigue mediante intervenciones de educacin sanitaria.

Piedrola G. G; Medicina Preventiva y Salud Pblica. Masson, 9 ed. Pg. 15, 16.







38.- Se trata de masculino de 70 aos que consulta por pirosis, disfagia leve
ocasional y episodios de regurgitacin nocturna desde hace 15 das. Comenta que
desde hace 2 aos viene presentando por ocasiones pirosis y regurgitacin. Se
reporta endoscopia alta con esofagitis erosiva grave. El tratamiento
farmacolgico ms adecuado para el paciente, es?

a) Antagonistas de los receptores H2.
b) Sucralfato.
c) Tratamiento combinado con anti-H2 y sucralfato
d) Inhibidores de la bomba de protones.

Se ha demostrado que existe una relacin directa entre la duracin de la supresin
del acido gstrico y la menor acidez del esfago.
Los inhibidores de la bomba de protones (IBP) son los agentes preferidos para la
curacin de las lesiones agudas y para la mantencin de remisin. Los bloqueadores
H2 presentan como nica ventaja su rapidez de accin, pero su potencia es menor.
Su uso de rutina asociado con los IBP no se recomienda, ya que los IBP actan
sobre las bombas de cido activadas y cualquier inhibicin de la secrecin por otro
agente retarda su mximo efecto Los anticidos son tiles para sntomas
ocasionales y su uso no se contrapone con los IBP.
De no existir factores significativos modificables en los hbitos o en la anatoma,
la ERGE debe considerarse frecuentemente una patologa crnica. No existe
evidencia de peso que contraindique el uso crnico de IBP. Los pacientes con
esofagitis erosiva son los ms susceptibles a desarrollar complicaciones que los en
GERD (endoscopy negative
Gastroesophageal reflux disease).Todos los IBP son tiles pero no son
necesariamente iguales. Puede haber diferencias tnicas en el nmero de clulas
parietales o polimorfismo del citocromo p450 con diferente metabolizacin de los
IBP.
La infeccin por Helicobacter pylori confunde cualquier intento de comparar IBP.
La gastritis de predominio antral puede producir hipergastrinemia e
hipersecrecin. Si es de predominio corporal puede disminuir la produccin de
cido. Los estudios comparativos deben hacerse en individuos sin esta infeccin.
La duracin del efecto de los IBP es importante. Muchas publicaciones demuestran
la mayor duracin de la accin de la accin del esomeprazol, pero clnicamente
doble dosis de los otros IBP pueden tener resultados comparables. Katz y cols
compararon los 5 IBP disponibles en pacientes con ERGE, todos resultaron en pH >
4 por al menos 8 a 10 h, pero la duracin de accin del esomeprazol fue superior en
el rango de los pH intragstricos entre 2 a 6.
En los pacientes con enGERD el efecto de los IBP no es tan dramtico, puede
deberse a que no hay tantos elementos objetivos de juicio como la curacin de las
erosiones.

Los IBP pueden usarse una vez al da, en la maana y antes del desayuno. Los
pacientes con dao extraesofgico o cuadros severos obtienen mejor efecto con
doble dosis fraccionada
(Antes de desayuno y cena). Esto mejora el control de la acidez nocturna.
En los pacientes cuya pHmetra demuestre que persiste escape nocturno algunos
favorecen agregar un antagonista H2 al acostarse. Rackoff y cols demostraron que
74% de los pacientes presentaban mejora de los sntomas nocturnos. Una nueva
formulacin de liberacin y absorcin rpida de omeprazol tambin podra ser til
en estos casos.
La ausencia de sntomas no significa ausencia de dao. Los casos graves (y por
supuesto el Barrett) deben tener seguimiento endoscpico para asegurar su
evolucin. Un porcentaje significativo de pacientes sin Barrett continan
presentando RGE patolgico y bajo pH intragstrico a pesar de IBP bien llevado
con total remisin de sntomas.
El primer objetivo es la remisin de los sntomas de reflujo clsico. Esto debe
lograrse en la primera semana si la prescripcin es adecuada. Las manifestaciones
extraesofgicas tardan varios meses en controlarse, pero deben exhibir una
mejora rpida en las primeras semanas que confirme la buena orientacin de las
medidas indicadas.
La estrategia teraputica de comenzar el tratamiento en forma poco agresiva y
escalarlo si no hay respuesta, me parece poco adecuada.
Al paciente se le debe insistir que los frmacos son una parte importante del
tratamiento pero no la nica y que los cambios de hbitos y costumbres, la baja de
peso, el ejercicio
Etc, son claves para el xito.


Bibliografa
1.- Miner P, Katz P, Chen Y, Sostek M. Gastric acid control with esomeprazole,
lansoprazole, omeprazole,
pantoprazole, and rabeprazole: A five-way crossover study. Am J Gastroenterol
2003; 98: 2616-20.
2. Katz P, Miner P, Chen Y, Sostek M. Effects of 5 marketed proton pump
inhibitors on acid suppression relative to a range of pH thresholds. Am J
Gastroenterol 2004; 99: S34.
3.- Rackoff A, Agrawal A, Hila A, et al. Histamine-2 receptor antagonists at night
improve GERD symptoms for patients on proton pump inhibitor therapy. Am J
Gastroenterol 2004; 99: S18.
4.- Castell D, Goldlust B, Morelli G, et al. Omeprazole immediate-release oral
suspension is more effective than pantoprazole delayed-release capsules in
reducing nighttime gastric acidity in GERD patients. Am J Gastroenterol 2004;
99: S39.

5. Milkes D, Gerson L, Triadafilopoulos G. Complete elimination of reflux symptoms
does not guarantee normalization of intraesophageal and intragastric pH in
patients with gastroesophageal reflux disease
(GERD). Am J Gastroenterol 2004; 99: 991-6.


39.- Acude a consulta recin nacido con sospecha de luxacin de la articulacin de la
cadera, la siguiente maniobra es la que nos corrobora dicha patologa:


a) Signo de Galeazzi
b) Signo de Barlow
c) Signo de Pistn
d) Signo de Ortolani


Prueba de Ortolani : Con esta prueba se detecta una cadera ya luxada, se coge con
la mano el miembro flexionado, la cadera se coloca en abduccin mientra se levanta
el fmur con cuidado y se sita los dedos a nivel del trocnter mayor. Si la prueba
es positiva se siente la reduccin de la cadera dentro del acetbulo.
Prueba de Barlow: es una prueba inductora para identificar una cadera inestable
pero an localizada en su sitio; no es una prueba adecuada para diagnosticar luxacin
de cadera. La cadera en aduccin ligera y con la palma de la mano se empuja suave y
cuidadosamente hacia atrs, la presencia de un movimiento de pistn o al
percepcin de una cabeza femoral subluxada sobre el borde posterior del acetbulo.
Prueba de Galeazzi: con el nio acostado se le flexionan las caderas y rodillas de
modo que los talones se apoyen sobre la mesa y reconocer el acortamiento relativo
del muslo.








Skinner, H. Diagnstico y tratamiento en Ortopedia. Ed. Manual Moderno. Mxico,
2004. pp. 625


40.- Una herramienta importante para realizar el siguiente diagnstico es la triada
de Gregg. A cual de las siguientes infecciones nos referimos?

a) Citomegalovirus
b) Toxoplasmosis
c) Rubola
d) Sfilis congnita


Greeg, en 1941, fue el primero que describi la trada caracterstica del SRC:
cardiopata congnita, cataratas y sordera. Posteriormente estudios en animales
corroboraron los mismos hallazgos encontrados en los fetos humanos; defectos
cardiacos, oculares, esquelticos, SNC (caso ndice) restriccin del crecimiento
fetal y bito, siendo mayor el riesgo de teratognesis cuando la infeccin ocurre en
el periodo de la organognesis

La trada de Gregg que consiste en hipoacusia neurosensorial, malfiormaciones
congnitas (PCA o estenosis pulmonar) y anomalas oculares (catarata, glaucoma,
retinitis en sal y pimienta) se presenta en la rubola; otras manifestaciones son:
corioretinitis, prpura trombocitopnica y microcefalia. En la infeccin por
citomegalovirus tambin se presenta microcefalia y coriorretinitis, as como
calcificaciones periventriculares.


BIBLIOGRAFA
1. De Santis M. Cavalliere A, Straface G, Caruso A. Rubella infection in pregnancy.
Reproductive Toxicology 2006;21:390-8.


41.- Mujer de 18 aos diagnosticada de embrazo ectpico mediante ecografa que
tambin muestra gran cantidad de lquido libre en Douglas, con mal estado general.
No tiene hijos y conserva la trompa contralateral en perfecto estado. Cul es el
tratamiento de eleccin?:


a) Expectante y determinacin del B-HCG.
b) Salpingectoma.
c) Legrado de cavidad uterina.
d) Alta y revisin en una semana

SANGRADOS 1era MITAD EMBARAZO
EMBARAZO ECTOPICO Tratamiento
Quirrgico.
Salpingostoma lineal.
Salpingectoma total.
Ordeamiento fimbrial.
Legrado uterino instrumental post-laparoscpico.
(Reaccin deciduoide)
Mdico.
Metrotexate y Mifepristona (RU 486)





42.- Se trata de mujer de 22 aos acude al servicio de ginecologa, por referir
ciclos opso-menorreicos, desde el inicio de su menarquia, en los ltimos 7 das ha
incrementado 15 Kg. de lo que pesaba habitualmente. Exploracin Fsica: acn facial
importante, as como bigote.

El diagnostico ms probable en esta paciente es:

a) Sx. De Asherman
b) Sx. Amenorrea Galactorrea
c) Sx. Karman

d) Sx. Stein Leventhall


Sndrome de Ovario Poliqustico (SOP) es uno de los ms comunes trastornos
endocrinos que afectan a las mujeres alrededor del 5% al 10% de las mujeres en
edad reproductiva (12-45 aos) y se piensa que es una de las principales causas de
la infertilidad femenina.
Las caractersticas principales son la obesidad, anovulacin (dando lugar a la
menstruacin irregular) o amenorrea, acn, y las cantidades excesivas o los
efectos de andrognicos (masculinizantes) hormonas.
Los sntomas y la severidad del sndrome varan mucho entre las mujeres. Si bien
las causas son desconocidas, resistencia a la insulina, la diabetes y la obesidad
estn fuertemente correlacionadas con el SOP.
Bulun SE, Adashi EY. The physiology and pathology of the female
reporductive axis. In: Kronenberg HM, Melmed S, Polonsky KS, Larsen PR,
eds. Williams Textbook of Endocrinology. 11th ed. Philadelphia, Pa: Saunders
Elsevier; 2008:chap 16.


43.- Despus de un trabajo de parto con expulsin normal, y tras una hora
aproximada en periodo de alumbramiento en el que se practic masaje uterino y se
increment moderadamente la dosis de oxitocina, no aprecian signos de
desprendimiento placentario, se indica una extraccin manual de placenta, que
resulta imposible por no existir plano de separacin entre la placenta y la pared
uterina. El diagnstico ms probable es?:


a) Engatillamiento placentario.
b) Placenta adherente por acretismo placentario.
c) Placenta succenturiata con cotiledn aberrante.
d) Placenta circunvalata


Se denomina a la placenta como acreta cuando sta se implanta en zonas donde la
decidua es deficiente o anormal y por tanto hay una infiltracin del miometrio por
vellosidades coriales; esta infiltracin puede ser focal, parcial o total. A su vez
esta condicin se subdivide en acreta, increta y percreta. La placenta increta y
percreta infiltran todo el espesor de la pared miometrial, en la percreta adems
las vellosidades, perforan la serosa y llegan en algunas ocasiones a infiltrar rganos
vecinos, especialmente la vejiga. La PA est limitada a la superficie miometrial. La
frecuencia de presentacin del AP vara entre 10 y 48 por 10.000 partos. (Oishi A
1999, Hung TH 1999, Zaki ZM, 1998).

En las mujeres con acretismo placentario se han visto factores de riesgo, dentro
de los cuales se encuentran:
1. Edad y multiparidad: La presentacin AP, aumenta con la paridad de la
paciente y la edad, siendo muy rara en primparas.
2. Placenta previa: esta se ha encontrado en el 30% de los casos de PA.
Igualmente se ha visto PA en el 9.3% de las pacientes con placenta previa
(Miller DA, 1997).
3. Cesrea anterior, o cirugas uterinas previas: Se ha visto este
antecedente en el 25% de los casos. En el 29% de los casos la PA estaba
implantada en la cicatriz uterina y solo en el 5% la placenta estaba
implantada en otro sitio. (Miller Da, 1997).
4. Dilatacin y legrado, en el 25% de los casos.
5. Infeccin uterina previa, remocin manual de la placenta, leiomiomas y
otras anomalas uterinas: La asociacin con estas entidades es
inconstante.
6. Niveles anormalmente elevados de feto-protena y de b-HCG, en el segundo
trimestrre. (Hung TH, 1999).
Una placenta adherente o penetrante no es fcil de diagnosticar antes del
alumbramiento. Despus de ste, se manifiesta como retencin placentaria y
sangrado uterino. El diagnstico generalmente se realiza, despus de intentar la
extraccin manual de la placenta.
Las manifestaciones clnicas propias de la placenta adherente, de la placenta
acreta y de la placenta increta, consisten en una manifiesta dificultad o
imposibilidad para la expulsin o extraccin de la placenta. Como consecuencia de la
atona parcial y de la hemostasis insuficiente en las zonas de despegamiento
placentario, se producir una hemorragia ms o menos grave que en nada se
diferenciar de la hemorragia de la atena uterina. Y no ser solamente al intentar
el alumbramiento artificial que se pondran de manifiesto las razones ntimas de la
retencin placentaria; alumbramiento que ser engorroso en. la placenta ahderente
e imposible en las variedades acreta e increta.
En varias ocasiones puede no existir hemorragia y en estos casos la nica
manifestacin de este estado morboso ser la prolongacin del perodo del
alumbramiento. La placenta adherente, como toda placenta retenida, es pronto
presa de un proceso infeccioso sin embargo se han sealado casos de placentas
retenidas aspticamente durante muchos mese3. al cabo de los cuales han sido
expulsadas sin causar trastorno alguno; pero hay que hacer observar que en estas
enfermas se ha tratado de retensin de mebranas por abortes ovulares.


1. Arredondo-Soberon F, Sabella V, Garza-Leal J, Valente PT. Placenta
increta en primer trimestre de embarazo. Ginecol Obstet Mex 1995; 63:
279-81.
2. Cantanzarite V, Stanco L, Schrimmer S et al. Managing placenta
previa/accreta. Contemp Obstet Gynecol 1996; 41: 66-95.
3. Ecker JL, Sorem KA, Soodak L, et al. Placenta Increta Complicating a Firs-
Trimester Abortion A case report. Journal Reproductive Medicine. 1992;
37-10.
4. Finberg G, William J. Placenta accreta: prospective sonographic diagnosis in
patients with placenta previa / accreta. Contemp Obstet Gynecol 1996: 41:
66-95.
5. Gist RS, Voung V, Brody S, Rees P, Landry AD. Placenta increta occurring in
a bligter ovum. South Med J. 1996; 89(5): 545-7.
6. Harden,MA, Walters MD, Valente PT Postabortal hemorahage due to
placenta increta: A case report. Obstet Gynecol. 1990; 75: 523.
7. Hudon L, Belfort MA, Broome DR. Dosis and management of placenta
percreta: A review. Obster Gynecol survey 1998; 53: 509-517.
8. Hung TH, Shau WY, Hsieh CC, et al. Risk factors for placenta accreta.
Obstec Gynecol 1999; 93: 545-50.
9. Kinoshita T, Ogawa K, Yusumizu T, Kato J. Spontaneous rupture of the
uterus due to placenta percreta at 25-weeks gestation: a case report J
Obster Gynaecol Res 1996; 22: 125-8.
10. Kirkinen P, Helin-Martikainen HL, Vanninen R, Patanen K. Placenta accreta:
imaging by gray-scaleand contrast enhanced color Doppler somography and
magnetic resonance imaging. J Clin Ultrasound 1998; 26: 90-4.

44.- Femenino de 22 aos, que presenta una tumoracin de 2 cm de dimetro en el
cuadrante nfero-externo de la mama izquierda, indolora, de consistencia firme,
superficie lisa, forma ovoidea, mvil y bien delimitada del parnquima vecino, sin
antecedentes de derrame por el pezn, sin piel de naranja ni retraccin del
pezn, el diagnstico presuncional es:


a) Carcinoma.
b) Ectasia de los conductos mamarios.
c) Fibroadenoma.
d) Quiste solitario.


FIBROADENOMA MAMARIO
Tumor benigno ms frecuente en las mujeres entre los 20 y 35 aos.
ETIOLOGIA
Existen mltiples teoras siendo la ms aceptada la hormonal, generalmente son
nicos, solo el

20% son mltiples o bilaterales. De tamao variable hasta de 10 cm. Ocupa el
13.6% de la patologa mamaria benigna.
CUADRO CLNICO
Lesin nodular de consistencia dura, de larga evolucin y no dolorosa. Normalmente
llegan a los 3 cm. De dimetro. Durante la fase tarda del ciclo menstrual el tumor
suele presentar un leve aumento de tamao. Durante la menopausia presentan
regresin hasta la calcificacin (signo de palomitas de maz).
DIAGNOSTICO
Es clnico, se presenta como un tumor bien delimitado, desplazable, no adherido a
piel ni aplanos profundos, liso o multilobulado en ocasiones. Se localiza
frecuentemente en cuadrantes externos.
EXAMENES DIAGNOSTICOS
ULTRASONIDO MAMARIO.- Identifica un ndulo slido, bien delimitado de
bordes regulares .
TRATAMIENTO.-
Conservador con vigilancia estrecha dependiendo del tamao y en caso de ser
necesario exresis del ndulo para estudio histopatolgico



hospitalgeneral.salud.gob.mx/
BIBLIOGRAFIA
1. Snchez BC. Tratado de Enfermedades de la glndula mamaria. Ed. Manual
Moderno. Cap. 13- 15.
2.- De Vita V. Cancer of the Breast. In Cancer: Principles and Practice of
Oncology: Fifth Ed. Philadelphia: Lippincott-Raven, Chapter 36; pp: 1521-1616.
3.-Consenso Nacional Acerca del Tratamiento de Cncer de Mama. En Tumores de
mama: Diagnstico y Tratamiento. 2 Ed. McGraw-Hill Interamericana; pp: 119-126.
4.-Eberlein T. Current management of carcinoma of the breast. Ann Surgery 1994;
220: 121-136.
5. Encyclopedie Medico. Chirurgicale Praxis Medica, Editions Techiques de Mexico,
tomo 5, ao 2005.



45.- Se trata de mujer de 20 aos con menarca a los 12 aos ritmo menstrual
45x4. Refiere vida sexual activa desde los 17 aos con frecuencia de 4 veces por
semana. A la exploracin se encuentra acn intenso en la frente, mejillas y mentn.
Acude a consulta por que desea adoptar un tratamiento anticonceptivo por va oral,
lo mas adecuado es:

a) Norgestimato
b) Gestodeno
c) Levonorgestrel

d) Ciproterona



El efecto antiandrognico especfico del acetato de ciproterona acta por
inhibicin competitiva de la unin de la 5 - alfa - dihidrotestosterona con el
receptor citoslico de las clulas blanco, que disminuye la produccin y la excrecin
de sebo y el aumento y el desarrollo del vello.
Es un derivado de la 17 - alfa - hidroxiprogesterona que posee accin
progestgena. Su accin antigonadotrfica se suma a la del etinilestradiol. El
acetato de ciproterona no posee accin estrognica sino un efecto antiestrognico,
y tampoco posee accin nociva sobre la funcin de la corteza suprarrenal;


Indicaciones en la mujer: Manifestaciones de androgenizacin de grado severo,
por ejemplo, hirsutismo grave, alopecia androgentica de tipo grave, a menudo
acompaados por manifestaciones graves de acn y/o seborrea.
Indicaciones en el hombre: Atenuacin del impulso en las desviaciones sexuales.
Tratamiento antiandrgeno del carcinoma de prstata inoperable.

BIBLIOGRAFA
1. Swift S. Current opinion on the classification and definition of genital tract
prolapse. Curr Opin Obstet
Gynecol 2002; 14: 503-7.
2. De Caro R, Aragona F, Herms A, Guidolin D, Bizzi E, Pagano F. Morphometric
analysis of the fibroadipose tissue of the female pelvis. J Urol 1998; 160: 707-13.
3. Gill E, Hurt W. Pathophysiology of pelvic organ prolapse. Clin Obstet Gynecol
1998; 25(4): 757-69.
4. DeLancey, J. Anatomic aspects of vaginal eversion after Hysterectomy. Am J
Obstet Gynecol. 1992;
166(6 pt 1): 1717-24.


46.- Femenino de 28 aos de edad, que acude a consulta prenatal de rutina, G5 P4,
con 28 SDG, refiere que no ha sentido movimiento fetal durante los ltimos 2 das.
Su embarazo ha sido complicado debido a que padece hipertensin crnica, para lo
cual se le recetaron tabletas de alfa-metildopa 2 veces al da. Al examen, su FU es
de 30cm, y las maniobras de Leopold demuestran que el feto se encuentra en
situacin transversa. Su TA es 145/85mmHg. No se encuentra latido cardiaco con
el Doppler. Cul de los siguientes es el paso ms apropiado a seguir en el manejo?



a) Realizar un test sin estrs
b) USG
c) Amniocentesis
d) Beta-HCG (cuantitativa)

Probable bito:
Sintomatologa y diagnstico Signos funcionales: No se perciben movimientos
fetales por 12-24 horas. Disminucin o ausencia de sntomas y/o signos como
nauseas vmito, hipertensin, albuminuria) Paraclnicos: *ecografa: diagnstico
precoz y exacto: Doppler. *Radiologa: hay 3 signos: +deformacin del crneo
+curvatura y torsin de la columna +presencia de gas en el feto *lquido amnitico:
puede estar meconiado, o sanguinolento Signos locales: en los senos hay secrecin
calostral, sangrado leve y oscuro por vagina, el feto se vuelve blando a la palpacin,
fetocardia (-), puede haber detencin y/o disminucin de la altura uterina, bajo
peso corporal, entre otros


Bibliografa: 1. OBSTETRICIA, Schwarcz R, editorial El ateneo, 2003. 2. Sociedad
espaola de ginecologa y obstetricia, junio 2002. 3. OBSTETRICIA CLNICA,
Llaca V, edicin 2000, captulo 24; Pg, 315-316.



47.- Femenino de 36 aos, es atendida en consulta externa con reporte de
papanicolaou que reporta un NIC I, la especuloscopa se observa crvix con
ectropin periorificiario.
El agente etiolgico ms probable causante de esta infeccin es:


a) Neisseria gonorreae.
b) Clamidya trachomatis.
c) Virus del papiloma humano.
d) Treponema pallidum.

















9.5.2 Las pacientes a quienes se les realiz citologa cervical, cuyo resultado es
LEIBG (infeccin por VPH, displasia leve o NIC 1); LEIAG (displasia moderada y
grave o NIC 2 y 3) o cncer deben enviarse a una clnica de colposcopa, para
realizar estudio colposcpico.
9.5.3 Si el resultado de la citologa es LEIBG, la colposcopa es satisfactoria y sin
evidencia de LEIBG, se realizar control citolgico en un ao (Apndice Normativo
A)
9.5.4 Si la citologa es de LEIBG, la colposcopa es satisfactoria y existe evidencia
de lesin, se debe tomar una biopsia dirigida.
9.5.4.1 Si la biopsia dirigida es negativa, se realizar nueva colposcopa para
verificar el diagnstico y en caso necesario, tomar nueva biopsia dirigida y
revalorar.
9.5.4.2 Si la biopsia dirigida es reportada como LEIBG se podr dar tratamiento
conservador: criociruga, electrociruga o laserterapia (slo si cumple con las
condiciones referidas en el Apndice 1) o se podr mantener a la paciente en
vigilancia en la clnica de colposcopa, con colposcopa y estudio citolgico cada seis
meses, durante 24 meses.
Jueves 31 de mayo de 2007 DIARIO OFICIAL (Primera Seccin)
9.5.4.3 Si la biopsia dirigida es reportada como LEIAG (Lesin Intraepitelial
Escamosa de Alto Grado) se realizar tratamiento conservador (electrociruga o
laserterapia). En las mujeres posmenopusicas, dependiendo de las condiciones
anatmicas del crvix, se realizar tratamiento conservador en la clnica de
colposcopa o tratamiento quirrgico (histerectoma extrafascial) en el servicio que
corresponda.
9.5.4.4 Si la biopsia dirigida reporta cncer microinvasor o invasor, la paciente se
transferir a un Servicio o Centro Oncolgico para su tratamiento
correspondiente.
9.5.4.5 Si la citologa reporta LEIBG y la colposcopa es no satisfactoria, se
tomar cepillado endocervical (Apndice Normativo A)
9.6 En caso de colposcopa no satisfactoria, negativa a LEIBG y con cepillado
endocervical negativo, se continuar su control en la clnica de colposcopa en seis
meses, con colposcopa y citologa.
9.6.1.1 Si el cepillado endocervical reporta LEIBG se tratar a la paciente como
LEIAG, con mtodos conservadores escisionales.



Jueves 31 de mayo de 2007 DIARIO OFICIAL (Primera Seccin)
Modificacin a la Norma Oficial Mexicana NOM-014-SSA2-1994, Para la
prevencin, deteccin, diagnstico, tratamiento, control y vigilancia
epidemiolgica del cncer crvico uterino.

Al margen un sello con el Escudo Nacional, que dice: Estados Unidos Mexicanos.-
Secretara de Salud.
MODIFICACION A LA NORMA OFICIAL MEXICANA NOM-014-SSA2-1994,
PARA LA PREVENCION,
DETECCION, DIAGNOSTICO, TRATAMIENTO, CONTROL Y VIGILANCIA
EPIDEMIOLOGICA DEL CANCER CERVICO UTERINO.


El agente etiolgico del cncer de cuello uterino es el papiloma virus humano
(hpv).
Existen lesiones precursoras del cncer cervical, son las llamadas lesiones
intraepiteliales de cuello uterino (tambin conocidas como sil: squamous
intraepithelial lesion) son lesiones que no atravesaron la membrana basal del
epitelio y que por lo tanto no pueden invadir ni diseminarse por el resto del cuerpo,
como s lo hace un cncer invasor. anteriormente a las lesiones intraepiteliales se
las llamaba neoplasia intraepitelial cervical (tambin conocidas como cin: cervical
intraepithelial neoplasia), y anteriormente se las llamaba displasias de cuello
uterino.


48.- Se trata de paciente femenino de 37 aos de edad, G-3, C-2. Es ingresada a
hospital presentando cefalea, acfenos, fosfenos y epigastralgia en barra con
embarazo de 34 semanas. E.F T/A 160/110, FC 84 x, FR 18 x, no presenta
fiebre, somnolienta, sin agregados cardioventilatorios, hepatalgia. F.U. de 25 cm.
Producto nico vivo. FCF 110 lpm, genitales sin prdidas ni modificaciones
cervicales. Laboratorio: hb 9.8 g/dl, plaquetas de 54 mil, TP 11 seg TPT 27, TGO
160 ng/dl TGP 160 ng/dl, hiperbilirrubinemia indirecta, albuminuria 300 mg/dl,
Acido rico de 8.1 mg/dl, creatinina de 1.5 mg/dl.
El diagnstico ms probable es:


a) Sx anticuerpos antifisfolpidos
b) Prpura trombocitopnica trombtica
c) Sindrome de hellp.
d) Hgado graso




DEFINICIN:
Es una complicacin de la preeclampsia en la cual adems de la Hipertensin
Arterial y proteinuria hay presencia de anemia hemoltica, enzimas hepticas
elevadas y recuento bajo de plaquetas



MANIFESTACIONES CLINICAS:
Malestar general, fatiga y molestias inespecficas 90%
Cefalea 70%
Epigastralgia 64%
Vmito 22%
Fosfenos 15%
Visin Borrosa 11%
Acfenos 3%
Ictericia
Anemia no explicada
Oliguria








BIBLIOGRAFIA
Sibai baha, El sndrome HELLP. Universidad de Valencia , revista quincenal de
Obstetricia clnica y ginecologa, Octubre 2003.
V. Cararach, Sndrome de HELLP y Repercusiones maternas. X curso intensivo de
formacin continuada materno fetal. Enero de 2003.
Toirac, Abelardo. Sndrome de Weistein HELLP Hospital Ginecoobstetrico Tamara
Bunke. Junio 2002

De la Fuente, David. Sndrome HELLP. Medicina Universitria 2003; 5 (19): 101 -9
Andrea G. Witlin, DO, Baha M. Sibai, MD. Diagnosis and Management of women
with Hemolysis Elevate Liver Enzymes, and Pletelet Count (HELLP) syndrome.
Hospital Physician. Febrero 1999.
CIFUENTES B, Rodrigo. Ginecologa y obstetricia

49.- Se trata de paciente femenino de 21 aos que acude a consulta refiriendo
presentar 6 meses sin regla. Se manifiesta preocupada por su sobrepeso, ha
estado a dieta y ha perdido 6 kg en 8 meses. Actualmente pesa 46 Kg. con talla de
1,65 mts. Signos vitales dentro de sus parmetros normales. Se realiza prueba de
embarazo con resultado negativa. Cul es la causa ms probable de su
amenorrea?:


a) Hipogonadismo hipogonadotropo.
b) Disgenesia gonadal.
c) Sndrome de ovario poliqustico.
d) Adenoma hipofisario.





El hipogonadismo femenino se caracteriza por un fallo gonadal debido a la
alteracin del propio ovario o secundario a un fallo hipotlamo-hipofisario.
Este fallo de la gnada puede ocurrir en distintos momentos de la vida y por causas
diversas, lo que va a condicionar una presentacin clnica diferente.
El ovario posee dos funciones relevantes: producir gametos femeninos (oognesis)
y secretar hormonas esenciales en la regulacin de la funcin reproductora y que
influyen en la diferenciacin y el desarrollo de los rganos sexuales
(hormonognesis).
Los mecanismos celulares y moleculares responsables de la produccin de oocitos y
hormonas por la gnada femenina son parcialmente independientes entre s. Sin
embargo, ambas funciones son llevadas a cabo de modo concertado gracias a la
accin de un complejo sistema de control que implica interacciones mltiples entre
el hipotlamo, la hipfisis y el ovario.
Para la adquisicin de la capacidad reproductora a partir de la pubertad se
requiere una adecuada formacin e integracin funcional durante las etapas
tempranas del desarrollo. Los elementos que componen este eje reproductor son:
1) el sistema neuronal hipotalmico responsable de la produccin del neuropptido
GnRH; 2) las clulas gonadotropas de la hipfisis anterior que secretan LH
(hormona luteinizante) y FSH (hormona folculo estimulante), y 3) el ovario.
Hay que destacar que la formacin de la gnada y de los elementos hipotlamo-
hipofisarios del eje gonadotropo se llevan a cabo de modo independiente en etapas
tempranas del desarrollo embrionario, producindose posteriormente su
integracin funcional en circuitos de retroalimentacin positivos y negativos,
esenciales para una correcta funcin reproductora.


FALLO GONADAL PRIMARIO O HIPOGONADISMO
HIPERGONADOTRFICO
Incluye a aquellos pacientes en los que la produccin y accin de los esteroides
estn reducidas. En los primeros 4 aos de vida y a partir de los 9-10 aos, la
hipfisis, al faltarle la retroalimentacin negativa de estos esteroides, aumenta la
produccin de gonadotropinas. Sin embargo, la elevacin de las gonadotropinas no
puede normalizar la funcin gonadal.
Las causas pueden ser congnitas y adquiridas.




Etiologa comn a ambos sexos
Alteraciones congnitas del receptor de las gonadotropinas
Recientemente se han clonado y mapeado los genes de los receptores de

as gonadotropinas (LH-R y FSH-R) (20), estando ambos en 2p21, y se han
identificado diversas mutaciones y deleciones que conducen a una disminucin de la
funcin de estos receptores, independiente de las gonadotropinas (hipogonadismo
hipergonadotropo); en otros casos se produce un aumento de la funcin de estos
receptores.
Mutaciones que ocasionan prdida de la funcin del receptor de la LH
Hasta la fecha se han encontrado seis sustituciones de una base del gen del LH-R
que conducen a dos mutaciones nonsense o sin sentido que ocasiona un codn de
parada, y a cuatro mutaciones missense o mutaciones con error de sentido en los
que cambia un aminocido en la protena. La mayora son homocigotos y se
considera que el trastorno es A-R. Clnicamente, existe una forma grave y una
menos grave, lo que tiene relacin con la actividad del receptor. En las mujeres hay
amenorrea, falta de respuesta del ovario a la LH y respuesta normal a la FSH con
desarrollo folicular normal.
Mutaciones que ocasionan prdida de la funcin del receptor de la FSH
Son ms raras. Clnicamente las mujeres homocigotas presentan disgenesia ovrica
e hipogonadismo hipergonadotropo, mientras que las mujeres heterocigotas son
fenotpicamente normales.


BIBLIOGRAFA
1. Caete R, Jimnez L. Ontogenia y diferenciacin gonadal. Gnadas.
Actualizaciones en Endocrinologa.
McGraw-Hill Interamericana. Captulo 1, 2000.
2. Barrio R, Ezquieta B. Mecanismos genticos de la diferenciacin sexual: sus
alteraciones.
Gnadas. Actualizaciones en Endocrinologa. McGraw-Hill Interamericana.
Captulo13,
2000.
3. De Roux N, Morel Y, Hardelin JP. Genetic anomalies of the gonadotropic axis.
Rev Prat
1999; 49 (12): 1277-1282.
4. Tena-Sempere M, Aguilar E, Pinilla L. Fisiologa del eje hipotlamo-hipofiso-
ovrico. Gnadas.
Actualizaciones en Endocrinologa. McGraw-Hill Interamericana. Captulo 2, 2000.
5. Labarta JI y cols. Hipogonadismo hipergonadotrpico. En Pubertad normal y
patolgica, 2. curso de postgrado. Valladolid, 1996.
6. Veldhuis JD. Neuroendocrine mechanisms mediating awakening of the human
gonadotropic axis in puberty. Pediatr Nephrol 1996; 10 (3): 304-317.
7. Hopwood NJ. Pathogenesis and management of abnormal puberty. Spec Top
Endocrinol
Metab 1985; 7: 175-236.

8. Minagawa M, Yasuda T, Niimi H J. Spinal and femoral bone mass accumulation
during normal adolescence: comparison with female patients with sexual precocity
and with hypogonadism. Clin Endocrinol Metab 1996; 81 (3): 1248-1253.
9. Park KH, Lee SJ, Kim JY, Kim JY, Bai SW, Kim JW. A concomitant decrease in
cortical and trabecular bone mass in isolated hypogonadotropic hypogonadism and
gonadal dysgenesis. Yonsei Med J 1999; 40 (5): 444-9.




50.- Femenino de 25 aos con antecedentes de G/2, P/1, C/1 acude al servicio de
consulta externa, refiere que presenta una secrecin transvaginal bastante lquida,
de baja viscosidad, maloliente de color amarillo y gris, espumoso. Esta entidad es
propia de infeccin por:


a) Cndida albicans
b) Gardenerella
c) Tricomonas
d) Gonococos


El protozoario Thricomona Vaginalis es el responsable del 25% de las
vaginitis.
20-50% de las mujeres cursan asintomticas.
La tricominiasis es predominantemente una infeccin transmitida
sexualmente.
Debe sospecharse de abuso sexual en caso de encontrar Trichomona en
pacientes peditricos.

DIAGNOSTICO:
Flujo vaginal amarillo-verdoso con burbujas, muy ftido, irritacin vulvo-
vaginal, disuria.
El pH suele ser mayor de 4.5.
Estudio en fresco en donde se observa al microscopio el organismo
flagelado caracterstico.
75% se diagnostican con el estudio del Papanicolaou.


Referencias bibliogrficas:

1. Secretara de Salud. Norma Oficial Mexicana NOM -039-SSA2-2002, Para la
prevencin y control de las infecciones de transmisin sexual. D.O.F. 19 de
Septiembre 2003.

2. Kettler H, White K, Hawkes S. Mapping the landscape for sexually transmitted
infections: key findings and recommendations. Geneva, TDR (TDR/STI/ IDE/04.1).
3. CDC. Trends in Reportable Sexually Transmitted Diseases in the United States.
CDC,
National Report. 2004
4. Distribucin de los casos nuevos de enfermedades por mes Estados Unidos
Mexicanos
2004. Sistema nico de Informacin para la Vigilancia Epidemiolgica/Direccin
General
de Epidemiologa/SSA
5. Aral S O. Sexual risk behaviour and infection: epidemiological considerations.
Sex. Transm.
Inf. 2004;80:8-12











51.- En la sala de urgencias recibe a una paciente que inicia con convulsiones por
preclampsia usted decide administrar el siguiente frmaco ya que es el de
eleccin en sta patologa:


a) Diacepam.
b) Fenitona.
c) Sulfato de magnesio.
d) Donadores de xido ntrico.


Manejo de la Preeclampsia

1. Manejo ambulatorio: HTA sin proteinuria significativa, se recomienda el reposo
en cama. Monitoreo de TA, peso, presencia de protenas en orina. Ecografas
peridicas para ver el feto y evaluar posibles retardo de crecimiento.
2. Manejo hospitalario: para mujeres con HTA inducida por el embarazo y 2+ o
ms o proteinuria significativa y en quienes fall el manejo ambulatorio.

3. Laboratorio y evaluacin del peso: debe realizarse diariamente. Evaluacin de la
dinmica fetal. Monitoreo de sntomas como cefalea, alteraciones visuales y dolor
epigstrico.
4. El parto es el tratamiento de eleccin: el cual debe realizarse cuando el feto
est maduro pero puede realizarse en forma temprana si la salud de la madre est
en peligro o si hay evidencia de distress fetal. El parto est indicado cuando la
paciente cumple con los criterios de preeclampsia severa. Betametasona 12.5 mg
IM dos veces por da puede estimular la maduracin de los pulmones fetales.
5. Terapia antihipertensiva: est indicada slo si la TA es persistentemente >
160/110 , es importante disminuir la TA hasta una diastlica de 90 a 100 mmHg
porque la presin normal podra resultar en hipoperfusin de la placenta. Los
diurticos nunca estn indicados, estas pacientes ya son hipovolmicas. Los IECA
no deben ser usados durante el embarazo. Las medicaciones de largo plazo,
incluyen alfa metildopa, atenolol y labetalol.
6. Terapia anticonvulsivante:
A- Profilaxis de las convulsiones: est indicada en todas las pacientes pre-
eclmpticas durante el trabajo de parto y el parto y por un mnimo de 24 hs luego
del mismo. Algunos mantienen la terapia con magnesio hasta que comienza la
diuresis. El Sulfato de Magnesio es la droga de eleccin. La dosis profilctica es de
4 a 6 g de sulfato de magnesio IV y contina con 2 g c/ hora.
B- Tratamiento de las convulsiones: Sulfato de Magnesio 1 g/min IV hasta
controlar las convulsiones hasta un mximo de 4 a 6 g. El nivel teraputico es de 4
meq/l. Toxicidad del magnesio: ausencia de reflejo patelar, debilidad muscular,
parlisis respiratoria y depresin cardaca, 10 ml al 10 % de gluconato de calcio
puede ser administrada IV. La terapia con sulfato de magnesio contina por lo
menos 24 horas en el post parto, la terapia puede detenerse si la excrecin
urinaria es > 200 ml/h por cuatro horas consecutivas.
C- Prevencin: 81 mg de aspirina diarios pueden ser administrados luego del
primer trimestre en mujeres con hipertensin crnica o historia previa de
preeclampsia, sin embargo la eficacia de esta indicacin ha sido cuestionada.

Myers JE, Baker PN. Hupertensive diseases and eclampsia. Curr Opin Obstet
Gynecol 2002; 14: 119-125
2. Tierney, McPhee, Papadakis. Diagnstico clnico y tratamiento 2003. 38 ed,
Mxico, Manual Moderno, 2003: 770-773
3. Wilson MI, Goodwin TM, Pan VI, Ingles SA. Molecular epidemiology of
preeclampsia. Obstet and Gynecol Survey 2003; 58(1):39-66
4. Burrow GM. Complicaciones mdicas durante el embarazo. 4 ed, Mxico,
McGraw-Hill panamericana: 1996: 1-25
5. Guyton AC, Hall JE. Embarazo y lactancia en: Tratado de fisiologa mdica, 10
ed, Mxico, McGraw-Hill Interamericana 2001: 1135-45
6. Vaticon D. Fisiologa de la fecundacin, embarazo, parto y lactancia, en:
Tresguerres JAF. Fisiologa Humana. Mxico, Interamericana McGraw-Hill, 1992:
1086-1109

7. Pridjian G, Puschett JB. Preeclampisa. Part 1: Clinical and Pathophysiologic
Considerations. Obstet and Gynecol Survey 2002; 57 (9): 598-618
8. Pridjian G, Puschett JB. Preeclampisa. Part I1: Experimental and Genetic
Considerations. Obstet and Gynecol Survey 2002; 57 (9): 619-40
9. IMSS. Embarazo de alto riesgo. Gua diagnstica teraputica. Rev Med IMSS
1998; 36(1):45-60


52.- Mujer de 34 aos que cursa con 39 SDG; a la exploracin fsica reflejos
patelares hiperactivos, inquieta, se reportan cifras de TA 145/95, se realiza
laboratorio que reporta proteinuria 2+,. El diagnstico ms probable es:


a) Glomerulonefritis aguda
b) Hipertensin esencial
c) feocromocitoma
d) Preeclampsia


La hipertensin es la complicacin mdica ms comn del embarazo , aunque para
algunos autores es la segunda complicacin mdica del embarazo slo despus de la
anemia; es ms frecuente en jvenes durante el primer embarazo y en nulparas de
mayor edad, hipertensas previas y diabticas.

En Mxico, tambin es la complicacin ms frecuente del embarazo, la incidencia es
de 47.3 por cada 1 000 nacimientos y es adems, la primera causa de ingreso de
pacientes embarazadas a las unidades de terapia intensiva (debido a hemorragia
masiva, para recibir soporte hemodinmico), segn la secretara de salud (2001) la
mortalidad por complicaciones del embarazo ocupa el 15 lugar en la mortalidad
hospitalaria en general. Adems, la tasa de preeclampsia se ha incrementado 40%
en el periodo entre 1990 y 1999

y constituye hasta 40% de los partos prematuros
iatrognicos.
.

Preeclampsia
La preeclampsia es un sndrome clnico caracterizado por hipertensin con
disfuncin orgnica mltiple, proteinuria, edemas.
Es definida como un incremento de al menos 140/90 mmHg despus de la semana
20 de gestacin, un incremento en la presin sangunea diastlica de al menos 15
mmHg respecto a un nivel previo a la semana 20 combinado con proteinuria (> 300
mg en 24 horas). Las mediciones de la presin arterial citadas deben ser medidas
al menos 2 ocasiones con por lo menos 6 horas de separacin. La proteinuria puede
ser una toma simple de orina al azar que indique al menos 30 mg/dL
3
++ en dos

muestras de orina
1
segn el tipo de prueba. El criterio del incremento de 30 mmHg
en la presin sistlica y/o 15 mmHg en la presin diastlica respecto a valores
previos a la semana 20 de gestacin ha sido eliminado por ser poco especfico
15
















Myers JE, Baker PN. Hupertensive diseases and eclampsia. Curr Opin Obstet
Gynecol 2002; 14: 119-125
2. Tierney, McPhee, Papadakis. Diagnstico clnico y tratamiento 2003. 38 ed,
Mxico, Manual Moderno, 2003: 770-773
3. Wilson MI, Goodwin TM, Pan VI, Ingles SA. Molecular epidemiology of
preeclampsia. Obstet and Gynecol Survey 2003; 58(1):39-66
4. Burrow GM. Complicaciones mdicas durante el embarazo. 4 ed, Mxico,
McGraw-Hill panamericana: 1996: 1-25
5. Guyton AC, Hall JE. Embarazo y lactancia en: Tratado de fisiologa mdica, 10
ed, Mxico, McGraw-Hill Interamericana 2001: 1135-45
6. Vaticon D. Fisiologa de la fecundacin, embarazo, parto y lactancia, en:
Tresguerres JAF. Fisiologa Humana. Mxico, Interamericana McGraw-Hill, 1992:
1086-1109
7. Pridjian G, Puschett JB. Preeclampisa. Part 1: Clinical and Pathophysiologic
Considerations. Obstet and Gynecol Survey 2002; 57 (9): 598-618
8. Pridjian G, Puschett JB. Preeclampisa. Part I1: Experimental and Genetic
Considerations. Obstet and Gynecol Survey 2002; 57 (9): 619-40
9. IMSS. Embarazo de alto riesgo. Gua diagnstica teraputica. Rev Med IMSS
1998; 36(1):45-60


53.- Se trata de paciente femenino que cursa con 12 semanas de gestacin, lleva
tres das sangrando por genitales, con nuseas continuas y constantes, tero
mayor aumentado de tamao y valores de beta HCG muy elevados, debemos
sospechar de:


a) Amenaza de aborto.
b) Aborto diferido.
c) Mola hidatdica.
d) Amenaza de aborto en un tero con miomas.





La enfermedad trofoblstica gestacional agrupa a diferentes entidades
interrelacionadas: mola completa, generalmente diploide con origen cromosmico
paterno, mola parcial generalmente triploide, tumor trofoblstico del lecho
placentario y coriocarcinoma, con tendencias variables a la invasin local y a las
metstasis, cuyo denominador comn es la hipersecrecin de hCG. El
coriocarcinoma es diploide y proviene de ambos progenitores, excluyendo
probablemente su origen directo en la mola completa. El tumor trofoblstico del
lecho placentario est constituido por trofoblasto mononuclear intermedio no
conteniendo vellosidades corinicas e inmunohistoquimicamente caracterizado por
expresar muchas de sus clulas hPL y unas pocas hCG.


Cuadro clnico
Tras un periodo de amenorrea, y a partir del segundo mes, hay un aumento de los
sntomas subjetivos del embarazo, sobre todo nuseas y vmitos o hipermesis en
un 30%. Hay metrorragias irregulares en el 96% de los casos, en principio de
escasa cantidad, pero que se va incrementando. Dicha hemorragia no es continua
sino que se repite cada dos o tres das, de color roja o negruzca, que se produce
por la ruptura de vasos maternos al separarse las vesculas de la decidua.
Ocasionalmente, hay expulsin de restos molares, que lo refiere la paciente como
expulsin de vesculas en el 11 % de los casos y que es patognomnico pero aparece
tardamente. Tambin puede presentar mal estado general, dolor difuso en
hipogastrio y anemia. Esta ltima en relacin con las metrorragias.
Pueden haber signos y sntomas de hipertiroidismo, como taquicardia, sudoracin, y
temblores, en el 7%, y es debido a que la fraccin -hCG es similar a la hormona
TSH. Existen signos de preeclampsia o hipertensin gestacional del primer
trimestre hasta en el 50% de los casos, y signos de insuficiencia respiratoria
aguda en el 2% por embolismo pulmonar de las clulas trofoblsticas, o por la
asociacin entre hipertiroidismo e hipertensin arterial. Como complicaciones
pueden aparecer coagulopatas y metstasis.
A la exploracin el tero esta aumentado de tamao, en el 50% de los casos, por
encima de lo que correspondera a la edad gestacional,
3
de consistencia blanda, sin
signos de actividad fetal a partir de la semana 12 y siempre que se trate de una
mola total. Tambin puede que la paciente presente un tamao uterino menor al
esperado para la edad gestacional.
3
El cuello est cerrado, con metrorragia en
cantidad variable, y raramente se observa la expulsin de vesculas. Pueden haber
quistes teca-lutenicos bilaterales en ovario en cerca del 20% de los casos,
3
debido
al estmulo de la -hCG.

La enfermedad trofoblstica maligna va a cursar con metrorragias por lo general
intensas, hay una elevacin de la -hCG y en la ecografa se objetiva la cavidad
uterina con signos de ocupacin atpica.


Diagnstico
Por la clnica, y pruebas complementarias como la determinacin de la -hCG y la
ecografa. La determinacin de la -hCG se basa en que el trofoblasto produce la
hormona gonadotropina corinica, presentando cifras elevadas, y su cuantificacin
va a servir para diagnstico, valorar el pronstico, y el seguimiento postratamiento.
La ecografa revela un tero aumentado de tamao que no corresponde con la
amenorrea, con ecos en su interior, puntiformes que corresponderan a las
vesculas y que asemejan copos de nieve o panal de abeja. No se aprecia saco
gestacional ni estructuras fetales y, en ambos ovarios se aprecian quistes teca-
lutenicos como formaciones ovricas redondas, econegativas, con mltiples
tabiques en su interior








BIBLIOGRAFA
1. Mazur MT, Kurman RJ. Gestational trophoblastic disease and related
lesions. En: Kurman RJ editor. Blaunsteins pathology of the female genital
tract. 4th ed. New York: Springer-Verlag. 1994, p. 1049-93.
2. Kurman RJ, Young RH, Norris HJ, Main CS, Lawrence WD, Scully RE.
Immunocytochemical localization of placental lactogen and chorionic
gonadotrophin in the normal placenta and trophoblastic tumors, with
emphasis on intermediate trophoblast and the placental site trophoblastic
tumor. Int J Gynecol Pathol 1984; 3: 101-21.
3. Berkowitz RS, Golstein DP. The management of molar pregnancy and
gestational trophoblastic tumours. En Knapp RC, Berkowitz RS, editores.
Gynecologic Oncology, 2nd ed. New York: Mc Graw-Hill 1992, p. 328-38.
4. De Agustn P, Ruiz A, Lpez F, Contreras F. Patologa de la enfermedad
trofoblstica. Simposio Enfermedad Trofoblstica 1972; 79-98.

Salem S. Ultrasound diagnosis of trophoblastic disease. En: Sanders RC, James
AE(Jr) editores. Ultrasonography in Obstetrics and Gynaecology. New York:
Appleton-Century Crofts: 1977; p. 255-66.


54.- Se trata de mujer de 39 aos con embarazo de 32 semanas. Ingresa al
servicio de urgencias por presentar cuadro de 1 da de evolucin por sangrado
transvaginal leve, rojo brillante, sin contractilidad uterina. A.G.O.: G-3, P-0, C-2.
E.F.: IMC: 30 kg/m2, F.U. de 28 cm, fcf presente, especuloscopa crvix cerrado
con huellas de sangrado. USG reporta producto nico vivo con fetometra normal,
lquido amnitico normal y placenta anterior que cubre parcialmente el orificio
cervical interno.
El siguiente paso en la atencin de esta paciente es:

a) Manejo por consulta externa y vigilar sangrado
b) Manejo por consulta externa con uteroinhibidores
c) Ingreso a hospital e inductores de maduracion pulmonar
d) Ingreso a hospital y cesrea.


PLACENTA PREVIA
DEFINICIN:
Es cuando la placenta se implanta sobre o muy cerca del orificio cervical interno y
una parte de la placenta precede a la parte fetal que se presenta.
INCIDENCIA:
Esta es difcil determinar ya que muchos casos pasan desapercibidos, sobre todo
cuando ocurren los abortos en embarazos tempranos.
La prevalencia vara de 1 en 100 a 1 en 850 nacidos vivos, pero solo el 20% total.
Etiologa:
Edad avanzada, multparas, paciente con cesreas previas, paciente con aborto de
repeticin, esto debido a las gestaciones previas.
TIPOS:
Insercin baja.- Es cuando el borde placentario se encuentra en el segmento
inferior a menos de 6 cm del orifico cervical interno.
Marginal.- Es cuando el borde placentario alcanza los mrgenes del orificio cervical
interno.
Parcial. - Es esta la placenta cubre parcialmente el orificio cervical interno.
Total.- La placenta cubre la totalidad del orificio cervical interno an con
dilatacin cervical avanzada.
DIAGNOSTICO:
La caracterstica es el STV de aparicin brusca en forma indolora en el segundo o
tercer trimestre. Frecuentemente hay ausencia de dolor a actividad uterina que
son parmetros para hacer el diagnstico.
La mayor incidencia de sangrado aparece a las 33-34 sdg.

ESTUDIOS DE GABINETE:
El estudio mas utilizado es la ultrasonografa obsttrica.
TRATAMIENTO:
El manejo va a depender de factores como son:
Edad gestacional, magnitud del sangrado, si hay trabajo de parto, variedad de
placenta previa y complicaciones materna.








BIBLIOGRAFA:
1.- Waxler P, Gottesfeld KR. Early diagnosis of placenta previa. Obstet Gynecol
1979;54:231-32.
2.- Cabrero-Roura L. Riesgo elevado obsttrico. Ed. Masson 1996; pp; 109-118.
3.- Patrick J, Placenta Previa, Clinical Obst and Gynecology 1990;33(3): 414-421.

4.- Chapman M, Furtenes ET, Significance of ultrasound in location of placenta in
early pregnancy Br J Obst Gynecol 197;86: 846.












55.- Paciente de 48 aos que presenta sinusorrragia de 3 meses y sin alteraciones
menstruales previas, por lo dems asintomtica. El diagnstico ms probable es:


a) Adenomiosis
b) Plipo endocervical
c) Malformaciones Mllerianas
d) Plipo endometrial


Plipo es toda formacin ssil o pediculada que se fija a la cavidad uterina. Las
formaciones polipoideas dentro del aparato genital femenino se dividen en
endometriales y cervicales. Dentro de los plipos cervicales se diferencian los
ectocervicales y los endocervicales; estos ltimos son considerados como los
verdaderos por estar constituidos por epitelio cilndrico. No se conocen con
certeza los factores asociados a su histognesis (teora ms admitida: hiperplasia
focal de la mucosa endocervical por estmulo estrognico) (1). Los plipos
cervicales se consideran la forma ms frecuente de tumor benigno de crvix.
Constituyen de un 3 a un 10% de las consultas ginecolgicas; presentan mxima
incidencia entre la 4 y 5 dcada de la vida. Algunos estudios demuestran que
hasta en el 56% de casos de mujeres postmenopusicas se encuentran asociados
plipos cervicales y endometriales; es mucho menor esta frecuencia antes de la
menopausia (2).
La clnica de presentacin ms comn es la hemorragia intermenstrual, espontnea
o durante el coito (3). Tambin pueden ser asintomticos (hallazgo casual en
estudio ecogrfico) o, en ocasiones, alcanzar grandes tamaos: llegan a aflorar por
el orificio cervical externo; se denominan "plipos gigantes" cuando superan 2 cm
(4). Para el diagnstico habitualmente se utiliza la ecografa. Los casos
sintomticos se consideran de indicacin quirrgica por su riesgo de degeneracin a
adenocarcinoma cervical.

Clnicamente se manifiesta con sangrado genital fuera de la menstruacin, sobre
todo durante la relacin sexual. Son benignos, no se malignizan pero deben ser
extirpados para controlar la sinusorragia.









56.- Which of the following drugs combination would be most appropriate in the
patient treatment of acute pelvic inflammatory disease?


a) Ampicillin / cefoxitin
b) Tetracycline / gentamicin
c) Cefoxitin / Doxycycline
d) Ampicillin / Amikacin


E.P.I.
Regmenes de tratamiento CDC 2002


Parenteral.
Cefotetan 2gr iv/12h Cefoxitina 2gr iv/6h + Doxiciclina 100 mgs
iv/8h.
Clindamicina 900 mg iv/8h + Gentamicina iv/im (2mg/kg de carga,
luego 1.5 mg/kg/8h.
Alternativas.
Ofloxacina 400 mg iv/12h Levofloxacino 500 mg iv diario con o sin
Metronidazol 500 mg iv/12 h.

Oral.
Ofloxacina 400 mg vo diario x 14 das Levofloxacina 500 mg vo
diario x 14 das con o sin Metronidazol 500 mg vo diario por 14 das.
Ceftriaxona 250 mg im x 1 dosis Cefoxitina 2 gr im x 1 dosis y Probenecid 1 gr vo
x 1 dosis u otra cefalosporina de 3 gen im + Doxiciclina 100 mgs vo diario or
14 das con o sin Metronidazol 500 mgs vo diario x 14 das.




57.- Cuando nos enfrentamos ante un cuadro clnico de amenorrea, galactorrea y
prdida de campo visual el diagnstico ms probable es:


a) Adenoma hipofisario no funcionante.
b) Intoxicacin por benzodiacepinas.
c) Prolactinoma.
d) Meningioma del tubrculo solar.




El prolactinoma es el tumor hipofisario ms frecuente, es de naturaleza benigna y
pequeo en 90 % de los casos. El cuadro clnico tpico en la mujer se compone de
trastornos menstruales, galactorrea y/o esterilidad; se acompaa de sntomas
neurolgicos slo cuando se extiende por arriba de la silla turca. Niveles de
prolactina superiores a 100 ng/mL son prcticamente diagnsticos de prolactinoma,
siempre y cuando no existan embarazo y/o hipotiroidismo. La primera opcin
teraputica del prolactinoma es la farmacolgica con dopaminrgicos, lo que
prcticamente ha eliminado la ciruga. Los dopaminrgicos suprimen la sntesis y
secrecin de prolactina con la consecuente normalizacin del eje hipotlamo-
gonadotrpico. Los dopaminrgicos son efectivos para inducir la ovulacin y
favorecen la consecucin de embarazo. Para cualquier dimensin del prolactinoma
se usan los dopaminrgicos durante uno a dos aos y generalmente despus de
suspenderlos se puede esperar que el tumor se reduzca de tamao y se corrija la
hiperprolactinemia. En contraste, en el hombre generalmente se encuentran
macroprolactinomas, mayores de 10 milmetros con extensin extraselar
acompandose de sntomas neurolgicos; sin embargo, tambin responden
favorablemente a los dopaminrgicos.

Gac Md Mx Vol. 140 No. 5, 2004
Referencias
1. Schlechte JA. Prolactinoma. N Engl J Med 2003;349:2035-2041.
2. Zrate A, Canales ES, Jacobs LS, Soria J, Daughaday WH. Restoration of
ovarian function in patients with the amenorrhea-galactorrhea syndrome after
long-term therapy with L-Dopa. Fertil Steril 1973;24:340.
3. Tyson JE, Carter JN, Andreassen B, Huth J, Smith B. Nursing mediated
prolactin and luteinizing hormone secretion during puerperal lactation. Fertil Steril
1978;30:154.
4. Schlechte JA, Sherman BM, Chapler FK, VanGilder J. Long-term followup of
women with surgically treated prolactin-secreting pituitary tumors. J Clin
Endocrinol Metab 1986;62:1296-301.

5. Losa M, Mortini P, Barzaghi R, Gioia L, Giovanelli M. Surgical treatment of
prolactin-secreting pituitary adenomas: early results and long-term outcome. J
Clin Endocrinol Metab 2002;87:3180-3186.
6. Zrate A, Canales ES, Cano C, Pilonieta CJ. Follow-up of patients with
prolactinomas after discontinuation of long-term therapy with bromocriptine. Acta
Endocrinol 1983;104:139-42.
7. Zrate A, Canales ES, Alger M. The effect of pregnancy and lactation on
pituitary prolactin secreting tumors. Acta Endocrinol 1979;92:407-11.
8. Bevan JS, Webster J, Hburke J, Scanlon MF. Dopamine agonists and
pituitary tumor shrinkage. Endocr Rev 1992;13:220-240





58.- Masculino que se presenta a consulta externa, refiere que su pareja ha sido
diagnosticada por infeccin de virus del papiloma. La forma ms frecuente de la
infeccin por papiloma virus en varones es?

a) Verrugas plantares
b) Verrugas larngeas
c) Subclnica en genitales
d) Condilomas prepuciales



En diferentes estudios de varones compaeros de mujeres con lesiones cervicales
por virus del papiloma humano se demostr que el 88% tenan signos histolgicos
de condilomas, de los cuales el 72% eran en forma subclnica es decir no haba
ningn tipo de lesin visible o sntoma. Otros estudios han demostrado que hasta el
66% o mas de los compaeros de mujeres con neoplasias intraepiteliales cervicales
tienen infecciones subclnicas por virus del papiloma humano en el pene.


Obstet Gynecol 2001;185:308-17.
Kurtycz DFI, Hoerl HD. Thin-Layer Technology: Tempered Enthusiasm. Diag
Cytopath 2000;23(1):1-5.


59.- Femenino de 45 aos de edad que se encuentra en terapia intensiva por
manejo de spsis abdominal. Lleva dos das con tratamiento a base de ceftriaxona,
metronidazol y amikacina, el cual provee de una cobertura conveniente para los
microorganismos probablemente involucrados. Despus de revisar al paciente que

ha tenido una evolucin favorable, usted sugiere que se aplique el aminoglucsido en
monodosis ya que:


a) De esta forma aumenta su potencia.
b) Como tiene un efecto post antibitico corto su efecto es mejor
c) Disminuyen sus efectos adversos
c) Hace sinergismo con el Metronidazol








Una caracterstica farmacolgica de los aminoglucsidos es su actividad
dependiente de concentracin, por lo que su aplicacin en bolos, con lo que se
logran niveles pico ms altos, es de mayor conveniencia para aprovechar este
fenmeno. Adems, el importante efecto post-antibitico que tiene, permiten que
la actividad dure ms all del tiempo en que sus concentraciones se encuentran por
arriba de la mnima inhibitoria, favoreciendo una posologa cmoda cada 24 horas.
Por otra parte, se ha visto que los efectos adversos (nefrotoxicidad y
ototoxicidad), se encuentran ms relacionados con los niveles valle. Estos se
mantienen a niveles bajos cuando se utiliza el esquema en monodosis, disminuyendo
la probabilidad de la presentacin de estos efectos adversos.



Kasper DL, Braunwald E, Fauci AS, Hauser SL, Longo DL, Jameson JL. Harrisons
Principles of Internal Medicine. McGraw Hill. 16 Ed. 789-806 pp.



60.- Masculino de 44 aos diagnosticado por trastorno obsesivo-compulsivo. Los
actos obsesivos ms frecuentes en este trastorno son:

a) Recuentos mentales.
b) Evitar pisar las cruces de las baldosas.
c) Comprobaciones y rituales de limpieza.
d) Acumulacin y coleccin de objetos.
Sntomas del TOC

Obsesiones

Las obsesiones son ideas o impulsos no deseados que aparecen repetidamente en la
mente de la persona que padece TOC. Los pacientes suelen tener miedo a sufrir
dao ellos mismos, o alguien al que quieren, se preocupan irracionalmente por no
contaminarse, o tienen una necesidad excesiva de hacer las cosas correctamente o
con perfeccin. Una y otra vez, la persona piensa algo que le inquieta, como por
ejemplo, "Mis manos pueden estar contaminadas--debo lavarlas" o "Puedo haber
dejado el gas abierto" o "Estoy hacindole dao a mi hijo." Estos pensamientos
angustiosos se inmiscuyen con los otros pensamientos del paciente, y causan
ansiedad. A veces, las obsesiones son de carcter violento o sexual, o tienen que
ver con enfermedades.


Compulsiones
En respuesta a sus obsesiones, la mayora de las personas con TOC recurren a
comportamientos repetitivos llamados compulsiones. Los ms frecuentes son los
rituales de limpieza y las comprobaciones. Otros comportamientos compulsivos
incluyen recuentos (a menudo al mismo tiempo que se realizan otras acciones
compulsivas, tales como lavarse las manos), hacer repeticiones, acaparamiento, y
recolocaciones sin fin de objetos en un esfuerzo para mantenerlos perfectamente
alineados. Tambin son corrientes los problemas mentales, tales como repetir
frases mentalmente y hacer listas. Estos comportamientos, en general, tienen por
objeto proteger de peligros a la persona que padece TOC, o a los otros.
Algunas personas con TOC tienen rituales establecidos; otros tienen rituales que
son complejos y cambiantes.
El ejecutar estos rituales slo proporciona alivio temporal de la ansiedad, pero no
hacerlos incrementa la ansiedad de la persona.


61.- Femenino de 19 aos, atendida en sala de urgencias ginecoobsttricas,
Antecedente: cursa embarazo de 38 SDG. Exploracin Fsica: en trabajo de parto.
Repentinamente presenta sangrado profuso transvaginal y dolor abdominal.
A la paciente se le coloca un monitor fetal externo. El tono uterino est
incrementado y hay desaceleraciones variables y ocasionales de la FCF que llegan a
90. El manejo apropiado en esta paciente es:


a) Tocolisis con un agonista beta adrenrgico
b) Continuar el monitoreo de la madre y el producto
c) Amniotoma
d) Cesrea

El manejo de stas pacientes es estricto y urgente:
1- Ingreso inmediato de la paciente en vigilancia a sala de partos.
2- Venopuntura. Si existen signos de shock, instaurar preferentemente dos vas,
una central y otra perifrica.
3- Solucin de Ringer.
4- Monitoro de signos vitales cada 15.
5- Sonda vesical y monitoro de diuresis. Diuresis de 50 ml/h asegura perfusin
perifrica correcta.
Evitar diuresis inferiores a 30 ml/h.
6- Monitoro contnuo de LCF.
7- Laboratorio de urgencia: hemograma, ionograma, urea y creatinina,
coagulograma.
8- Sangre como mnimo 4U para reserva (concentrado de glbulos rojos o sangre
entera).
9- Ecografa obsttrica para localizar y medir el hematoma y descartar otras
causas de sangrado.
Dependiendo del grado de desprendimiento realizar:
1- Desprendimiento leve: sin compromiso materno ni fetal y el embarazo es de
pretrmino, conducta expectante. Uteroinhibicin y maduracin pulmonar fetal.
2- Desprendimientos moderados a severos: si hay compromiso materno o fetal u
bito fetal, terminar inmediatamente la gestacin.
3- Independientemente de la va del parto, es preceptiva la prctica de la
amniotoma, para reducir la presin intraamnitica y disminuir el paso de
tromboplastina hstica a la circulacin y la extravasacin de sangre al miometrio.
Las principales complicaciones son:
1- Hipotensin, shock hipovolmico.
2- Coagulacin intravascular diseminada.
3- Necrosis cortical y necrosis tubular aguda renal.
Obstetricia. Scwarcz, Sala, Duverges. 7 edic. Edit. El Ateneo. (Biblioteca
Fac. Med. UNNE).



62.- Es el caso de paciente femenino de 18 aos la cual presenta amenorrea
acompaada de profundas alteraciones del olfato. Cul de los siguientes
diagnsticos es el ms probable? :

a) Sndrome de amenorrea-galactorrea.

d) Amenorrea por alteracin hipotalmica.
b) Amenorrea de causa uterina.
c) Sndrome de ovario poliqustico.


AMENORREA HIPOTALAMICA. (Hipogonadotrpica)
Psicgena. (Stress emocional)
Anorexia nerviosa. (Deficiencia nutricional)
Ejercicio excesivo. (Carrera de fondo, natacin, gimnasia, ballet)
Frmacos. (Fenotiazina, reserpina, bloqueadores ganglionares,
anticonceptivos)
Pseudocisis.
Sndrome de Kallman (Deficiencia selectiva de gonadotropinas y
anosmia).
El ejemplo clsico de la alteracin hipotalmica que lleva a desrdenes del ciclo
menstrual es el Sndrome de Kallman.



Lectura Recomendada:
Etiopatogenia de la amenorrea hipotalmica funcional Interaccin de las
respuestas hormonales del Sistema Nervioso Central y Neuropptidos Perifricos

Revista Argentina de Endocrinologa y Metabolismo
Copyright 2008 por la Sociedad Argentina de Endocrinologa y Metabolismo
Vol 45 No. 2



63.- Femenino de 25 aos con antecedentes de G/2, P/1, C/1 acude al servicio de
consulta externa, refiere que presenta una secrecin transvaginal bastante lquida,
de baja viscosidad, maloliente de color amarillo y gris, espumoso.
El tratamiento de eleccin para esta entidad es:

b) Ampicilina 1g VO c/ 6 hrs.
c) Clotrimazol + Metronidazol 500 mg VO c/12 hrs. por 7 a das
a) Metronidazol 500 mg. VO c/12 por 7 a 10 das
d) Clindamicina 300 mg VO cada 12 hrs. x 5 das.





Referencias bibliogrficas:
1. Secretara de Salud. Norma Oficial Mexicana NOM -039-SSA2-2002, Para la
prevencin y
control de las infecciones de transmisin sexual. D.O.F. 19 de Septiembre 2003.
2. Kettler H, White K, Hawkes S. Mapping the landscape for sexually transmitted
infections:
key findings and recommendations. Geneva, TDR (TDR/STI/ IDE/04.1).
3. CDC. Trends in Reportable Sexually Transmitted Diseases in the United States.
CDC,
National Report. 2004
4. Distribucin de los casos nuevos de enfermedades por mes Estados Unidos
Mexicanos
2004. Sistema nico de Informacin para la Vigilancia Epidemiolgica/Direccin
General
de Epidemiologa/SSA
5. Aral S O. Sexual risk behaviour and infection: epidemiological considerations.
Sex. Transm.
Inf. 2004;80:8-12



64. - Femenino de 33 aos gestante, acude a ultrasonido de control. Reportando:
Placenta marginal, embarazo de 37 SDG. Antecedentes: G=V P= IV.
La complicacin esperada en esta paciente es:

a) Desprendimiento de placenta normoinserta.

b) Parto pretrmino.
c) Restriccin en el crecimiento intrauterino.
d) Hemorragia materna.


Placenta previa: La placenta en lugar de estar implantada en el cuerpo del tero
por fuera del orificio cervical interno, se localizaa por encima del orificio interno o
cerca de l. En la placenta marginal el borde la placenta est en el margen del
orificio. La multiparidad y la edad avanzada parecen favorecer la placenta previa.
El hecho ms carcterstico de la placenta previa es la hemorragia no dolorosa, la
cual no suele aparecer hasta casi del final de segundo trimestre o ms tarde.
Complicaciones maternas: Hemorragia y muerte.
Pritchard, Williams Obstetricia, 3Edicin, Salvat editores, pag. 395-399










65.- Femenino de 19 aos acude a consulta por padecer un cuadro de dolor
abdominal de inicio sbito, refiere alteraciones en su ciclo menstrual. La
exploracin fsica revela una tumoracin dolorosa en el anexo izquierdo. La prueba
de embarazo es negativa. La radiografa muestra una masa opaca en la fosa ilaca
izquierda con reas de calcificacin. El diagnstico clnico ms probable es:

a) Cistadenoma mucinoso
b) Teratoma qustico
c) Quiste folicular
d) Cistadenoma seroso

Los tumores de clulas germinales constituyen casi el 20% de los tumores de
ovario y de ellos un 95% son benignos, siendo el tipo ms frecuente el teratoma
maduro benigno o tambin denominado quiste dermoide (1).
Aproximadamente el 80% se presentan en mujeres en edad frtil. Se originan a
partir de clulas embrionarias pluripotenciales presentes habitualmente en ovario,

testculo, mediastino, retroperitoneo y regin sacrocoxgea, esto explica que la
coloracin de las faneras del quiste coincida con el fenotipo del paciente (2).
El teratoma qustico benigno con relativa frecuencia es un tumor bilateral (del 7 al
25% segn los autores) (1) y se caracteriza por una cpsula gruesa, bien formada,
revestida por epitelio plano estratificado. Bajo este se pueden encontrar una
variedad de apndices cutneos que incluyen glndulas sudorparas, apocrinas y
sebceas. La cavidad se llena de los detritus de ste y sus anexos, que es de color
amarillo plido, grasoso, espeso y suele contener pelo. Otros tejidos que se pueden
encontrar son dientes, cartlago, plexos coroideos, falanges, tejido nervioso y en
ocasiones tejido tiroideo (struma ovarii) con potencial tirotxico o de degeneracin
maligna tiroidea.
La mayor parte de los quistes dermoides son asintomticos y la forma ms
frecuente de presentacin son el dolor abdominal (48%) y hemorragia uterina
anormal o concomitante (15%) o aumento del volumen abdominal (15%). La rotura de
un quiste dermoide es rara, entre el 1-1,2% y constituye una urgencia quirrgica
(3).
El tratamiento es quirrgico, siendo posible la mayor parte de las veces una
reseccin del mismo, respetando el resto del ovario.
La ecografa constituye el modelo diagnstico de eleccin y la combinacin de
ecografa con radiografa simple de abdomen proporciona un diagnstico ms
exacto en la mayora de los casos, siendo la resonancia magntica o la TAC el que
aporte el diagnstico diferencial.
Las Rx de quiste dermoide se caracteriza por una cpsula bastante radioopaca y el
lquido oleoso que contiene es radiolcido, esta conjugacin presenta muchas veces
una caracterstica de aspcto radiolgico de calcificacin en la pared del mismo.
Clico nefrtico, teratoma ovrico y radiografa simple de aparato urinario Romero
Prez P, Martnez Hernndez MC.
Servicio de Urologa. Policlnico San Carlos Denia (Alicante).Actas Urol Esp.
2007:31(8):936-937ACTAS UROLGICAS ESPAOLAS SEPTIEMBRE 2007.

66.- Mujer de 23 aos diagnosticada de E. Ectpico a nivel ampular, con saco
gestacional menor de 3 cm., con actividad cardaca embrionaria, asintomtica y con
niveles de B-HCG menores para su edad gestacional. El tratamiento ms indicado
es:

a) Salpingocentesis
b) Reseccin segmentaria

c) Tratamiento mdico con Metrotexate.
d) Histerectoma total con doble anexectoma.

TRATAMIENTO

METOTREXATE
cido 4 amino 10 metil folnico, antagonista del cido flico
Glucosa hiperosmolar
Prostaglandina F2a
Actinomicina D
Mifespristona

TRATAMIENTO CON METOTREXATO
Mayor xito:

Embarazo menor a 6 SDG
Masa tubaria menor de 3.5cm
Feto sin latido cardaco


PROTOCOLO CON DOSIS NICA DE METOTREXATE

Da 0 hGC, Biometra hemtica, transaminasas, creatinina, grupo sanguneo
Da 1 hGC
Da 4 hGC
Da 7 hGC

Disminucin de hCG <15%. Segunda dosis
Si la hCG declina seguirla semanalmente
Si la hCG est en meseta o en ascenso, segunda dosis
El raspado endometrial se realiza slo en pacientes con hCG <2000 mIU/mL
al momento de comenzar el tratamiento

Graczykowski JW, Mishell DR. Methotrexate prophylaxis for persistent ectopic
pregnancy after conservative treatment by salpingostomy. Obstet Gynecol.



67.- Mujer de 26 aos, con tumor anexial de 6 cm lquido, dolor abdominal, fiebre,
leucorrea, con historia de cervicovaginitis de repeticin y dispareunia crnica,
ltima menstruacin hace una semana.
El manejo ms adecuado es:

a) Histerocopa

b) Colposcopa
c) Histerosalpingografa
d) Laparoscopa


CLASIFICACION LAPAROSCOPICA DE LA ENFERMEDAD PELVICA
INFLAMATORIA
El stndar de oro para el diagnstico de EPI es la laparoscopia, ya que adems de
visualizar directamente los rganos plvicos, permite la toma de muestras para
estudios bacteriolgicos.
Los criterios laparoscpicos para el diagnstico de EPI, se describen en la tabla
No.3.
Tabla No. 3
CLASIFICACION LAPAROSCOPICA DE LA EPI
Leve Eritema, edema, las trompas se mueven
libremente. No hay exudado purulento.
Moderada Eritema, edema mas marcado, material
purulento evidente. No hay movimiento
libre de las trompas. La fimbria puede no
ser evidente.
Severa Presencia de pioslpinx y/o absceso

Diagnstico
Dolor abdominal bajo y/o dolor a la movilizacin cervical
y anexial y:
Fiebre de 38 C o mayor
leucocitosis > 10, 500 mm3, eritrosedimentacin y PCR elevada
Bacterias o leucocitos en liquido peritoneal
Masa anexial palpable o por USG
Laparoscopia
Sndrome de Fitz- Hugh-Curtis
EPI





68.- Una mujer de 67 aos refiere intenso prurito vulvar y sensacin quemante, al
examen el introito vaginal se encuentra estentico. Cul de los siguientes es el
tratamiento apropiado?

a) 5-fluoracilo
b) Testosterona tpica
c) Corticoesteroides fluorados
d) Estrgeno tpico


Vulvovaginitis atrfica
El hipoestrogenismo conduce a atrofia de la vagina y el vestbulo vulvar, que los
hace fcilmente irritables y susceptibles a infecciones secundarias. Las pacientes
refieren sensacin de quemadura, prurito, disuria, hipersensibilidad y dispareunia.
Puede encontrarse al examen fsico atrofia, fisuras superficiales, y un flujo vaginal
acuoso
1
. Hay disminucin del tamao del introito
2
, prdida de la rugosidad y la
vagina toma una apariencia lisa y brillante.
Los hallazgos histolgicos revelan un epitelio vaginal delgado, disminucin de los
lechos capilares, y la citologa muestra, a medida que la atrofia progresa, aumento
de las clulas basales y disminucin o ausencia de las clulas superficiales
2
.


Se aconseja evitar el uso de jabones y dems irritantes de la piel. Se pueden
utilizar lubricantes simultneamente con los estrgenos o como terapia nica, si
hay alguna contraindicacin a las hormonas.
El tratamiento con estrgenos por va sistmica o transvaginal mejora y restaura
los signos y sntomas, y una a dos semanas despus de iniciar el tratamiento los
cambios de atrofia empiezan a mejorar rpidamente, se reduce el pH y se induce
maduracin vaginal y de la mucosa uretral, reduciendo la frecuencia de las
infecciones urinarias
3
. La dosis y va de administracin debe ser debidamente
individualizada
4
. Contraindicaciones al tratamiento con estrgenos, incluyen: la
presencia de tumores estrgenosensibles, falla heptica terminal y antecedentes
de tromboembolizacin relacionada con ellos.
Menopausia y Piel. Parte II: Manifestaciones clnicas
dermatolgicas durante la menopausia
MARA ISABEL BARONA C


69.- Femenino de 22 aos de edad, acude al servicio de urgencias por referir
malestar general, fosfenos, nusea y vmito. Antecedentes: cursa con embarazo de
36.5 SDG, G2, A1, C0. Exploracin fsica: TA 185/110 mmHg, FC 120 lpm, FR 35 x,

T37.9, alerta, inquieta, aprecia ictericia, cardiopulmonar sin compromiso, abdomen
globoso a expensas de tero grvido, se detecta PUVI, longitudinal, ceflico, dorso
a la derecha, FCF 142 x, al tacto genital crvix, central, formado, cerrado,
extraccin de guantes sin evidencia de prdidas genitales, extremidades inferiores
edema +++.
El dato de laboratorio que apoya el diagnstico en esta paciente es:

a) Hematuria.
b) Proteinuria.
c) Coluria.
d) Piuria.



Referencia:
La preeclampsia se define por aumento de la presin arterial y la presencia de
proteinuria durante el embarazo.
1) Leve: las pacientes suelen tener unas cuantas manifestaciones, y su presin
arterial diastlica es menor de 110 mmHg. En ocasiones hay edema. La cifra
de plaquetas es mayor de 100 000/l.
2) Grave: los sntomas son ms notorios y persistentes. La presin arterial casi
siempre es con niveles mayores de 160/110 mmHg. Puede haber
trombocitopenia que avance hasta coagulacin intravascular diseminada.
Se requiere hospitalizacin para las mujeres con preeclampsia; debe obtenerse una
biometra hemtica completa con cifra de plaquetas y determinacin de
electrolitos, que adems incluya enzimas hepticas. Se obtiene una muestra de
orina de 24 horas para determinar la depuracin de creatinina y protenas totales
al ingreso hospitalario.
Se debe controlar la hipertensin arterial, para evitar sufrimiento fetal, as como
empeoramiento de la paciente.

Bibliografa:

1. McPhee S, Papadakis M, et. al. Diagnstico Clnico y Tratamiento 2010. Lange,
McGraw Hill,
49 edicin, Mxico, 2010.
2. Sibai BM, Diagnosis, prevention, and management of eclampsia. Obstet
Gynecol. 2005. Feb; 105;: 402 410.


70.- Femenino de 36 aos, es atendida en consulta externa con reporte de
papanicolaou que reporta un NIC I, la especuloscopa se observa crvix con
ectropin periorificiario.
El mtodo ms sensible para corroborar el diagnstico en esta paciente es:


a) Papanicolaou.
b) Colposcopa
c) Exudado vaginal.
d) Prueba de koh.








9.5.2 Las pacientes a quienes se les realiz citologa cervical, cuyo resultado es
LEIBG (infeccin por VPH, displasia leve o NIC 1); LEIAG (displasia moderada y
grave o NIC 2 y 3) o cncer deben enviarse a una clnica de colposcopa, para
realizar estudio colposcpico.
9.5.3 Si el resultado de la citologa es LEIBG, la colposcopa es satisfactoria y sin
evidencia de LEIBG, se realizar control citolgico en un ao (Apndice Normativo
A)
9.5.4 Si la citologa es de LEIBG, la colposcopa es satisfactoria y existe evidencia
de lesin, se debe tomar una biopsia dirigida.
9.5.4.1 Si la biopsia dirigida es negativa, se realizar nueva colposcopa para
verificar el diagnstico y en caso necesario, tomar nueva biopsia dirigida y
revalorar.
9.5.4.2 Si la biopsia dirigida es reportada como LEIBG se podr dar tratamiento
conservador: criociruga, electrociruga o laserterapia (slo si cumple con las
condiciones referidas en el Apndice 1) o se podr mantener a la paciente en
vigilancia en la clnica de colposcopa, con colposcopa y estudio citolgico cada seis
meses, durante 24 meses.
Jueves 31 de mayo de 2007 DIARIO OFICIAL (Primera Seccin)
9.5.4.3 Si la biopsia dirigida es reportada como LEIAG (Lesin Intraepitelial
Escamosa de Alto Grado) se realizar tratamiento conservador (electrociruga o
laserterapia). En las mujeres posmenopusicas, dependiendo de las condiciones
anatmicas del crvix, se realizar tratamiento conservador en la clnica de
colposcopa o tratamiento quirrgico (histerectoma extrafascial) en el servicio que
corresponda.
9.5.4.4 Si la biopsia dirigida reporta cncer microinvasor o invasor, la paciente se
transferir a un Servicio o Centro Oncolgico para su tratamiento
correspondiente.
9.5.4.5 Si la citologa reporta LEIBG y la colposcopa es no satisfactoria, se
tomar cepillado endocervical (Apndice Normativo A)
9.6 En caso de colposcopa no satisfactoria, negativa a LEIBG y con cepillado
endocervical negativo, se continuar su control en la clnica de colposcopa en seis
meses, con colposcopa y citologa.
9.6.1.1 Si el cepillado endocervical reporta LEIBG se tratar a la paciente como
LEIAG, con mtodos conservadores escisionales.



Jueves 31 de mayo de 2007 DIARIO OFICIAL (Primera Seccin)
Modificacin a la Norma Oficial Mexicana NOM-014-SSA2-1994, Para la
prevencin, deteccin, diagnstico, tratamiento, control y vigilancia
epidemiolgica del cncer crvico uterino.
Al margen un sello con el Escudo Nacional, que dice: Estados Unidos Mexicanos.-
Secretara de Salud.

MODIFICACION A LA NORMA OFICIAL MEXICANA NOM-014-SSA2-1994,
PARA LA PREVENCION,
DETECCION, DIAGNOSTICO, TRATAMIENTO, CONTROL Y VIGILANCIA
EPIDEMIOLOGICA DEL CANCER CERVICO UTERINO.


71.- Una mujer de 23 aos de edad gesta 1, tuvo un parto vaginal espontneo con un
producto con peso de 4,350 g. despus de 5 minutos de traccin suave del cordn
umbilical se expuls la placenta, que parece estar intacta. Se inici el masaje del
fondo uterino y se pidi a la enfermera que administrara 20 unidades de oxitocina
en 100 ml de solucin Ringer lactato. Despus de una inspeccin cuidadosa del canal
del parto se observa una laceracin de segundo grado y una laceracin de 2 cm en
la pared vaginal izquierda que se intent reparar. En la exploracin fsica se
encuentra un fonso uterino blando y atnico. Los signos vitales son: temperatura
37.1C, TA 164/92, FC 130x, FR 18 X. Cul de los siguientes es el mejor
tratamiento?

(a) Oxitocina 10 unidades directas en goteo intravenoso

(b) Metilergonovina 0.2 mg IM

(c) Prostaglandina F 0.25 mg IM

(d) Legrado





Morgan M, Siddighi S. Ginecologa y obstetricia, National Medical Series. 5
edicin. Mc Graw Hill. Pp. 28. La atona uterina es la causa ms comn de
hemorragia puerperal. El masaje energtico y la oxitocina diluida no han sido tiles
para interrumpir la hemorragia y por tanto el siguiente paso es agregar un frmaco
uterotnico. La metilergonovina est contraindicada porque la paciente se
encuentra hipertensa a pesar de la hemorragia intensa, el siguiente frmaco es la
prostaglandina. La administracin de oxitocina no diluida, 10 UI por va IV podra
causar hipotensin grave. La exploracin manual podra ser apropiada si se
sospecha laceracin como causa de hemorragia. El legrado es apropiado para la
hemorragia puerperal tarda, cuando se sospecha retencin de los productos de la
concepcin.




72.- Mujer de 24 aos la cual presenta un ndulo mamario palpable de aparicin
brusca. La ecografa revela un ndulo anecognico, de limites muy precisos,
morfologa regular y refuerzo posterior, nico de 3.5 cms. de dimetro. El
diagnstico ms probable es?:

a) Cncer.
b) Displasia fibrosa.
c) Fibroadenoma.
d) Quiste




Quistes. Los quistes mamarios son fciles de detectar con la ecosonografa.
Pueden ser lesiones nicas o mltiples que se observan como imgenes
redondeadas, anecognicas, de paredes delgadas, contornos bien definidos, con
importante reforzamiento acstico posterior y sombras laterales delgadas. Pueden
presentar septos intraqusticos y, en ocasiones, se pueden observar ecos internos
que sugieren detritus celulares o proceso inflamatorio. Se debe descartar la
presencia de lesiones intraqusticas o la coexistencia de otras alteraciones
benignas o malignas. En caso de ser sintomticos, el tratamiento adecuado es la
puncin y aspiracin de la lesin con aguja guiada por palpacin o ecosonografa de
acuerdo con ell tamao, profundidad y caractersticas del contenido. El uso del
ultrasonido garantiza el vaciamiento completo.










REFERENCIAS
Barth V, Prechtel K. Mama normal. En: Barth V, Prechtel K, editores. Atlas de
patologa de la glndula mamaria.
2da ed. Madrid: Editorial Mdica Panamericana, 1991.
Bush H, McCredie A. Carcinoma of the breast during pregnancy and lactation. In:
Allen HH, Nisker JA. Cancer in pregnancy. New York: Futura Publishing Co. Inc.,
1986.
Byrd BF, Bayer DS, Robertson JC, Stephenson JE Jr.
Treatment of breast tumor associated with pregnancy and lactation. Ann Surg.
1962; 155:940-7.













73.- Femenino de 51 aos, con mioma uterino de tamao equivalente a una gestacin
de 12 semanas, que presenta hipermenorreas y hemoglobinemia de 9 gr%. No se
demuestra patologa asociada. Se encuentra en espera para la prctica de una
histerectoma programada a realizar en 4 meses. En esta paciente est indicado el
tratamiento preoperatorio con:

a) Estrgenos.
b) Inhibidores de la fibrinlisis.
c) Derivados del cornezuelo del centeno.
d) Anlogos de la GnRH


Anlogos de la GnRH: Son derivados de la hormona GnRH en donde se ha realizado
una sustitucin peptdica en posicin 6 y en algunos casos en la 10, obteniendo
compuestos hasta unas 200 veces ms potentes debido a mayor afinidad por los
receptores y a su resistencia a la degradacin por peptidasas. Aunque su accin
inicial produce un incremento en la produccin de FSH-LH (efecto flure-up o
llamarada) tras 5-
6 das de exposicin contnua, los receptores son internalizados producindose un
estado de hipogonadismo hipogonadotropo y niveles de estradiol similares a los de
la postmenopausia. Los anlogos de GnRH estn disponibles en distintas frmulas:
administracin nasal (varias aplicaciones al da), subcutnea (aplicacin diaria) o
intramuscular (preparados depot mensuales o trimestrales) (Shaw RW 1999).
Marco Filicori y sus colaboradores de la Universidad de Bolonia fueron los
primeros en utilizar en 1983 los aGnRH en un estudio que confirm su eficacia para
reducir el tamao de los miomas uterinos y secundariamente sntomas como
alteraciones menstruales, dolor plvico y sntomas de presin local. Otros autores
como Minaguchi H y colaboradores continan comprobando la efectividad de los
anlogos de la GnRH en el tratamiento del mioma uterino tras evaluar en el ao
2000 seis estudios con un total de 602 pacientes tratadas con nafarelina. La
disminucin del tamao se calcula entre un 30-70%, y se ha observado como el
mayor porcentaje de reduccin ocurre tras el primer mes de tratamiento, no
existiendo reducciones o siendo stas mnimas despus del tercer mes
(Healy et al 1986; Friedman et al 1989; Matta et al 1989; Williams y Shaw 1990).
En miomas pediculados o con gran proporcin de calcio o colgeno (hialinizacin) la
repuesta es tambin menor. Debe tenerse en cuenta que si despus de dos meses
de tratamiento no se ha producido un significativo descenso del tamao del mioma,
ste ya no debe ser esperado y debe pensarse en la posibilidad de la existencia de
un tumor muscular maligno no diagnosticado (Messia AF et al 1998). Tras finalizar
el tratamiento y recuperarse el estado de hipogonadismo, el mioma retorna
rpidamente a su tamao inicial (Friedman AJ et al 1987; Matta WH et al 1989).


En casos prximos a la menopausia, la reduccin del tamao del mioma y su
sintomatologa, permitira hablar de una solucin mdica del problema, pero en todo
caso la utilizacin de aGnRH facilitara la intervencin quirrgica al acortar el
tiempo de intervencin, la hemorragia y el acceso a localizaciones complicadas
como el caso de miomas interligamentarios o situados en istmo o crvix. En el caso
de la ciruga histeroscpica la reduccin del tiempo de ciruga permitira reducir el
volumen de fluidos aportado a cavidad uterina y los riegos de absorcin e
hiponatremia.
Los mecanismos de accin por los que los aGnRH actan son: o Hipoestrogenemia:
es necesario mantener la hipoestrogenemia, pues la elevacin de sus niveles lleva a
un rpido incremento del tamao del mioma. El crecimiento del mioma es
dependiente de los niveles de estrgenos (aumentan de tamao con el embarazo y
se reducen durante la menopausia o el tratamiento con aGnRH, pudiendo volver a
crecer durante la THS), pero aunque los estrgenos parecen ser importantes en el
crecimiento del mioma, su relacin debe ser algo ms compleja pues no se han
descrito incrementos significativos del tamao de miomas durante el tratamiento
con gonadotrofinas en RA (situaciones con elevados niveles de E2), algunos de ellos
no se modifican durante el embarazo o incluso decrecen y se han encontrado
crecimientos despus del tratamiento con citrato de clomifeno (antiestrgeno). En
relacin con la hipoestrogenemia podran estar los cambios inducidos en el flujo
vascular uterino (incrementos en el ndice de resistencia de las arterias uterinas)
que suponen una reduccin de la vascularizacin o las modificaciones de distintos
factores de crecimiento. o Cambios histolgicos: el tratamiento con aGnRH puede
producir degeneracin roja, infiltracin linfocitaria, y necrosis, as como reduccin
de la proliferacin celular e incremento de la apoptosis. Pero en otras
circunstancias no es posible encontrar 7 diferencias. No se ha encontrado una
relacin entre los cambios histolgicos y el porcentaje de reduccin del tamao del
tero, y existe una gran variabilidad entre distintas pacientes o entre distintos
miomas de una misma paciente, no existiendo pues un patrn histolgico
caracterstico de respuesta ante el tratamiento con aGnRH.

BIBLIOGRAFA
1. Abad L, Abad de Velasco L, Parilla JJ. Etiopatogenia. Papel de las hormonas
esteroideas, factores de crecimiento y otras sustancias. Cuad Med Reprod
1999;5(1):15-29.
2. Albano C, Platteau P, Devroey P. Gonadotropin-releasing hormone antagonist:
how good is the new hope? Curr Opin Obstet Gynecol
2001;13(3):257-62.
3. Coutinho EM.Treatment of large fibroids with high doses of gestrinone.
Gynecol Obstet Invest 1990;30(1):44-47.
4. Chavez NF, Stewart EA. Medical treatment of uterine fibroids. Clin Obstet
Gynecol 2001;44(2):327-84.
5. De Leo V, la Marca A, Morgante G. Shortterm treatment of uterine
fibromyomas with danazol. Gynecol Obstet Invest 1999;47(4):258-262.

6. Eldar-Geva T, Healy DL. Other medical management of uterine fibroids.
Baillieres Clin Obstet Gynaecol 1998;12(2):269-88.
7. Felberbaum RE, Germer U, Ludwig M, Riethmuller-Winzen H, Heise S,
Buttge I, Bauer O, Reissmann T, Engel J, Diedrich K. Treatment of uterine
fibroids with a slow-release formulation of the gonadotrophin releasing hormone
antagonist Cetrorelix.HumReprod 1998;13(6):1660-8.



74.- Al realizar el diagnstico de hirsutismo asociado a ovario poliqustico, usted
elige el siguiente frmaco para su tratamiento por ser el ms adecuado:


a) Clomifeno
b) Acetato de ciproterona
c) Estrgenos
d) Corticoide


El acetato de ciproterona parece ser ms efectivo que otros frmacos para el
hirsutismo en mujeres causado por la produccin ovrica excesiva de andrgenos
Una de las causas de hirsutismo (crecimiento piloso excesivo) en mujeres es la
hiperproduccin de andrgenos a partir del ovario. Varios frmacos pueden
utilizarse para contrarrestar los efectos del andrgeno. El acetato de ciproterona
es un frmaco antiandrognico. Los efectos adversos informados con su uso fueron
aumento de peso, depresin, fatiga, sntomas mamarios y disfuncin sexual. La
revisin de los ensayos encontr que el acetato de ciproterona parece ejercer un
efecto en el hirsutismo similar a otros frmacos utilizados para el tratamiento del
hirsutismo por exceso de andrgenos. No existen pruebas suficientes para
comparar los efectos adversos de las opciones de tratamiento.

Van der Spuy ZM, le Roux PA. Acetato de ciproterona para el hirsutismo (Revisin
Cochrane traducida). En: La Biblioteca Cochrane Plus, nmero 4, 2007. Oxford,
Update Software Ltd. Disponible en: http://www.update-software.com. (Traducida
de The Cochrane Library, 2007 Issue 4. Chichester, UK: John Wiley & Sons, Ltd.).










75.- Femenino 40 aos, G-3 P-2 A-1, se detecta anemia ferropnica, de 9.5 g/dl,
refiere ciclos menstruales de 31,32 x 8,9 das de duracin, acompaados de
cogulos, los cuales aparecieron despus del nacimiento de su segundo hijo hace 12
aos. e.f.: con ligera palidez de tegumentos, S/V dentro de los parmetros
normales, , genitales con evidencia de sangrado activo, al tacto vaginal bimanual se
detecta tero de consistencia firme voluminoso, irregular, aproximadamente de 12
cm. anexos libres. El tratamiento para esta paciente es:


a) Histerectoma total sin conservar anexos.
b) Histerectoma total conservando anexos.
c) Histerectoma vaginal.
d) Histerectoma radical.


Tratamiento: La ciruga es el tratamiento ms comn del mioma uterino:
Miomectomia: Extirpar slo el mioma, los tumores nicos y accesibles, la reseccin
histeroscpica de miomas submucosos tambin es posible realizarlo con
electrocoagulacin en pacientes con hemorragias (>90%).
Histerectoma:
La ciruga puede ser abdominal o laparoscpica, la decisin final de la tcnica
depender de la eleccin del cirujano, en base al caso individual y a su experiencia,
el procedimiento puede ser:
-histerectoma subtotal.
-histerectoma total (remocin cervical). Tcnicamente ms sencillo en su
realizacin. La incidencia de carcinoma con origen en el mun cervical es menor al
1% en nuestros das.






Bibliografa:
1. Hanafi m. predictors of leiomyoma recurrence after myomectomy. am coll obstet
ginecol 2005;
105: 877-880.
2. Inclan j, mojarra j. miomectoma histeroscpica. abordaje actual para el manejo
de los miomas
submucosos. reporte de un caso y revisin de la literatura. bol clin hosp. infant edo
son
2001;18: 29-34.

3. Goldrath mh, husain m. the hysteroscopic management of endometrial
leiomyomatosis. j am
assoc gynecol laparosc 1997; 4: 263-267.
4. Clement pb, scully re. mullerin adenofibroma of the uterus with invasion of
myometrium and pelvic veins. int j gynecol pathol 1990; 9: 363-371.
5. Ravina jh, herbreteau d, ciraru-vigneron n, bouret jm, houdart e, aymard a,
merland jj. arterial embolization to treat uterine myomata. lancet 1995; 346: 671-
672.
6. Kunhardt-urquiza e, cruz si, fernndez-martnez rl, hernndez-ziga ve:
miomatosis de localizacin poco frecuente. ginecol obstet mex 1997; 65: 541-544.


76.- A 20-year-old man comes to the physician he has noticed blood in his urine on
several occasions in the past year. Each episode of hematuria occurred in
association with an upper respiratory tract infection or a flulike illness. Physical
examination is unremarkable. A urine dipstick test shows mild proteinuria and
microhematuria. Serum levels of electrolytes, creatinine, and blood urea nitrogen
are within normal limits. Serum levels of IgA are elevated. Which of the following
is the most likely diagnosis?


a) Berger disease
b) Goodpasture syndrome
c) Henoch-Schnlein purpura
d) Postinfectious glomerulonephritis

HEMATURIA GLOMERULAR AISLADA PERSISTENTE:

Definicin

Presencia de hematuria de origen glomerular (con acantocitos, y ocasionalmente
cilindros hemticos), sin otro elemento de inflamacin glomerular, es decir, sin
HTA, ni edema, con proteinuria menor a 1 gm /24 hs, y funcin renal normal,
estable.

En caso de hematuria persistente, aislada, las posibilidades principales es que se
trate de Glomerulonefritis por IgA (lo ms frecuente), enfermedad de
membrana basal fina, Sd. de Alport, (raro) o el inicio de una glomerulonefritis
crnica (< probable) (15) .Tambin puede tratarse de una GN Post-Streptoccica,
con persistencia de hematuria.

A continucacin se describirn brevemente:
Nefropata por IgA: Enfermedad de Berger y sndrome de Shnlein-Henoch, se
considern por algunos como espectro de una misma enfermedad, siendo Berger la

forma limitada al rin.
Es la Glomerulonefritis aguda ms frecuente. Se da entre los 15 y 35 aos, y es
3 veces ms frecuente en hombres, tiene tres patrones clnicos de presentacin:

1. Como hematuria macroscpica, 24 48 hs. posterior a cuadro respiratorio alto,
asociado a dolor lumbar. Hematuria dura 2 6 das, rara vez presenta HTA o
deterioro de funcin renal, y tiene proteinuria mnima. Recurre hasta en un 50%.
De esta forma se presenta un 40-50%.

2. Como hematuria microscpica, con proteinuria leve, detectada solo en Sed.
Orina, por rutina. 30 40 % se presenta as.

3. Como Sd. Nefrtico (HTA, Edema, Hematuria), pudiendo evolucionar algunos de
estos casos como GNRP. Se presentan as un 10 %.
Generalmente cursan con proteinuria menor a 1-2 gm/24 hs,. Pocos desarrollan Sd.
nefrtico (10 %)



Existe un grupo de pacientes (20 40 %), que evoluciona con falla renal progresiva,
llegando a IRT en 5 25 aos. Se ha detectado como factor de riesgo de esta
evolucin la presencia de Sd. nefrtico o proteinuria > 1gm/24h, , edad, HTA,
deterioro de la funcin renal, ausencia de hematuria macroscpica. Si no tiene
ninguno de estos factores, tiene poca probabilidad de desarrollar falla renal, y no
se ha demostrado que el tratamiento, en este grupo de bajo riesgo, altere la
evolucin de la enfermedad.


Al laboratorio, el complemento es generalmente normal, rara vez elevado. Existe
aumento de IgA circulante entre un 30 50 %, sin ser especfico de esta
enfermedad.
Para el diagnstico, se requiere de biopsia renal, donde se identifican por
inmunohistoqumica, la presencia de depsitos de IgA. Biopsia de piel carece de
especificidad y de sensibilidad para diagnstico de enfermedad de Berger.

BIBLIOGRAFA

1.Jennette C, Falk R: Small Vesel Vasculitides. N Engl J Med 337:1512, 1997
2.Kashtan, CE Alport Sndrome and thin glomerular basement Membrane disease.
JAM Soc Nephrol 1998;9:1736.
3.Falk R et al: Primary glomerular disease, en Breneer & Rectors, The Kidney, 6a
ed, BM Brenner (editor). Phyladelfia, Saunders, 2000. pp 1263-1349.
4.Hricik, DE et al: Glomerulonephritis. N Eng J Med 339:889,1998
5.Antony BF. Attack rates of acute nephritis after type 49 streptoccocal

infection of the skin and of the respiratory tract. J Clin Inv, 1969;48:1697.
6.Oliviera DBG. Poststreptococcal glomerulonephritis:getting to know an old
enemy. Clin Rxp Immunol 1997;107:8-10
7.LangeK, et all. Evidence for the in situ origin of poststreptococcal
glomerulonephritis: glomerular localization of endpstreptosin and the clinical
significance of the subsequent antibody response. Clin Nephrol 1983;19:3-10
8.Rodriguez-Iturbe, B. Epidemic poststreptococcal glomerulonephritis, Kidney int
1984, 25:129.
9.Tejani A. Poststreptococcal glomerulonephritis: current clinical and pathologic
concepts. Nephron 1990;55:15.
10. Potter EV. Twelve to seventeen year follow up of patients with
poststreptococcal acute glomerulonephritis in Trinidad. N Engl J Med 1982;
307:725-9.




77.- Mujer de 22 aos que acude a consulta por prdida de peso, diarrea,
palpitaciones, temblor y labilidad emocional. No tiene antecedentes importantes.
Refiere evacuaciones diarreicas de 5 a 7 por da, sin moco o sangre.
EF: peso 52 kg, estatura 1.59, FC 108x, TA: 120/65, retraccin palpebral,
hiperemia conjuntival y de carncula, cuello con tiroides aumentada de tamao 3
veces de forma difusa, extremidades superiores con hiperhidrosis palmar, temblor
fino distal, reflejos exaltados
Los estudios que debemos solicitar para hacer el diagnstico es:

a) Coproparasitoscpico, coprolgico y coprocultivo
b) Ultrasonido tiroideo
c) Electrocardiograma y ecocardiograma
d) Pruebas de funcin tiroidea




EVALUACIN DE UN PACIENTE CON SOSPECHA DE
ENFERMEDAD TIROIDEA








78.- Femenino de 34 aos la cual labora en lavandera, hace varios aos presenta
enrojecimiento de los pliegues proximales de las uas de varios dedos de las manos,
que ocasionalmente le supuran. Cul es el diagnstico ms probable?:

a) Dermatitis de contacto.
b) Liquen plano.
c) Dermatoficia.

d) Paroniquia candidisica crnica.




Infeccin por Candida del lecho ungueal que se presenta con mayor frecuencia
como una onicolisis asociada con paroniquia, aunque tambin se observa la
destruccin completa del lecho ungueal y la erosin de la zona distal y lateral de
las uas de los dedos, sin distrofia ungueal total. La perionixis candidisica se trata
de la inflamacin con eritema, edema, dolor y salida ocasional de pus blanquecino
escaso y espeso del reborde ungueal de uno o varios dedos de la mano. Con
frecuencia, se acompaa de onicopata (discoloracin, onicolisis, distrofia total,
etc.).
Predisponentes: Todas las labores manuales que generen humedad. (Muy frecuente
en amas de casas, trabajadores de restaurantes, lavanderas, etc.)

BIBLIOGRAFA:

1. Crespo Erchiga V. Protocolo diagnstico de contaminantes. En "Micologa para
dermatlogos" Ed.
Janssen, Madrid, 1994, pp:49-70.
2. Crespo V, De Lus B, Delgado V, Crespo A y Vera. Espectro etiolgico de las
onicomicosis en nuestro medio. CO7. II Congreso Nacional de Micologa. Santiago
de Compostela. Junio, 1994.
3. Crespo Erchiga V, Delgado Florencio V y Martnez Garca S. Micologa
dermatolgica. Ed. M.R.A. Barcelona, 2006.
4. Daniel III CR. The Diagnosis of Nail Fungal Infection. Editorial Arch Dermatol
1991;127:1566-1567.
5. Delgado Florencio V. Protocolo de identificacin de dermatofitos. En "Micologa
para dermatlogos".
Ed. Janssen, Madrid, 1994, pp:27-41.
6. Delgado Florencio V. Estrategia en el diagnstico y tratamiento de las micosis
superficiales. Ed.
Aula Mdica, Madrid, 1994.
7. Delgado V, Abad Romero-Balmas J, Armijo Moreno M y Dulanto F.
Scopulariopsis brevicaulis como agente de onicomicosis. Actas Deermo-Sif. 1976;
9-10:693-700.
8. English MP. Nails and fungi. Br J Dermatol 1976; 94:697-701.
9. Fevilhade de Chauvin M. Onicomicosis. Dermatologa prctica. 1994; 9:1-2.







79.- Masculino diagnosticado con sfilis congnita tarda, una de las
manifestaciones de esta infeccin es:

a) Hepatomegalia
b) Rinorrea
c) Queratitis
e) Pseudoparlisis de Parrot

La sfilis congnita tarda se manifiesta despus de los 2 aos de vida, el sntoma
ms frecuente es la queratitis, que es parte de la trada de Hutchinson (queratitis,
hipocausia y dientes en tonel o Hutchinson), as como deformidades seas (tibias
en sable, frente olmpica, engrosamiento clavicular), rodilla de clutton, paresia
juvenil y tabes dorsal. La hepatomegalia, rinorrea, roseola sifiltica y la
pseudoparlisis de parrot son manifestaciones de la sfilis precoz que se
manifiesta en los primeros dos aos de vida.

Evans HE, Frenkel LD: Congenital Syphilis. Clin Perinatol 1994; 21(1)
Honeyman: Manual de Dermatologa, 2da ed. 1988; pg. 46
Lukehart SA, Holmes KK: Sfilis. en: Principios de Medicina Interna. 12 ed. Ed.
Harrison. Cap. 128, pg. 765.


80.- Masculino de 44 aos que cursa con infeccin aguda por VIH, ste paciente
clnicamente se presenta con:


a) Fiebre, adenopatias, faringitis y rash
b) Cefalea, alteraciones gastrointestinales y visceromegalias
c) Adenopatas, cefalea y diarrea
d) Rash, prurito y cefalea



Manifestaciones clnicas en la fase aguda VIH.
Los principales sntomas y signos son:
Fiebre y/o sudoracin, 97%
Adenopatas, 77%
Odinofagia (dolor al tragar), 73%
Erupcin cutnea, 70%
Artralgias y mialgias (dolor de articulaciones y msculos), 58%
Trombopenia, 51%

Leucopenia, 38%
Diarrea, 33%
Cefalea (dolor de cabeza), 30%
Elevacin de las transaminasas, 23%
Anorexia, nuseas o vmitos, 20%
Hepato y/o esplenomegalia (aumento tamao de hgado o bazo), 17%
Referencias:
Ioannidis JP, Contopoulos-Ioannidis DG. Chemokine receptor genotypes and
HIV disease progression: A preliminary meta-analysis [Abstract 13311]. 12
Conferencia Mundial del SIDA, Ginebra, Suiza.
Jesper Eugen-Olsen J, Iversen AKN, Benfield TL, et al. Chemokine
receptor CCR2b 64I polymorphism and its relation to CD4+ T cell counts
and disease progression in a Danish cohort of HIV infected individuals
[Abstract 13327]. 12 Conferencia Mundial del SIDA, Ginebra, Suiza.
Kupfer B, Kaiser R, Matz B, Brackmann HH, Schneweis KE: The homozygous
defect in the CCR5 gene gives a high degree of protection against HIV
infection also when inoculated parenterally [Abstract 23255]. 12
Conferencia Mundial del SIDA, Ginebra, Suiza.
Kaiser R, Kupfer B, Rockstroh JK et al: CCR5-genotype and CC-chemokine
levels are not significantly different in infected and uninfected recipients
of a HIV-1 contaminated clotting factor concentrate. [Abstract 13380]. 12
Conferencia Mundial del SIDA, Ginebra, Suiza.

S-ar putea să vă placă și